Вы находитесь на странице: 1из 195

ADVANCED FINANCIAL ACCOUNTING AND REPORTING

Problem Portion

Numbers 1 and 2 (Partnership Formation)

A, B and C decided to form ABC Partnership. It was agreed that A will contribute an equipment
with assessed value of P100,000 with historical cost of P800,000 and accumulated depreciation
of P600,000. A day after the partnership formation, the equipment was sold for P 300,000.

B will contribute a land and building with carrying amount of P1,200,000 and fair value of
P1,500,000. The land and building are subject to a mortgage payable amounting to P300,000 to
be assumed by the partnership. The partners agreed that B will have 60% capital interest in the
partnership. The partners also agreed that C will contribute sufficient cash to the partnership.

1. What is the total agreed capitalization of the ABC Partnership?


A. 1,500,000
B. 2,000,000
C. 2,500,000
D. 3,000,000

2. What is the cash to be contributed by C in the ABC Partnership?


A. 500,000
B. 600,000
C. 700,000
D. 800,000

Numbers 3 and 4 (Partnership Operation – Capital Account Transactions)

On January 1, 2018, A, B and C formed ABC Partnership with total agreed capitalization of
P1,000,000. The capital interest ratio of the ABC Partnership is 5:1:4 while the profit or loss
ratio is 3:2:5, respectively for A, B and C.

During 2018, A and B made additional investments of P200,000 and P500,000, respectively. At
the end of 2018, B and C made drawings of P300,000 and P100,000, respectively. On December
31, 2018, the capital balance of B is reported at P200,000.

3. What is the net income or net loss of ABC Partnership for the year ended December 31,
2018?
A. 500,000 loss
B. 1,000,000 loss
C. 800,000 income
D. 1,200,000 income

4. What is the capital balance of C on December 31, 2018?


A. 150,000
B. 50,000
C. 200,000
D. 250,000

Page 2

Numbers 5, 6, and 7 (Partnership Operation – Distribution of profit or loss)

On January 1, 2018, A, B and C formed ABC Partnership with original capital contribution of
P300,000, P500,000 and P200,000. A is appointed as managing partner.

During 2018, A, B and C made additional investments of P500,000, P200,000 and P300,000,
respectively. At the end of 2018, A, B and C made drawings of P200,000, P100,000 and
P400,000, respectively. At the end of 2018, the capital balance of C is reported at P320,000. The
profit or loss agreement of the partners is as follows:

 10% interest on original capital contribution of the partners.


 Quarterly salary of P40,000 and P10,000 for A and B, respectively.
 Bonus to A equivalent to 20% of Net Income after interest and salary to all partners
 Remainder is to be distributed equally among the partners.

5. What is the partnership profit for the year ended December 31, 2018?
A. 900,000
B. 1,020,000
C. 1,050,000
D. 960,000

6. What is A’s share in partnership profit for 2018?


A. 190,000
B. 340,000
C. 540,000
D. 200,000

7. What is B’s share in partnership profit for 2018?


A. 200,000
B. 290,000
C. 50,000
D. 90,000

Page 3

Number 8 (Admission of partner by purchase)

On December 31, 2018, the Statement of Financial Position of ABC Partnership provided the
following data with profit or loss ratio of 1:6:3:
Current Assets 1,000,000 Total Liabilities 600,000
Noncurrent Assets 2,000,000 A, Capital 900,000
B, Capital 800,000
C, Capital 700,000

On January 1, 2019, D is admitted to the partnership by purchasing 40% of the capital interest of
B at a price of P500,000.
What is the capital balance of B after the admission of D on January 1, 2019?
A. 540,000
B. 480,000
C. 420,000
D. 300,000

Number 9 (Retirement of partner)

On December 31, 2018, ABC Partnership’s Statement of Financial Positions shows that A, B and
C have capital balances of P500,000, P300,000 and P200,000 with profit or loss ratio of 1:3:6.
On January 1, 2019, C retired from the partnership and received P350,000. At the time of C’s
retirement, an asset of the partnership is undervalued.

What is the capital balance of A after the retirement of C?


A. 462,500
B. 537,500
C. 562,500
D. 525,000

Number 10 (Retirement of partners)

On December 31, 2018, ABC Partnership’s Statement of Financial Position shows that A, B and
C have capital balances of P400,000, P300,000 and P100,000 with profit or loss ratio of 1:4:5.
On January 1, 2019, C retired from the partnership and received P80,000. At the time of C’s
retirement, the assets and liabilities of the partnership are properly valued.

What is the capital balance of B after the retirement of C?


A. 284,000
B. 308,000
C. 316,000
D. 320,000
Page 4
Number 11 (Partnership Dissolution – Admission of New Partner by Investment)

On December 31, 2018, the Statement of Financial Position of ABC Partnership provided the
following data with profit or loss ratio of 1:6:3:
Current Assets 1,300,000 Total Liabilities 300,000
Noncurrent Assets 2,000,000 A, Capital 1,400,000
B, Capital 700,000
C, Capital 900,000
On January 1, 2019, D is admitted to the partnership by investing P1,000,000 to the partnership
for 20% capital interest.
If the all the assets of the existing partnership are properly valued, what is the capital balance of
C after the admission of D?
A. 960,000
B. 900,000
C. 840,000
D. 1,200,000

Numbers 12 and 13 (Admission of new partner by investment)

On December 31, 2018, the Statement of Financial Position of ABC Partnership provided the
following data with profit or loss ratio of 5:1:4:
Current Assets 1,500,000 Total Liabilities 500,000
Noncurrent Assets 2,000,000 A, Capital 1,100,000
B, Capital 1,200,000
C, Capital 700,000
On January 1, 2019, D is admitted to the partnership by investing P500,000 to the partnership for
10% capital interest. The total agreed capitalization of the new partnership is P3,000,000.

12. What is the capital balance of D after his admission to the partnership?
A. 500,000
B. 300,000
C. 350,000
D. 400,000

13. What is the capital balance of C after the admission of D to the partnership?
A. 580,000
B. 820,000
C. 500,000
D. 780,000

Page 5

Numbers 14 and 15 (Partnership Liquidation – Lump Sum Liquidation)

On December 31, 2018, the Statement of Financial Position of ABC Partnership with profit or
loss ratio of 6:1:3 of partners A, B and C respectively, revealed the following data:

Cash 1,000,000 Other Liabilities 2,000,000


Receivable from A 500,000 Payable to B 1,000,000
Other noncash assets 2,000,000 Payable to C 100,000
A, Capital 700,000
B, Capital (650,000)
C, Capital 350,000

On January 1, 2019, the partners decided to liquidate the partnership. All partners are legally
declared to be personally insolvent. The other noncash assets were sold for P1,500,000.
Liquidation expenses amounting to P100,000 were incurred.

14. How much cash was received by B at the end of partnership liquidation?
A. 250,000
B. 150,000
C. 290,000
D. 270,000

15. How much cash was received by C at the end of partnership liquidation?
A. 270,000
B. 150,000
C. 350,000
D. 220,000

Page 6

Numbers 16, 17 and 18 (Partnership Liquidation – Installment Liquidation)

On December 31, 2018, the Statement of Financial Position of ABC Partnership with profit or
loss ratio of 5:3:2 of respective partners A, B and C. showed the following information:

Cash 1,600,000 Total Liabilities 2,000,000


Noncash assets 1,400,000 A, Capital 100,000
B, Capital 500,000
C, Capital 400,000

On January 1, 2019, the partners decided to liquidate the partnership in installment. All partners
are legally declared to be personally insolvent.
As of January 31, 2019, the following transactions occurred:
 Noncash assets with a carrying amount P1,000,000 were sold at a gain of P100,000.
 Liquidation expenses for the month of January amounting to P50,000 were paid.
 It is estimated that liquidation expenses amounting to P150,000 will be incurred for the
month of February, 2019.
 20% of the liabilities to third persons were settled.
 Available cash was distributed to the partners.

As of February 28, 2019, the following transactions occurred:


 Remaining noncash assets were sold at a loss of P100,000.
 The final liquidation expenses for the month of February amounted to P100,000.
 The remaining liabilities to third persons were settled at a compromise amount of
P1,500,000.
 Remaining cash was finally distributed to the partners.

16. What is the amount of cash received by partner C on January 31, 2019?
A. 260,000
B. 240,000
C. 300,000
D. 350,000

17. What is the share of B in the maximum possible loss on January 31, 2019?
A. 275,000
B. 110,000
C. 120,000
D. 165,000

18. What is the amount of total cash withheld on January 31, 2019?
A. 550,000
B. 1,600,000
C. 1,750,000
D. 1,700,000
Page 7

Numbers 19, 20 and 21 (Corporate Liquidation)

Cagayan Company is experiencing financial problems which resulted to ultimate bankruptcy.


The statement of financial position of the entity before liquidation is presented below:
Cash 100,000 Income tax payable 200,000
Inventory 300,000 Salaries payable 300,000
Land 200,000 Note payable 800,000
Mortgage payable 100,000
Accounts payable 400,000
Contributed capital 500,000
Deficit (1,700,000)

 The note payable is secured by the inventory with net realizable value of P250,000.
 The mortgage payable is secured by the land with fair value of P120,000.

19. What is the amount received by the holder of the note payable at the end of corporate
liquidation?
A. 320,000
B. 300,000
C. 250,000
D. 260,000

20. What is the amount received by the holder of the mortgage payable at the end of corporate
liquidation?
A. 120,000
B. 200,000
C. 150,000
D. 100,000

21. What is the amount received by the employees at the end of corporate liquidation concerning
their salaries?
A. 100,000
B. 120,000
C. 72,000
D. 300,000
Page 8

Numbers 22 and 23 (Corporate Liquidation)

Surigao Company is bankrupt and has undergone corporate liquidation. Presented below is its
statement of financial position before the start of liquidation:

Cash 300,000 Accounts Payable 100,000


Machinery 500,000 Salaries Payable 200,000
Building 1,200,000 Income tax Payable 300,000
Loan Payable 400,000
Mortgage payable 500,000
Contributed capital 800,000
Deficit (300,000)

 Liquidation expenses amounting to P600,000 were paid.


 The loan payable is secured by the machinery with fair value of P300,000.
 The mortgage payable is secured by the building.
 At the end of liquidation, the holder of loan payable received P340,000.

22. What is the amount received by the holder of accounts payable at the end of liquidation?
A. 85,000
B. 15,000
C. 40,000
D. 60,000

23. What is the amount of net free assets available at the end of liquidation?
A. 80,000
B. 40,000
C. 120,000
D. 200,000

Page 9

Numbers 24, 25 and 26 (Joint Arrangement classified as Joint Operation)

Entity A and Entity B incorporated Entity C to manufacture a microchip to be used by the


incorporating entities as component for their final products of cellular phones and tablets.

The contractual agreement of the incorporating entities provided that the decisions on relevant
activities of Entity C will require the unanimous consent of both entities.

Entity A and Entity B have rights to the assets, and obligations for the liabilities, relating to the
arrangement. The ordinary shares of Entity C will be owned by Entity A and Entity B in the ratio
of 60:40. At the end of first operation of Entity C, the financial statements provided the
following data:

Inventory 1,000,000 Accounts payable 2,000,000


Land 3,000,000 Note payable 1,000,000
Building 5,000,000 Loan payable 4,000,000
Share capital 1,000,000
Retained earnings 1,000,000
Sales revenue 5,000,000

The contractual agreement of Entity A and Entity B also provided for the following concerning
the assets and liabilities of Entity C:
 Entity A owns the land and incurs the loan payable of Entity C.
 Entity B owns the building and incurs the note payable of Entity C.
 The other assets and liabilities are owned or owed by Entity A and Entity B on the basis of
their capital interest in Entity C.
 The sales revenue of Entity C includes sales to Entity A and Entity B in the amount of
P1,000,000 and P2,000,000, respectively. As of the end of the first year, Entity A and Entity
B were able to resell 30% and 60% of the inventory coming from Entity C to third persons.

24. What is the amount of total assets to be reported by Entity A concerning its interest in Entity
C?
A. 5,400,000
B. 3,000,000
C. 3,600,000
D. 5,000,000

25. What is the amount of total liabilities to be reported by Entity B concerning its interest in
Entity C?
A. 1,800,000
B. 2,200,000
C. 2,800,000
D. 2,400,000

26. What is the amount of sales revenue to be reported by Entity A concerning its interest in
Entity C?
A. 2,300,000
B. 2,100,000
C. 3,000,000
D. 2,500,000

Page 10

Numbers 27 and 28 (Joint Arrangement classified as Joint Venture Equity Method)

On January 1, 2018, Entity A, a public entity, and Entity B, a public entity, incorporated Entity C
which has its fiscal and operational autonomy. The contractual agreement of the incorporating
entities provided that the decisions on relevant activities of Entity C will require the unanimous
consent of both entities. Entity A and Entity B will have rights to the net assets of Entity C.

Entity A and Entity B invested P1,000,000 and P1,500,000, respectively, equivalent to 40:60
capital interest of Entity C. The financial statements of Entity C provided the following data for
its two-year operation:
Net income (loss) Dividends declared
2018 200,000 100,000
2019 (2,000,000) -

27. What is the balance of Investment in Entity C to be reported by Entity A in its Statement of
Financial Position on December 31, 2019?
A. 1,080,000
B. 1,040,000
C. 240,000
D. 200,000
28. What is the balance of Investment in Entity C to be reported by Entity B in its Statement of
Financial Position on December 31, 2019?
A. 1,500,000
B. 1,620,000
C. 360,000
D. 900,000

Page 11

Numbers 29 and 30 (Joint venture - Intercompany Transaction)

On January 1, 2018, Entity A, a public entity, and Entity B, a public entity, incorporated Entity C
by investing P3,000,000 and P2,000,000 for capital interest ratio of 60:40. The contractual
agreement of the incorporating entities provided that the decisions on relevant activities of Entity
C will require the unanimous consent of both entities. Entity A and Entity B will have rights to
the net assets of Entity C.

The financial statements of Entity C provided the following data for 2018:

 Entity C reported net income of P1,000,000 for 2018 and paid cash dividends of P400,000 on
December 31, 2018.

 During 2018, Entity C sold inventory to Entity A with gross profit of P50,000. Eighty percent
of those inventories were resold by Entity A to third persons during 2018 and the remainder
was resold to third persons during 2019.

 On July 1, 2018, Entity C sold a machinery to Entity B at a loss of P20,000. At the time of
sale, the machinery has remaining useful life of 2 years.

29. What is the investment income to be reported by Entity A for the year ended December 31,
2018?
A. 603,000
B. 606,000
C. 594,000
D. 597,000

30. What is the balance of Investment in Entity C to be reported by Entity B on December 31,
2018?
A. 2,242,000
B. 2,241,000
C. 2,238,000
D. 2,248,000

Page 12

Numbers 31 and 32 Joint Venture – IFRS for SMEs (Fair Value Model or Equity
Method)

On January 1, 2018, Entity A and Entity B, both SMEs, incorporated Entity C, a jointly
controlled entity by investing P500,000 each in exchange for 10,000 ordinary shares each of
Entity C. Entity A and Entity B each incurred P20,000 transaction costs.
The contractual agreement of the incorporating entities provided that the decisions on relevant
activities of Entity C will require the unanimous consent of both entities. Entity A and Entity B
will have rights to the net assets of Entity C.
For the year ended December 31, 2018, Entity C reported net income of P100,000 and declared
dividends in the amount of P30,000.
On December 31, 2018, the ordinary shares of Entity C are quoted at P56.

31. If Entity A elected fair value model to account its investment in Entity C, what is the net
effect on Entity A’s profit or loss for the year ended December 31, 2018?
A. 55,000 net profit
B. 60,000 net profit
C. 15,000 net profit
D. 40,000 net profit

32. If Entity B elected equity method to account its investment in Entity C, what is the carrying
amount of Entity B’s Investment in Entity C on December 31, 2018?
A. 520,000
B. 540,000
C. 535,000
D. 555,000

Numbers 33 and 34 Joint Venture – IFRS for SMEs (Cost Method or Equity Method)

On January 1, 2018, Entity A and Entity B, both SMEs, incorporated Entity C, a jointly
controlled entity by investing P200,000 each in exchange for 20,000 ordinary shares each of
Entity C. Entity A and Entity B each incurred P10,000 transaction costs.
The contractual agreement of the incorporating entities provided that the decisions on relevant
activities of Entity C will require the unanimous consent of both entities. Entity A and Entity B
will have rights to the net assets of Entity C.
For the year ended December 31, 2018, Entity C reported net income of P50,000 and declared
dividends in the amount of P10,000.
On December 31, 2018, the investment in Entity C has value in use of P215,000.

33. If Entity A elected cost method to account its Investment in Entity C, what is the carrying
amount of Entity A’s Investment in Entity C on December 31, 2018?
A. 210,000
B. 215,000
C. 230,000
D. 200,000

34. If Entity B elected equity method to account its Investment in Entity C, what is the net effect
in Entity B’s profit or loss for the year ended December 31, 2018?
A. 25,000 net profit
B. 5,000 net profit
C. 10,000 net profit
D. 15,000 net profit

Page 13

Number 35 (Installment sales)

Nikko Company, which began operations on January 5, 2018, appropriately uses the installment
method of revenue recognition. The following information pertains to the operations for 2018
and 2019:

2018 2019
Sales 300,000 450,000
Collections from :
2018 sales 100,000 50,000
2019 sales - 150,000
Accounts written off from
2018 sales 25,000 75,000
2019 sales - 150,000
Gross profit rates 30% 40%
What amount should be reported as deferred gross profit on December 31, 2019?
A. 75,000
B. 80,000
C. 112,000
D. 125,000

Numbers 36 and 37 ( Installment sales)

Appliance Company reports gross profit on the installment basis. The following data are
available:

2018 2019 2020


Installment sales 240,000 250,000 300,000
Cost of goods – installment sales 180,000 181,250 216,000
Gross profit 60,000 68,750 84,000

Collections
2018 installment contracts 45,000 75,000 72,500
2019 installment contracts 47,500 80,000
2020 installment contracts 62,500

Defaults
Unpaid balance of 2018 installment contracts 12,500 15,000
Value assigned to repossessed merchandise 6,500 6,000
Unpaid balance of 2019 installment contracts 16,000
Value assigned to repossessed merchandise 9,000

36. What is the realized gross profit before loss on repossession for 2020?
A. 49,775
B. 57,625
C. 48,975
D. 56,625

1. What is the loss on repossession for 2020?


A. 5,250
B. 2,600
C. 7,850
D. 9,000

Page 14
Number 38 (Installment sales)

Davao Company uses the installment method of income recognition. The entity provided the
following pertinent data:

2018 2019 2020


Installment sales 300,000 375,000 360,000
Cost of goods sold 225,000 285,000 252,000

Balance of Deferred Gross Profit at Year end


2018 52,500 15,000 -
2019 54,000 9,000
2020 72,000

What is the total balance of the Installment Accounts Receivable on December 31, 2020?
A. 270,000
B. 277,500
C. 279,000
D. 300,000

Numbers 39 and 40 (Installment Sales)

On January 1, 2018, an entity sold a car to a customer at a price of P400,000 with a production
cost of P300,000. It is the entity’s policy to employ installment method to recognize gross profit
from installment sales.

At the time of sale, the entity received cash amounting to 25% of the selling price and old car
with trade-in allowance of P50,000. The said old car has fair value of P150,000. The customer
issued a 5-year note for the balance to be payable in equal annual installments every December
31 starting 2018. The note payable is interest bearing with 10% rate due on the remaining
balance of the note.

The customer was able to pay the first annual installment and corresponding interest due.
However, after the payment of the second interest due, the customer defaulted on the second
annual installment which resulted to the repossession of the car sold with appraised value of
P110,000. On December 31, 2019, the repossessed car was resold for P140,000 after
reconditioning cost of P10,000.

39. What is the entity’s realized gross profit for the year ended December 31, 2018?
A. 50,000
B. 120,000
C. 108,000
D. 128,000

40. What is the loss on repossession for the year ended December 31, 2019?
A. 30,000
B. 20,000
C. 10,000
D. 40,000

Page 15

Numbers 41, 42 and 43 (Revenue Recognition – Franchise Fees)

On January 1, 2018, an entity granted a franchise to a franchisee. The franchise agreement


required the franchisee to pay a nonrefundable upfront fee in the amount of P400,000 and on-
going payment of royalties equivalent to 5% of the sales of the franchisee. The franchisee paid
the nonrefundable upfront fee on January 1, 2018.

In relation to the nonrefundable upfront fee, the franchise agreement required the entity to render
the following performance obligations:

 To construct the franchisee’s stall with stand-alone selling price of P200,000.


 To deliver 10,000 units of raw materials to the franchisee with stand-alone selling price of
P250,000.
 To allow the franchisee to use the entity tradename for a period of 10 years starting January
1, 2018 with stand-alone selling price of P50,000.

On June 30, 2018, the entity completed the construction of the franchisee’s stall. On December
31, 2018, the entity was able to deliver 3,000 units of raw materials to the franchisee. For the
year ended December 31, 2018, the franchisee reported sales revenue amounting to P100,000.

The entity had determined that the performance obligations are separate and distinct from one
another.
41. What is the amount of nonrefundable upfront fee to be allocated to the construction of the
franchisee’s stall?
A. 200,000
B. 160,000
C. 250,000
D. 120,000

42. What is the amount of revenue to be recognized in relation to the use of delivery of raw
materials for the year ended December 31, 2018?
A. 100,000
B. 200,000
C. 60,000
D. 75,000

43. What is the amount of revenue to be recognized in relation to the use of entity’s tradename
for the year ended December 31, 2018?
A. 5,000
B. 4,000
C. 50,000
D. 10,000

Page 16

Numbers 44 and 45 (Revenue Recognition – Net Income of Franchisor)

On January 1, 2018, an entity granted a franchise agreement to a franchisee. The contract


provided that the franchisee shall pay an initial franchise fee of P500,000 and on-going payment
of royalties equivalent to 8% of the sales of the franchisee.

On January 1, 2018, the franchisee paid downpayment of P200,000 and issued a 3-year
noninterest bearing note for the balance payable in three equal annual installments starting
December 31, 2018. The note has present value of P240,183 with effective interest rate of 12%.
On June 30, 2018, the entity completed the performance obligation of the franchise at a cost of
P352,146. Aside from that, the entity incurred indirect cost of P22,009.

The franchisee started operation on July 1, 2018 and reported sales revenue amounting to
P50,000 for the year ended December 31, 2018. The franchisee paid the first installment on its
due date.

44. If the collection of the note receivable is reasonably assured, what is the gross profit to be
recognized by the entity for the year ended December 31, 2018 in relation to the initial
franchise fee?
A. 66,028
B. 44,014
C. 22,009
D. 88,037

45. If the collection of the note receivable is reasonably assured, what is the net income to be
reported by the entity for the year ended December 31, 2018?
A. 98,850
B. 94,850
C. 70,028
D. 92,037
Page 17

Numbers 46, 47 and 48 (Construction contract - Percentage of Completion Method)

On January 1, 2018, Solid Company accepted a long-term construction project for an initial
contract price of P1,000,000 to be completed on June 30, 2020. On January 1, 2019, the contract
price was increased to P1,500,000 by reason of change in the design of the project. The outcome
of the construction contract can be estimated reliably. The project was completed on December
31, 2020 which resulted to penalty amounting to P200,000. The entity provided the following
data concerning the direct costs related to the said project for 2018 and 2019:

2018 2019
Costs during the year 440,000 680,000
Remaining estimated costs to complete at year- 660,000 280,000
end

46. What is the construction revenue for the year ended December 31, 2018?
A. 340,000
B. 400,000
C. 440,000
D. 360,000

47. What is the realized gross profit for the year ended December 31, 2019?
A. 200,000
B. 80,000
C. 180,000
D. 100,000

48. What is the balance of construction in progress on December 31, 2019?


A. 1,200,000
B. 1,020,000
C. 1,120,000
D. 900,000
Page 18

Numbers 49, 50 and 51 (Construction contract - Cost Recovery Method)

On January 1, 2018, Hardrock Company started the construction of a building at a fixed contract
price of P1,000,000. On the same date, the customer paid a mobilization fee equal to 5% of
contract price that will be deductible from the first billing. The outcome of construction contract
cannot be estimated reliably

During 2018, the entity billed the customer equivalent to 30% of the contract price. During 2019,
the entity billed again the customer amounting to 20% of the contract price. During 2020, the
entity billed again the customer amounting to 40% of the contract price. The remaining billing
was made at the year of completion of the project.

The entity made collection from the customer at the end of 2018, 2019 and 2020, in the amount
of P120,000, P450,000 and P180,000, respectively. The entity provided the following data
concerning the direct costs related to the said project:

2018 2019 2020


Cumulative costs incurred at year-end 360,000 800,000 870,000
Remaining estimated costs to complete at year- 840,000 250,000 50,000
end

49. What is the realized gross profit for the year ended December 31, 2019?
A. 50,000
B. 200,000
C. 150,000
D. 0

50. What is the excess of construction in progress over progress billings or excess of progress
billings over construction in progress on December 31, 2020?
A. 30,000 excess billings
B. 80,000 excess billings
C. 20,000 excess construction in progress
D. 50,000 excess construction in progress

51. What is the balance of accounts receivable on December 31, 2020?


A. 150,000
B. 100,000
C. 120,000
D. 50,000

Page 19

Numbers 52, 53 and 54 (Home Office, Branch and Agency Transactions)

Siargao Company set up a branch in a province. The entity and its branch provided the following
data for the second year of branch operation:

Home Office Branch


Sales revenue to outside customer 1,000,000 500,000
Beginning inventory 50,000 30,000
Purchases from outside supplier 400,000 100,000
Shipment to branch 200,000
Shipment from home office 250,000
Ending inventory 80,000 50,000
Operating expenses 150,000 40,000

 The home office to branch markup based on cost is 25% this year and last year.
 20% of the beginning inventory of the branch came from outside supplier.
 24% of the ending inventory of the branch came from the last year’s shipment from the home
office while 50% of the ending inventory of the branch came from current year’s shipment
from the home office.

52. What is the net income reported by the branch in its separate income statement for the current
year?
A. 130,000
B. 124,000
C. 114,000
D. 95,000

53. What is the ending inventory to be reported by the entity in its combined statement of
financial position?
A. 128,000
B. 115,000
C. 130,000
D. 122,600

54. What is the overstatement in the cost of goods sold reported by the branch in its separate
income statement for the current year?
A. 54,000
B. 50,000
C. 52,000
D. 47,400
Page 20

Number 55 (Home office and branch)

The home office in Quezon City ships and bills merchandise to its provincial branch at cost. The
branch carries its own accounts receivable and makes its own collections. The branch also pays
its expenses. The branch transactions for 2018 are reflected in the following information:

Cash 20,000
Accounts receivable 80,000
Home Office 180,000
Shipments from Home Office 250,000
Sales 225,500
Expenses 55,500
December 31, 2018 inventory 65,000

What is the balance of the Investment in Branch account in the home office book?
A. 180,000
B. 195,000
C. 165,000
D. 175,000

Numbers 56 and 57 (Home office and branch)

Coffee Company decided to open a branch in Manila. Shipments of merchandise to the branch
totaled P54,000 which included a 20% markup on cost. All accounting records are kept at the
home office. The branch submitted the following report summarizing the operations for the year
ended December 31, 2018:

Sales on account 74,000


Sales on cash basis 22,000
Collections of accounts receivable 60,000
Expenses paid 38,000
Expenses unpaid 12,000
Purchase of merchandise for cash 26,000
Inventory on hand, December 31; 80% from home office 30,000
Remittance to home office 55,000

56. What is the branch inventory on December 31, 2018 at cost?


A. 25,000
B. 20,000
C. 26,000
D. 10,000

57. What is the branch net income for the current year?
A. 1,000
B. 4,000
C. 800
D. 500

Page 21

Numbers 58, 59, 60 and 61 (Business Combination - Acquisition of Net Assets)

Entity A acquired the net assets of Entity B by issuing 10,000 ordinary shares with par value of
P10 and bonds payable with face amount of P500,000. The bonds are classified as financial
liability at amortized cost.

At the time of acquisition, the ordinary shares are publicly quoted at P20 per share. On the other
hand, the bonds payable are trading at 110.

Entity A paid P10,000 share issuance costs and P20,000 bond issue costs. Entity A also paid
P40,000 acquisition related costs and P30,000 indirect costs of business combination.

Before the date of acquisition, Entity A and Entity B reported the following data:

Entity A Entity B
Current assets 1,000,000 500,000
Noncurrent assets 2,000,000 1,000,000
Current liabilities 200,000 400,000
Noncurrent liabilities 300,000 500,000
Ordinary shares 500,000 200,000
Share premium 1,200,000 300,000
Retained earnings 800,000 100,000

At the time of acquisition, the current assets of Entity A have fair value of P1,200,000 while the
noncurrent assets of Entity B have fair value of P1,300,000. On the same date, the current
liabilities of Entity B have fair value of P600,000 while the noncurrent liabilities of Entity A
have fair value of P500,000.

58. What is the goodwill or gain on bargain purchase arising from business combination?
A. 50,000 goodwill
B. 150,000 gain on bargain purchase
C. 120,000 goodwill
D. 70,000 gain on bargain purchase

59. What total amount should be expensed as incurred at the time of business combination?
A. 20,000
B. 70,000
C. 30,000
D. 50,000

60. What is Entity A’s amount of total assets after the business combination?
A. 4,520,000
B. 4,810,000
C. 4,750,000
D. 4,440,000

61. What is Entity A’s amount of total liabilities after the business combination?
A. 2,240,000
B. 2,510,000
C. 2,320,000
D. 2,130,000

Page 22

Numbers 62 and 63 (Business Combination – Acquisition of majority shares)

Entity A acquired 80,000 out of 100,000 outstanding ordinary shares of Entity B which enabled
the former to obtain control of the latter at an acquisition price of P1,000,000. Entity A paid
P100,000 acquisition related costs and P50,000 indirect costs of business combination.

At the date of acquisition, the net assets of Entity B are reported at P1,600,000. An asset of
Entity B is overvalued by P60,000 while one liability is undervalued by P40,000.
62. What is the initial measurement of noncontrolling interest in net assets in the consolidated
statement of financial position?
A. 320,000
B. 300,000
C. 250,000
D. 316,000

63. What is the goodwill or gain on bargain purchase arising from business combination?
A. 250,000 gain on bargain purchase
B. 150,000 gain on bargain purchase
C. 50,000 goodwill
D. 200,000 gain on bargain purchase

Numbers 64 and 65 (Step Acquisition)

On January 1, 2018, Entity A acquired 30,000 out of 100,000 outstanding ordinary shares of
Entity B for P90,000 or 30% interest. For the six months ended June 30, 2018, Entity B reported
net income of P40,000.

On July 1, 2018, Entity A acquired additional 60,000 ordinary shares of Entity B or 60% interest
at a price of P4 per share or total cost of P240,000. Entity A paid P20,000 acquisition related
costs and P10,000 indirect costs of business combination.

The acquisition price per share of the additional shares clearly reflected the fair value of the
existing interest of Entity A in Entity B. It is the policy of Entity A to initially measure the
noncontrolling interest in net assets of the acquiree at fair value. The fair value of the
noncontrolling interest in net assets of the acquiree is reliably measured at P50,000.

At the acquisition date, the net assets of Entity B were reported at P400,000. An asset of Entity B
was overvalued by P50,000 while one liability wass overvalued by P30,000.

64. What is the gain on remeasurement of the existing Investment in Entity B as a result of step
acquisition?
A. 18,000
B. 30,000
C. 24,000
D. 12,000

65. What is the goodwill or gain on bargain purchase as a result of the business combination?
A. 18,000 goodwill
B. 20,000 gain on bargain purchase
C. 24,000 goodwill
D. 30,000 goodwill
Page 23

Numbers 66, 67 and 68 (Consolidated Financial Statements)

On January 1, 2018, Entity A acquired 70% of outstanding ordinary shares of Entity B at a price
of P210,000. On the same date, the net assets of Entity B were reported at P260,000. On January
1, 2018 Entity A reported retained earnings of P2,000,000 while Entity B reported retained
earnings of P200,000.

All the assets and liabilities of Entity B are fairly valued except machinery which is undervalued
by P80,000 and inventory which is overvalued by P10,000. The said machinery has remaining
useful life of four years while 40% of the said inventory remained unsold at the end of 2018.

For the year ended December 31, 2018, Entity A reported net income of P1,000,000 and declared
dividends of P200,000 in the separate financial statements while Entity B reported net income of
P150,000 and declared dividends of P20,000 in the separate financial statements.

Entity A accounted the investment in Entity B using cost method in the separate financial
statements.

66. What is the noncontrolling interest in net assets on December 31, 2018?
A. 124,800
B. 130,200
C. 126,000
D. 133,800

67. What is the consolidated net income attributable to parent shareholders for the year ended
December 31, 2018?
A. 1,102,200
B. 1,162,200
C. 1,141,200
D. 1,095,200

68. What is the amount of consolidated retained earnings on December 31, 2018?
A. 3,012,200
B. 2,991,200
C. 2,952,200
D. 2,945,200

Page 24

Numbers 69, 70, 71 and 72 (Consolidated Financial Statements - Intercompany sales)

On January 1, 2019, Entity A acquired 60% of outstanding ordinary shares of Entity B at a gain
on bargain purchase of P40,000. For the year ended December 31, 2020, Entity A and Entity B
reported sales revenue of P2,000,000 and P1,000,000 in their respective separate income
statements. At the same year, Entity A and Entity B reported cost of goods sold of P1,200,000
and P700,000 in their respective separate income statements.

During 2019, Entity A sold inventory to Entity B at a selling price of P280,000 with gross profit
rate of 40% based on cost. On the other hand, Entity B sold inventory to Entity A at a selling
price of P400,000 with gross profit rate of 30% based on sales during 2020.

On December 31, 2019, 25% of the goods coming from Entity A remained in Entity B’s
inventory but all were eventually sold to third persons during 2020. As of December 31, 2020,
40% of the goods coming from Entity B were eventually sold to third persons.

For the year ended December 31, 2020, Entity A reported net income of P500,000 while Entity B
reported net income of P200,000 and distributed dividends of P50,000. Entity A accounted for its
inventory in Entity B using cost method in its separate financial statements.
69. What is the consolidated sales revenue for the year ended December 31, 2020?
A. 2,600,000
B. 2,320,000
C. 3,000,000
D. 2,720,000

70. What is the consolidated gross profit for the year ended December 31, 2020?
A. 1,120,000
B. 1,048,000
C. 1,028,000
D. 1,152,000

71. What is the noncontrolling interest in net income for the year ended December 31, 2020?
A. 100,800
B. 59,200
C. 51,200
D. 88,000

72. What is the consolidated net income attributable to parent’s shareholders for the year ended
December 31, 2020?
A. 766,800
B. 596,800
C. 606,800
D. 626,800

Page 25

Numbers 73, 74, 75 and 76 (Consolidated Statements-Intercompany gain or loss on


disposal)

On January 1, 2019, Entity A acquired 80% of outstanding ordinary shares of Entity B at a gain
on bargain purchase of P180,000. The following intercompany transactions occurred for between
the two entities:
 On January 1, 2019, Entity B sold a land to Entity A with a cost of P1,000,000 at a selling
price of P1,100,000. The land was eventually sold by Entity A to third persons during 2020.

 On January 1, 2019, Entity A sold a white machinery to Entity B with a cost of P200,000 and
accumulated depreciation of P40,000 at a selling price of P180,000. The machinery is already
4 years old at the date of sale. The residual value of white machinery is immaterial.

 On July 1, 2020, Entity B sold a black machinery to Entity A at with a cost of P270,000 and
accumulated depreciation of P180,000 at a selling price of P60,000. The machinery is already
6 years old at the date of sale. The residual value of black machinery is immaterial.

For the year ended December 31, 2020, Entity A reported net income of P800,000 while Entity B
reported net income of P500,000 and distributed dividends of P150,000. Entity A accounted for
its inventory in Entity B using cost method in its separate financial statements.

73. What is the consolidated depreciation expense of machinery for 2020?


A. 40,000
B. 55,000
C. 61,667
D. 42,333

74. What is the consolidated carrying amount of machinery on December 31, 2020?
A. 225,000
B. 215,000
C. 200,000
D. 210,000

75. What is the noncontrolling interest in net income for 2020?


A. 124,000
B. 105,000
C. 125,000
D. 104,000

76. What is the consolidated net income attributable to parent shareholders for 2020?
A. 1,538,750
B. 1,518,750
C. 1,398,750
D. 1,418,750
Page 26

Separate Financial Statements - Cost Method and Fair Value Model or Equity Method

Numbers 77, 78, 79 and 80

On January 1, 2020, Entity A acquired 90% of outstanding ordinary shares of Entity B at a price
of P900,000. Entity A paid P20,000 costs related to acquisition of shares.

At the acquisition date, the net assets of Entity B were reported at P950,000. All the assets of
Entity B are properly valued except for a machinery which is undervalued by P150,000. The
machinery has a remaining useful life of 5 years.

For the year ended December 31, 2020, Entity B reported net income of P200,000 and declared
dividends in the amount of P30,000.

The fair value of Investment in Entity B on December 31, 2020 is P1,000,000 while the cost of
disposal is 5%.

Entity A voluntarily prepared its separate financial statements.

77. If Entity A elects cost method to account its Investment in Entity B in its separate financial
statements, what is the carrying amount of the Investment in Entity B on December 31,
2020?
A. 900,000
B. 920,000
C. 1,000,000
D. 950,000

78. What is the investment income for 2020 if Entity A elects cost method to account its
Investment in Entity B in its separate financial statements?
A. 7,000
B. 27,000
C. 180,000
D. 107,000

79. If Entity A elects fair value model to account its Investment in Entity B in its separate
financial statements, what is the carrying amount of the Investment in Entity B on
December 31, 2020?
A. 900,000
B. 920,000
C. 1,000,000
D. 950,000
80. What is the net effect in profit or loss for 2020 if Entity A elects fair value model to
account its Investment in Entity B in its separate financial statements?
A. 7,000
B. 27,000
C. 180,000
D. 107,000

Page 27

Numbers 81, 82 and 83 (Nonprofit Organization – Statement of Financial Position)

In the first year of operations of a nonprofit organization, the following transactions occurred:

 The nonprofit organization received P1,000,000 fund from a donor who stipulated that it
shall be invested indefinitely and the dividend from such investment shall be used for
research project of the organization. Dividend amounting to P150,000 was received during
the year but only P50,000 was spent for the research project.

 The nonprofit organization received P300,000 fund from a donor who stipulated that it shall
be used for the acquisition of service car. The nonprofit organization used P100,000 of the
fund for the acquisition of a service car with useful life of 5 years. The car was acquired at
the middle of the year.

 The nonprofit organization received P500,000 fund who stipulated that it shall be used based
on the discretion of the Board of Trustees of the nonprofit organization. The nonprofit
organization used P100,000 for the acquisition of souvenir items which were sold by the
nonprofit organization for P150,000. The remaining P400,000 was designated by the Board
of Trustees for future fundraising projects.

81. What is the amount of permanently restricted net assets at the end of the first year?
A. 1,100,000
B. 1,300,000
C. 1,200,000
D. 1,000,000

82. What is the amount of temporarily restricted net assets at the end of the year?
A. 100,000
B. 300,000
C. 200,000
D. 700,000

83. What is the amount of unrestricted net assets at the end of the year?
A. 640,000
B. 540,000
C. 590,000
D. 630,000

Page 28

Nonprofit Organization – Statement of Activities and Statement of Cash Flows

Numbers 84, 85, 86 and 87


On January 1, 2020, a nonprofit organization received P1,000,000 cash donation from a donor
who stipulated that the amount should be invested indefinitely in revenue producing investment.
The deed of donation also provided that the dividend income shall be used for the acquisition of
computers of the nonprofit organization.

On December 31, 2020, the nonprofit organization received P100,000 cash as dividend income
from the investment of the fund.

On January 1, 2021, the nonprofit organization acquired a computer at a cost of P20,000 with a
useful life of 5 years without residual value.

84. In the statement of activities of the NPO for the year ended December 31, 2020, which of
the following is the proper effect of the transactions?
A. Increase in temporarily restricted net assets by P100,000.
B. Increase in unrestricted net assets by P1,000,000.
C. Increase in unrestricted net assets by P16,000.
D. Decrease in temporarily restricted net assets by P20,000.

85. In the statement of activities of the NPO for the year ended December 31, 2021, which of
the following is the proper effect of the transactions?
A. Increase in temporarily restricted net assets by P100,000.
B. Increase in unrestricted net assets by P1,000,000.
C. Increase in unrestricted net assets by P16,000.
D. Decrease in temporarily restricted net assets by P100,000.

86. How should the cash flows be reported in NPO’s Statement of Cash Flows for the year
ended December 31, 2020?
A. Cash receipts from operating activities by P100,000.
B. Cash receipts from financing activities by P1,100,000.
C. Cash disbursements for investing activities by P50,000.
D. Cash disbursements for financing activities by P1,000,000

87. How should the cash flows be reported in NPO’s Statement of Cash Flows for the year
ended December 31, 2021?
A. Cash receipts from operating activities by P100,000.
B. Cash receipts from financing activities by P1,100,000.
C. Cash disbursements for investing activities by P20,000.
D. Cash disbursements for investing activities by P100,000.
Page 29
Number 88
Government Accounting Manual
On December 31, 2018, the Department of Finance billed its lessee on one of its buildings in the
amount of P10,000. On January 31, 2019, the Department of Finance collected all of the
accounts receivable. On February 28, 2019, the Department of Finance remitted the entire
collected amount to the Bureau of Treasury. What is the journal entry to record the remittance by
the Department of Finance to the Bureau of Treasury?
A. Debit – Accounts Receivable P10,000 and Credit – Rent Income P10,000
B. Debit – Accounts Receivable P10,000 and Credit – Retained Earnings P10,000
C. Debit – Cash Collecting Officers P10,000 and Credit – Accounts Receivable P10,000
D. Debit – Cash – Treasury/Agency Deposit, Regular – P10,000 and
Credit Cash – Collecting Officer – P10,000

Numbers 89 and 90
On January 1, 2018, the Department of Public Works and Highways (DPWH) received a
P10,000,000 appropriation from the national government for the acquisition of machinery. On
February 1, 2018, DPWH received the allotment from the Department of Budget and
Management. On March 1, 2018, DPWH entered into a contract with CAT Inc. for the
acquisition of the machinery with a price of P8,000,000. On April 1, 2018, DPWH received the
Notice of Cash Allocation from Department of Budget and Management net of 1% withholding
tax for income tax of supplier and 5% withholding of Final Tax on VAT of supplier. On May 1,
2018, CAT Inc. delivered the machinery to DPWH. On June 1, 2018, DPWH paid the obligation
to CAT Inc. On July 1, 2018, DPWH remitted the withheld income tax and final VAT to BIR.

89. What is the journal entry on March 1, 2018?


A. No entry but just posting to appropriate RAPAL
B. No entry but just posting to appropriate RAPAL and to RAOD
C. No entry but just posting of ORS (Obligation Request and Status) to appropriate
RAOD
D. Debit Machinery P8,000,000 and credit Accounts Payable P8,000,000

90. What is the journal entry on April 1, 2018?


A. Debit Cash-MDS, Regular P7,520,000 and Credit Subsidy Income from National
Government P7,520,000.
B. Debit Machinery P8,000,000 and Credit Accounts Payable P8,000,000
C. Debit Accounts Payable P8,000,000 and Credit Due to BIR P480,000 and Cash-MDS,
Regular P7,520,000.
D. Debit Due to BIR P480,000 and Credit Subsidy Income from National Government
P480,000.

Number 91
Department of Health (DOH) received Notice of Cash Allocation in the amount of P100,000
from Department of Budget and Management. DOH made a total cash disbursements in the
amount of P95,000. What is the journal entry to recognize reversion of unused Notice of Cash
Allocation by DOH in its books?
A. Debit Subsidy Income from National Government P5,000 and credit Cash-MDS,
Regular P5,000.
B. Debit Retained Earnings of DFA P5,000 and credit Cash-MDS, Regular P5,000.
C. Debit Expenses of DFA P5,000 and credit Cash-MDS, Regular P5,000.
D. Debit Investment of DFA P5,000 and credit Cash-MDS, Regular P5,000.

Number 92
The Bureau of Treasury received P20,000 cash remittance from Department of Agrarian Reform
(DAR) from its miscellaneous income. What is the journal entry of the Bureau of Treasury in its
accounting books to record the receipt of cash remittance from the income of a national
government agency?
A. Debit Cash in Bank, Local Bank P20,000 and Credit Cash-Treasury/Agency Deposit,
Regular P20,000.
B. Debit Cash in Bank, Local Bank P20,000 and Credit Miscellaneous Income of DA P20,000.
C. Debit Cash in Bank, Local Bank P20,000 and Credit Savings of DA, Regular P20,000.
D. Debit Cash in Bank, Local Bank P20,000 and Credit Cash-Collecting Officer, DA P20,000.
Page 30

Number 93 (Foreign currency transaction)

On September 1, 2018, Bain Company received an order for equipment from a foreign customer
for 300,000 local currency units (LCU) when the US dollar equivalent was $96,000. Bain
shipped the equipment on October 15, 2018, and billed the customer for 300,000 LCU when the
US dollar equivalent was $100,000. Bain received the customer remittance in full on November
16, 2018, and sold the 300,000 LCU for $105,000. In the income statement for the year ended
December 31, 2018, what amount should Bain report as part of net income a foreign exchange
transaction gain?
A. $ 0
B. $4,000
C. $5,000
D. $9,000

Number 94 (Foreign currency transaction)


On September 1, 2018, Cano Company, a US corporation, sold merchandise to a foreign firm for
250,000Botswana pula. Terms of the sale require payment in pula on February 1, 2019.. On
September 1, 2018, the spot exchange rate was $.20 per pula. At December 31, 2018, Cano’s
year-end, the spot rate was $.19, but the rate increased to $.22 by February 1, 2019, when
payment was received. How much should Cano report as foreign exchange transaction gain or
loss as part of 2019 income?
A. $ 0
B. $2,500 loss
C. $5,000 gain
D. $7,500 gain

Number 95 (Foreign currency transaction)

Hunt Company purchased merchandise for £300,000 from a vendor in London on November 30,
2018. Payment in British pounds was due on January 30, 2019. The exchange rates to purchase
one pound were as follows:

November 30, 2018 December 31, 2018


Spot-rate $1.65 $1.62
30-day rate 1.64 1.59
60-day rate 1.63 1.56

In the income statement, what amount should Hunt report as foreign exchange transaction gain
as part of net income?
A. $12,000
B. $ 9,000
C. $ 6,000
D. $ 0

Page 31
Number 96 (Foreign currency transaction)

Ball Company had the following foreign currency transactions during 2018:

 Merchandise was purchased from a foreign supplier on January 20, 2018, for the US dollar
equivalent of $90,000. The invoice was paid on March20, 2018, at the US dollar equivalent
of $96,000.

 On July 1, 2018, Ball borrowed the US dollar equivalent of $500,000 evidenced by a note
payable in the lender’s local currency on July 1, 2020. On December 31, 2018, the US dollar
equivalents of the principal amount and accrued interest were $520,000 and $26,000,
respectively. Interest on the note is 10% per annum.

In Ball’s 2018 income statement, what amount should be included as foreign exchange
transaction loss as part of net income?
A. $ 0
B. $ 6,000
C. $21,000
D. $27,000

Number 97 (Foreign currency transaction)

On November 30, 2018, Tyrola Publishing Company, located in Colorado, executed a contract
with Ernest Blyton, an author from Canada, providing for payment of 10% royalties on Canadian
sales of Blyton’s book. Payment is to be made in Canadian dollars each January 10 for the
previous year’s sales. Canadian sales of the book for the year ended December 31, 2019, totaled
$50,000 Canadian. Tyrola paid Blyton his 2019 royalties on January 10, 2020. Tyrola’s 2019
financial statements were issued on February 1, 2020. Spot rates for Canadian dollars were as
follows:

November 30, 2018 $.87


January 1, 2019 $.88
December 31, 2019 $.89
January 10, 2020 $.90

How much should Tyrola accrue for royalties payable at December 31, 2019?
A. $4,350
B. $4.425
C. $4,450
D. $4,500
Page 32

Numbers 98, 99, 100 and 101 (Foreign Currency Transaction)

On November 1, 2020, an entity acquired on account goods from a foreign supplier at a cost of
$1,000. The accounts payable are paid on January 30, 2021.

On December 1, 2020, an entity sold on account the said goods to a foreign customer at a selling
price of $1,500. The accounts receivable are collected on February 28, 2021.

The entity is operating in Philippine economy wherein the functional currency is the Philippine
Peso.

The following direct exchange rates are provided:

Buying spot rate Selling spot rate


November 1, 2020 P40 P42
December 1, 2020 39 40
December 31, 2020 45 47

98. What is the sales revenue for 2020?


A. 58,500
B. 60,000
C. 67,500
D. 72,000

99. What is the carrying amount of accounts receivable on December 31, 2020?
A. 58,500
B. 60,000
C. 67,500
D. 72,000

100. What is the carrying amount of accounts payable on December 31, 2020?
A. 40,000
B. 42,000
C. 45,000
D. 47,000

101. What is the net foreign currency gain for 2020?


A. 4,000
B. 5,000
C. 3,000
D. 6,000

Page 33

Translation of Financial Statements in Functional Currency to Presentation Currency

Numbers 102, 103, 104 and 105

Entity A owns majority of the outstanding ordinary shares of Entity B which is operating in
United States of America wherein the functional currency is the USA $. However, the
presentation currency of Entity B is the Philippine Peso because that is the presentation currency
of Entity A. For the year ended December 31, 2020, Entity B presented its Statement of
Financial Position in its functional currency of USA $:
Current assets $10,000 Current liabilities $10,000
Noncurrent assets 40,000 Noncurrent liabilities 20,000
Ordinary share capital 5,000
Preference share capital 8,000
Retained earnings 7,000
Total Assets $50,000 Total Liabilities and shareholders $50,000

 The ordinary shares are issued on January 1, 2019 while the preference shares are issued on
July 1, 2019.
 B reported $1,000 net income during 2020 and declared dividends in the amount of $200 on
December 1, 2020.
 The translated amount of retained earnings on December 31, 2019 is P300,000.

The following direct exchange rates are provided:

January 1, 2019 P40


July 1, 2019 42
December 31, 2019 43
December 1, 2020 41
December 31, 2020 45
Average rate 2020 44

102. What is the amount of net assets in US dollars on December 31, 2019?
A. 19,200
B. 20,000
C. 19,000
D. 20,200

103. What amount of translation gain as component of other comprehensive income should be
presented in the of statement of comprehensive income for the year ended December 31,
2020?
A. 38,600
B. 39,200
C. 40,400
D. 41,800

104. What is the translated retained earnings balance on December 31, 2020?
A. 300,000
B. 335,800
C. 344,000
D. 281,800

105. What is the cumulative translation credit that should to be presented in the statement of
financial position on December 31, 2020?
A. 25,400
B. 28,200
C. 26,800
D. 24,600
Page 34

Numbers 106 and 107 (Standard Costing – Direct material variance)

Negros Company recently set-up its standard costs for its direct materials. The entity sets the
benchmark at 3 units of direct materials per product at a standard price of P5 per unit of direct
material.

During the year, the entity acquired 400 units of direct materials at a total cost of P2,400 or P6
per unit. The entity also manufactured 100 products using 250 units of direct materials.

106. What is the direct material price variance?


A. 250 unfavorable
B. 300 favorable
C. 350 favorable
D. 400 unfavorable

107. What is the direct material usage variance?


A. 150 unfavorable
B. 300 unfavorable
C. 250 favorable
D. 350 favorable

Numbers 108 and 109 (Standard costing - Direct labor variance)

Bacolod Company recently set-up its standard costs for its direct labor. The entity sets the
benchmark at 2 direct labor hours per product at a standard rate of P100 per direct labor hour.

During the year, the entity manufactured 10 products using 30 direct labor hours at total direct
labor costs of P2,400 or P80 per direct labor hour.

108. What is the direct labor rate variance?


A. 600 favorable
B. 400 unfavorable
C. 200 favorable
D. 800 unfavorable

109. What is the direct labor efficiency variance?


A. 400 favorable
B. 1,000 unfavorable
C. 600 unfavorable
D. 200 favorable

Page 35
Number 110 (Job Order Costing)
Simple Company employs actual costing for its production. The entity provided the following
data concerning its production during the year:
Decrease in direct materials during the year 500,000
Labor cost during the year 400,000
Actual factory overhead during the year 300,000
Increase in work in process during the year 200,000
Decrease in finished goods during the year 100,000
What is the cost of goods manufactured during the year?
A. 1,200,000
B. 1,000,000
C. 1,400,000
D. 1,100,000

Numbers 111, 112 and 113 (Job order costing)

Marawi Company employs normal costing for its production. The following data are provided
during the current year:

Net purchases of raw materials during the year 500,000


Total labor costs during the year 800,000
Depreciation of factory assets during the year 100,000
Utilities on the factory during the year 300,000
Beginning Ending
Raw materials inventory 200,000 300,000
Work in process inventory 500,000 200,000
Finished goods inventory 600,000 300,000
 The entity uses a single account for its direct material and indirect materials. Indirect material
used is one-fourth of the total direct material used.
 The indirect labor cost is 1/8 of the total labor costs.
 The overhead application rate is 80% of direct labor costs.
 Any over or under application of overhead is considered material.

111. What is the total manufacturing cost during the current year?
A. 1,560,000
B. 1,500,000
C. 1,640,000
D. 1,740,000

112. What is the cost of goods manufactured during the current year?
A. 2,040,000
B. 1,860,000
C. 1,940,000
D. 1,800,000

113. What is the over or under application of overhead?


A. 60,000 over application
B. 140,000 under application
C. 40,000 under application
D. 160,000 over application

Page 36

Numbers 114, 115 and 116 (Joint Product and By-Product Costing)

Silay Company is conducting a joint production at a total costs of P500,000. The joint production
results to the following inventories:

Alt Tab Del


Units produced 20,000 units 10,000 units 5,000 units
Selling price at split P150 P200 P5
off
Alt and Tab are considered main products while Del is considered by-product. The entity
considers its by-product as material. The by-product requires additional processing cost per unit
of P0.80 and its cost of disposal is P0.20 per unit.

114. What is the value to be given to product Del?


A. 25,000
B. 21,000
C. 24,000
D. 20,000

115. What is the joint cost allocated to product Alt if the entity employs physical method?
A. 333,333
B. 316,667
C. 317,333
D. 320,000

116. What is the joint cost allocated to product Tab if the entity employs relative sales value
method?
A. 300,000
B. 200,000
C. 192,000
D. 288,000
Page 37

Number 117 (Just-in-Time Inventory and Backflush Costing)

Talisay Company is employing backflush costing in connection with just-in-time production


process. The entity provided the following production data for the year:

 The entity acquired direct materials during the year at a cost of P100,000
 The entity reported direct labor cost of P200,000.
 The actual factory overhead incurred during the year amounted to P170,000.
 The standard factory overhead application rate is 75% of direct labor cost.
 The ending finished goods inventory is reported at P120,000.

What is the cost of goods sold under backflush costing?


A. 470,000
B. 350,000
C. 330,000
D. 300,000

Numbers 118, 119 and 120 (Backflush costing)

Panay Company has a cycle of 3 days, uses a Raw and In Process Account (RIP) and charges all
conversion costs to cost of goods sold. At the end of each month, all inventories are counted,
conversion costs components are estimated and inventory account balances are adjusted. Raw
material cost is backflushed from Raw and in Process (RIP) Account to finished goods. The
following information is provided for the month of June:

Beginning Balance of RIP account, including P1,000 conversion cost


5,000
Beginning Balance of finished goods account including P6,000 conversion cost
10,000
Raw materials received on credit
400,000
Direct labor cost
300,000
Factory overhead applied
500,000
Ending RIP inventory per physical count, including P7,000 conversion cost
20,000
Ending finished goods inventory per physical count, including P4,000 conversion cost
6,000

118. What is the amount of conversion cost included cost of goods sold in June?
A. 802,000
B. 796,000
C. 794,000
D. 800,000

119. What is the amount of direct materials backflushed from RIP to finished goods?
A. 391,000
B. 404,000
C. 387,000
D. 395,000

120. What is the amount of direct materials backflushed from finished goods to cost of goods
sold?
A. 395,000
B. 400,000
C. 393,000
D. 389,000

Page 38

Numbers 121 and 122 (Activity Based Costing)

Romblon Company is choosing between traditional costing and activity-based costing. The
following data are provided:

Activity-Based Costing

Activity center Cost driver Amount of activity Center


cost
Material handling Kilos handled 100,000 kg.
200,000
Painting Units painted 50,000 units
300,000
Assembly Machine hours 10,000 hours
500,000

Traditional Costing
Traditional Labor hours 100,000 hours
1,000,000

Job 1 contains 3,000 units. It weighs 10,000 kilos and uses 300 machine hours. The direct labor
hours on the job total 7,000 hours.

121. What is the applied overhead under traditional costing?


A. 70,000
B. 60,000
C. 80,000
D. 50,000

122. What is the applied overhead under Activity Based Costing?


A. 53,000
B. 56,000
C. 45,000
D. 43,000
Page 39

Numbers 123, 124, 125 and 126 (Process Costing without Spoilage)

Tacloban Company is employing process costing regarding its production cycle.

Conversion costs are added uniformly during the production process while direct materials are
added 10% at the start of production process, 50% at the middle of the production process and
the remainder at the end of production process.

The production data of the entity during the year are:

Beginning Work in Process Inventory 10,000 units (30% incomplete as to conversion


costs)
Units started during the year 30,000 units
Ending Work in Process Inventory 5,000 units (75% incomplete as to conversion
costs)

 There is no spoilage during the period.


 The costs of beginning inventory consist of P103,000 costs of direct materials and P107,500
conversion costs.
 The total manufacturing costs consist of P252,000 costs of direct materials and P146,250
conversion costs.

123. What is the cost per unit of direct material under average process costing?
A. 10
B. 9
C. 8
D. 7

124. What is the cost per unit of conversion cost under average process costing?
A. 10
B. 9
C. 8
D. 7
125. What is the cost per unit of direct material under FIFO process costing?
A. 10
B. 9
C. 8
D. 7

126. What is the cost per unit of conversion cost under FIFO process costing?
A. 5
B. 9
C. 8
D. 7

Page 40

Numbers 127, 128, 129, 130 and 131 (Process Costing with Spoilage)

Samar Company is employing process costing regarding its production cycle.

Conversion costs are added uniformly during the production process while direct materials are
added 20% at the start of production process, 45% at the middle of the production process and
the remainder at the end of production process. Normal spoilage is 10% of units started during
the year.

The entity is conducting inspection when the production process is at 45% of conversion cost.
The entity provided the following production data during the year:

Beginning Work in Process Inventory 10,000 units (40% incomplete as to conversion


costs)
Units started during the year 40,000 units
Ending Work in Process Inventory 5,000 units (80% complete as to conversion costs)
Units completed during the period 38,000 units

127. What is the abnormal spoilage in units during the year?


A. 7,000 units
B. 4,000 units
C. 3,000 units
D. 2,000 units

128. What is the equivalent unit of production for direct material under average process costing?
A. 42,650 units
B. 41,150 units
C. 38,250 units
D. 43,750 units

129. What is the equivalent unit of production for conversion cost under average process
costing?
A. 44,650 units
B. 45,150 units
C. 43,250 units
D. 46,150 units

130. What is the equivalent unit of production for direct material under FIFO costing?
A. 35,150 units
B. 37,250 units
C. 36,150 units
D. 38,450 units

131. What is the equivalent unit of production for conversion cost under FIFO costing?
A. 39,150 units
B. 41,250 units
C. 37,450 units
D. 38,650 units

Page 41

Numbers 132, and 133 (Foreign currency hedge)

On December 1, 2020, Entity A imported goods at a price of $1,000 payable on March 1, 2021.
In order to hedge this foreign currency denominated importation, Entity A entered into a forward
contract with a bank to purchase $1,000. Entity A is operating in Philippine economy where the
functional currency is Philippine peso. The following direct exchange rates are given:

December 1, 2020 December 31, 2020 March 1, 2021


Buying spot P43 P40 P41
Selling spot 45 44 49
132. What is the foreign currency gain or loss on the hedged item for 2020?
A. 2,000 loss
B. 1,000 gain
C. 3,000 gain
D. 4,000 gain
133. What is the foreign currency gain or loss on the hedging instrument for 2021?
A. 4,000 gain
B. 2,000 loss
C. 1,000 loss
D. 3,000 gain
Page 42

Numbers 134 and 135 (Hedging)

Kline Company purchased inventory on November 30, 2018 for $10,000 payable March 1, 2019.
On December 1, 2018, the entity entered into a forward contract to purchase $10,000in 90 days
to hedge the purchase of inventory on November 30, 2018. The relevant exchange rates are:

Spot rate Forward rate


November 30, 2018 P45 P47
December 1, 2018 46 48
December 31, 2018 50 51

134. What amount of foreign currency transaction gain from the forward contract should be
included in net income for 2018?
A. 50,000
B. 40,000
C. 30,000
D. 0

135. What amount of foreign currency transaction loss should be included in income from the
revaluation of accounts payable for 2018?
A. 40,000
B. 50,000
C. 10,000
D. 0

Number 136 (Hedging)

On December 1, 2018 Winston Company entered into a forward contract to purchase $10,000 in
90 days to hedge a commitment to purchase equipment being manufactured to the entity’s
specifications. The expected delivery date is March 1, 2019, at which time settlement is due to
the manufacturer. The hedge qualifies as a fair value hedge. The relevant exchange rates are:
Spot rate Forward rate
December 1, 2018 P48 P49
December 31, 2018 52 51

What amount of foreign currency transaction gain from the forward contract should be included
in net income for 2018?
A. 20,000
B. 40,000
C. 10,000
D. 0

Page 43

Numbers 137 and 138 (Fair value hedge)

On November 1, 2020, Entity A entered into a firm commitment for the exportation of goods at a
price of $2,000. Delivery will happen on January 31, 2020. In order to hedge this foreign
currency denominated firm commitment, Entity A entered into a forward contract with a bank to
sell $2,000. Entity A is operating in Philippine economy where the functional currency is
Philippine peso. Entity A elects to use fair value hedge to account this hedge of firm
commitment. The following direct exchange rates are given:

November 1, 2020 December 31, 2020 January 31, 2021

Spot rate P43 P40 P44


90-day forward rate 41 43 44
60-day forward rate 45 42 41
30-day forward rate 47 46 42

137. What is the carrying amount of firm commitment asset or liability on December 31, 2020?
A. 4,000 liability
B. 10,000 liability
C. 2,000 liability
D. 6,000 liability

138. What is the foreign currency gain or loss on hedging instrument for 2021?
A. 4,000 gain
B. 2,000 loss
C. 6,000 loss
D. 8,000 gain

Page 44

Numbers 139, 140, 141 and 142 (Cash flow hedge)


On November 1, 2020, Entity A anticipated the purchase of equipment on January 31, 2021 at a
price of $1,200. In order to hedge this highly probable forecasted importation, Entity A entered
into a forward contract with a bank to purchase $1,200. Entity A is operating in Philippine
economy where the functional currency is Philippine peso. The following direct exchange rates
are made available:

November 1, 2020 December 31, 2020 January 31, 2021


Spot rate P45 P44 P43
90-day forward rate 42 41 43
60-day forward rate 46 45 40
30-day forward rate 48 44 40

139. What is the unrealized holding gain or loss to be recognized as component of other
comprehensive income in the statement of comprehensive income for the year ended
December 31, 2020?
A. 2,400 gain
B. 1,200 gain
C. 3,600 loss
D. 4,800 gain

140. What is the unrealized holding gain or loss to be recognized as component of other
comprehensive income in the statement of comprehensive income for the year ended
December 31, 2021?
A. 4,800 loss
B. 1,200 loss
C. 3,600 gain
D. 2,400 gain

141. What is the cumulative unrealized gain or loss before reclassification to be reported as
component of other comprehensive income in the Statement of Changes in equity on
December 31, 2021?
A. 1,200 gain
B. 1,800 loss
C. 2,400 gain
D. 0

142. What is the cost of equipment in Philippine peso on January 31, 2021?
A. 48,000
B. 50,400
C. 49,200
D. 51,600
Page 45

Numbers 143, 144 and 145 (Cash Flow Hedge using option contract)

On November 1, 2020, Entity A anticipated the purchase of inventory on January 31, 2021 at a
price of $1,000. In order to hedge this highly probable forecasted importation, Entity A acquired
a call option from a bank giving it the right to purchase $1,000 at an option price of P40 by
paying an option premium of P300. Entity A is operating in Philippine economy where the
functional currency is Philippine peso. The following data are provided:

November 1, 2020 December 31, 2020 January 31, 2021


Spot rate P40 P 44 P43
Fair value of call option ? 4,500 ?

Entity A imported the goods on the date anticipated. Afterwards, Entity A was able to resell 30%
of the goods imported during 2021.

143. What is the unrealized holding gain or to be recognized as component of other


comprehensive income in the of statement of comprehensive income for the year ended
December 31, 2020?
A. 4,000
B. 4,500
C. 4,300
D. 4,200

144. What is the unrealized holding gain to be recognized in the profit or loss in the statement
comprehensive income for the year ended December 31, 2020?
A. 300
B. 200
C. 500
D. 100

145. What is the unrealized holding loss to be recognized as component of other comprehensive
income in the statement of comprehensive income for the year ended December 31, 2021?
A. 3,000
B. 2,000
C. 1,000
D. 4,000

END
Advanced Financial Accounting and Reporting
Solution in Problem Solving

Number 1 Answer B

Fair Market Value of Land and Building contributed by B 1,500,000


Less: Mortgage Payable to be assumed by ABC Partnership ( 300,000)
Capital Credit of B in ABC Partnership 1,200,000
Divided by B’s Capital Interest Ratio /60%
Total Agreed Capitalization of ABC Partnership 2,000,000

Fair Market Value of Land and Building contributed by B 1,500,000


Less: Mortgage Payable to be assumed by ABC Partnership ( 300,000)
Capital Credit of B in ABC Partnership 1,200,000
Capital Credit of A in ABC Partnership (Proceeds from sale of equipment) 300,000

Number 2 Answer A

Total Agreed Capitalization of ABC Partnership 2,000,000


Less: Total Capital Credit of A and B (P300,000 + P1,200,000) 1,500,000
Cash to be contributed by C in ABC Partnership 500,000

Number 3 Answer A

January 1, 2018 B’s Capital Balance (P1,000,000 x 10%) 100,000


Add: B’s additional investment during 2018 500,000
Less: B’s drawings at the end of 2018 (300,000)
Less: B’s capital balance on December 31, 2018 (200,000)
B’s Share in Net Loss for the year ended December 31, 2018 (100,000)
Divided by B’s interest in profit or loss /20%
Net loss of ABC Partnership for the year ended December 31, 2018 (500,000)

Number 4 Answer B

January 1, 2018 C’s Capital Balance (P1,000,000 x 40%) 400,000


Less: C’s Share in Net Loss during 2018 (P500,000 x 50%) (250,000)
Less: C’s drawings at the end of 2018 (100,000)
December 31, 2018 Capital Balance of C 50,000

Number 5 Answer C
December 31, 2018 C’s Capital Balance 320,000
Add: C’s drawings at the end of 2018 400,000
Less: C’s additional investment during 2018 (300,000)
Less: C’s capital balance on January 1, 2018 (200,000)
C’s share in partnership profit for the year ended December 31, 2018 220,000

C’s share in profit for the year 2018 220,000


Less: Interest on original capital contribution of C (200,000 x 10%) ( 20,000)
C’s share in the remaining profit after interest, salary and bonus 200,000
Multiply by number of partners x 3
Remaining profit after interest, salary and bonus 600,000
Divided by 80% / 80%
Net profit after salary and interest but before bonus to managing partner 750,000
Add: Total interest and salary (100,000 + 200,000) 300,000
Partnership profit for the year ended December 31, 2018 1,050,000
Page 2

Number 6 Answer C

Interest on capital (10% x 300,000) 30,000


Salary (40,000 x 4) 160,000
Bonus to A 150,000
Equal share in remaining profit (600,000 / 3) 200,000
Total share of A in partnership profit 540,000

Partnership profit for the year ended December 31, 2018 1,050,000
Less: Total interest and salary (100,000 + 200,000) (300,000)
Net profit after salary and interest but before bonus to managing partner 750,000
Multiply by Bonus percentage x 20%
Bonus to A as managing partner 150,000

Number 7 Answer B

Interest on capital (10% x 500,000) 50,000


Salary (10,000 x 4) 40,000
Equal share in remaining profit 200,000
Total share of B in partnership profit 290,000

Number 8 Answer B

Capital Balance of B before the admission of D 800,000


Less: Capital to be transferred to D (P800,000 x 40%) ( 320,000)
Capital Balance of B after the admission of D 480,000
Number 9 Answer D

Capital Balance of A before the retirement of C 500,000


Add: Share of A in asset revaluation (250,000 x 10%) 25,000
Capital Balance of A after the retirement of C 525,000

Cash received by C upon retirement 300,000


Capital of C before retirement 200,000
Share of C asset revaluation 150,000

Total asset revaluation (150,000 / 60%) 250,000

Number 10 Answer C

Capital Balance of B before the retirement of C 300,000


Add: Share of B from Bonus given by C (P20,000 x 4/5) 16,000
Capital Balance of B after the retirement of C 316,000

Capital balance of C before retirement 100,000


Cash received by C upon retirement 80,000
Bonus given by C 20,000

Page 3

Number 11 Answer A

Capital Balance of C before the admission of D 900,000


Add: Share of C in bonus given by D (P200,000 x 30%) 60,000
Capital Balance of C after the admission of D (16) (A) 960,000

Capital of A 1,400,000
Capital of B 700,000
Capital of C 900,000
Capital of D 1,000,000
Total capital 4,000,000

Contribution of D 1,000,0000
Interest of D (20% x 4,000,000) 800,000
Bonus given by D 200,000

Number 12 Answer B

Capital credit of D to the new partnership (P3,000,000 x 10%) 300,000

Number 13 Answer A

Capital Balance of C before the admission of D 700,000


Less: C’s share in asset impairment (P500,000 x 40%) (200,000)
Add: C’s share in bonus given by D (P200,000 x 40%) 80,000
Capital Balance of C after the admission of D 580,000

Contributed capital by D to the new partnership 500,000


Less: Capital credit to D in the new partnership (P3,000,000 x 10%) 300,000
Bonus given by D to the existing partners 200,000

Total contributed capital of all partners (P3,000,000 + P500,000) 3,500,000


Less: Total agreed capitalization of new partnership 3,000,000
Asset impairment to be shared by old partners only 500,000

Number 14 Answer A

Capital Balance of B before liquidation ( 650,000)


Add: Payable to B 1,000,000
Capital Balance of B after the right of offset 350,000
Less: Share of B in Total Loss on Liquidation (600,000 x 10%) 60,000
Capital Balance of B after loss on liquidation but before absorption of A’s insolvency 290,000
Less: Share of B in A’s debit capital balance (P160,000 x 1/4) (40,000)
Cash received by B at the end of partnership liquidation 250,000

Noncash assets 2,000,000


Sale price 1,500,000
Loss on sale 500,000
Liquidation expenses 100,000
Total loss on liquidation 600,000

Capital of A 700,000
Receivable from A (500,000)
Net capital of A 200,000
Share in total loss (60% x 600,000) (360,000)
Debit balance in capital of A (160,000)
Page 4

Number 15 Answer B

Capital Balance of C before liquidation 350,000


Add: Payable to C 100,000
Capital Balance of C after the right of offset 450,000
Less: Share of C in Total Loss on Liquidation (600,000 x 30%) 180,000
Capital Balance of C after loss on liquidation but before absorption of A’s insolvency 270,000
Less: Share of C in A’s debit capital balance (P160,000 x 3/4) (120,000)
Cash received by C at the end of partnership liquidation 150,000

Number 16 Answer B

Capital Balance of C before liquidation 400,000


Less: Share of C in Total Loss in Liquidation during January (P500,000* x 20%) (100,000)
Capital Balance of C after loss on liquidation but before absorption of A’s insolvency 300,000
Less: Share of C in A’s debit balance (P150,000 x 2/5) ( 60,000)
Cash received by C at the end of partnership liquidation 240,000

Cash balance before start of liquidation 1,600,000


Add: Net proceeds from sale of noncash asset during January (1,000,000 + 100,000) 1,100,000
Less: Cash paid for liquidation expenses during January (50,000)
Less: Cash paid for liabilities to third person during January (P2,000,000 x 20%) (400,000)
Less: Cash withheld for unpaid liabilities to third persons (P2,000,000 x 80%) (1,600,000)
Less: Cash withheld for estimated future liquidating expenses (150,000)
Cash available for distribution to partners 500,000
Less: Total capital of all partners (100,000+500,000+400,000) (1,000,000)
Total loss on liquidation for the first month of installment 500,000*

Capital of A before liquidation 100,000


Share in loss on liquidation (50% x 500,000) (250,000)
Debit balance in capital of A (150,000)

Number 17 Answer D

Estimated future liquidating expenses on January 31, 2019 150,000


Add: carrying amount of remaining noncash assets on January 31, 2019
(1,400,000 – 1,000,000) 400,000
Maximum possible loss on January 31, 2019 550,000
B’s share in maximum possible loss (P550,000 x 30%) 165,000

Number 18 Answer C
Cash withheld for future liquidating expenses 150,000
Add: Cash withheld for remaining unpaid liabilities to third persons 1,600,000
(P2,000,000 x 80%)
Total cash withheld on January 31, 2019 1,750,000

Number 19 Answer C

Amount received by holder of note payable (NRV of Inventory) 250,000


Note: Only the net realizable value of collateral inventory will be received since
there is no available net free asset.

Page 5

Number 20 Answer D

Amount received by holder of mortgage payable (Fair value of Land) 100,000


Note: The mortgage payable will be fully collected because it is fully secured
credit.

Number 21 Answer B

Cash 100,000
Add: Free assets from fully secured mortgage payable (P120,000 – P100,000) 20,000
Total Free assets for unsecured credits with priority 120,000
Amount received by employees for their salary 120,000
Note: Since only P120,000 free assets are available, it must all be given to
employees who are preferred over the government.

Number 22 Answer C

Amount received by partially secured loan payable 340,000


Less: Fair value of collateral – machinery (300,000)
Recovered amount from the unsecured portion of partially secured loans payable 40,000
Divided by unsecured portion of partially secured loan payable (400,000 – 300,000) /100,000
Recovery percentage on unsecured credits 40%

Amount received by holder of accounts payable (P100,000 x 40%) 40,000


Number 23 Answer A

Accounts payable 100,000


Add: Unsecured portion of partially secured loan payable (400,000 – 100,000
P300,000)
Total unsecured credits including unsecured portion of partially secured loans 200,000
payable
Multiply by recovery percentage of unsecured credits x 40%
Net free assets 80,000

Number 24 Answer C
Land owned by Entity A 3,000,000
Add: Interest of Entity A on co-owned inventory (P1,000,000 x 60%) 600,000
Total assets to be reported by Entity A concerning its interest in Entity C 3,600,000

Number 25 Answer A

Notes payable owed by Entity B 1,000,000


Add: Interest of Entity B on co-owed accounts payable (P2,000,000 x 40%) 800,000
Total liabilities to be reported by Entity B concerning its interest in Entity 1,800,000
C

Number 26 Answer B

Sales revenue reported by Entity C 5,000,000


Less: Unsold inventory of Entity A coming from Entity C (P1,000,000 x 70%) (700,000)
Less: Unsold inventory of Entity B coming from Entity C (P2,000,000 x 40%) (800,000)
Sales revenue to third persons 3,500,000
Sales revenue to be reported by Entity A (P3,500,000 x 60%) 2,100,000

Page 6

Number 27 Answer C

Entity A’s investment in Entity C – 1/1/2018 1,000,000


Share in 2018 net income of Entity C (2,000,000 x 40%) 80,000
Share in 2018 cash dividend of Entity C (100,000 x 40%) ( 40,000)
Share in 2019 net loss of Entity C (2,000,000 x 40%) ( 800,000)
Carrying amount – December 31, 2019 240,000

Number 28 Answer C

Entity B’s investment in Entity C 1,500,000


Share in 2018 net income of Entity C (200,000 x 60%) 120,000
Share in 2018 cash dividend of Entity C (100,000 x 60%) ( 60,000)
Share in 2018 net loss of Entity C (2,000,000 x 60%) (1,200,000)
Carrying amount – December 31, 2019 360,000

Number 29 Answer C

Unadjusted investment income of Entity A for 2018 (P1,000,000 x 60%) 600,000


Less: Unrealized gross profit in ending inventory of Entity A (P50,000 x 20% x 60%) ( 6,000)
Adjusted investment income of Entity A for 2018 594,000

Number 30 Answer A

Initial measurement of Investment in Entity C (Entity B’s Book) 2,000,000


Add: Unadjusted investment income of Entity B for 2018 (P1,000,000 x 40%) 400,000
Add: Unrealized loss on sale of machinery (P20,000 x 40%) 8,000
Less: Realized loss on sale of machinery (P20,000/2 x 6/12 x 40%) ( 6,000)
Less: 2018 dividend received from Entity C (P400,000 x 40%) ( 160,000)
December 31, 2018 Investment in Entity C (B’s Book ) 2,242,000

Number 31 Answer A

Transaction costs – Expense as incurred under Fair Value Model ( 20,000)


Unrealized holding gain on change in fair value 60,000
Dividend income (P30,000 x 50%) 15,000
Effect on net profit under Fair Value Model 55,000

Fair value (10,000 x 56) 560,000


Acquisition cost 500,000
Unrealized gain 60,000

Number 32 Answer D

Initial measurement of Investment under Equity Method (P500,000 + P20,000) 520,000


Add: Share in net income of Joint Venture (P100,000 x 50%) 50,000
Less: Dividend received from Joint Venture (P30,000 x 50%) ( 15,000)
Investment on December 31, 2018 under equity method 555,000
Note: There is no impairment loss because fair value less cost to sell of P560,000 is
higher than carrying amount.

Number 33 Answer A

Carrying amount of Investment under Cost Method (P200,000 + P10,000) 210,000

Page 7
Number 34 Answer C

Share in net income of joint venture (P50,000 x 50%) 25,000


Impairment loss of Investment under equity method (15,000)
Effect on net profit under Equity Method 10,000

Investment in Entity C (200,000 + 10,000) 210,000


Share in 2018 net income of Entity C (50,000 x 50%) 25,000
Share in cash dividend (50% x 10,000) ( 5,000)
Carrying amount – 12/31/2018 230,000
Recoverable amount – value in use 215,000
Impairment loss 15,000

Number 35 Answer A
2018 2019
Sales 300,000 450,000
Collections ( 150,000) ( 150,000)
Accounts written off ( 100,000) ( 150,000)

Installment accounts receivable, 12/31/2019 50,000 150,000


Gross profit rates 30% 40%

Deferred gross profit, 12/31/2019 15,000 60,000


Total (P75,000)

Number 36 Answer B
2018 2019 2020 Total
Collections 72,500 80,000 62,500
Gross profit rate:
2018: 60,000/240,000 25%
2019: 68,750/250,000 27.5%
2020: 84,000/300,000 _______ _______ 28%

Realized gross profit 18,125 22,000 17,500 57,625

Number 37 Answer C
2018 sale 2019 sale
Unrecovered cost:
Unpaid balance 15,000 16,000
Less: Deferred gross profit
2018: 15,000 x 25% 3,750
2019: 16,000 x 27.5% _______ 4,400
Unrecovered cost 11,250 11,600
Value of repossess merchandise 6,000 9,000
Loss on repossession ( 5,250) ( 2,600) (7,850)

Number 38 Answer B
2019 Sales 2020 Sales
Deferred gross profit – December 31, 2020 9,000 72,000
Divide by GPR (GP/IS) 24% 30%
Installment accounts receivable, December 31, 2020 37,500 240,000

Total balance of receivable on December 31, 2020 is (37,500 + 240,000) 277,500


Page 8

Number 39 Answer B

Unadjusted installment sales 400,000


Add: Undervaluation of traded car (P150,000 – P50,000) 100,000
Adjusted installment sales 500,000
Less: Cost of production of car ( 300,000)
Adjusted gross profit 200,000
Divided by Adjusted installment sales /500,000
Adjusted gross profit rate based on sales 40%

Installment receivable 400,000


Down payment 25% x 400,000) (100,000)
Trade in allowance ( 50,000)
Installment receivable balance 250,000

Annual installment (250,000/5) 50,000

Down payment 100,000


Fair value of trade in 150,000
Annual payment – first year 50,000
Total collections 2018 300,000

Realized gross profit 2018 (40% x 300,000) 120,000

Number 40 Answer C

Fair value of repossessed inventory 110,000


Less: Unrecovered cost of defaulted installment receivable (P200,000 x 60%) (120,000)
Loss on Repossession ( 10,000)

Number 41 Answer B

Allocated revenue to construction of stall (P400,000 x 20/50) 160,000

Stand-alone selling price Fraction


Construction of stall 200,000 20/50
Purchase of raw materials 250,000 25/50
Tradename 50,000 5/50
500,000

Number 42 Answer C

Revenue from delivery of raw materials (P400,000 x 25/50) x 3,000/10,000 60,000

Number 43 Answer B

Revenue from use of entity’s trade name (P400,000 x 5/50)/10years 4,000

Number 44 Answer D

Cash downpayment 200,000


Present value of note receivable 240,183
Initial franchise fee revenue 440,183
Less: Direct cost of initial franchise fee 352,146
Gross profit under accrual basis 88,037
Page 9
Number 45 Answer A

Gross profit under accrual basis 88,037


Add: Interest Income for year 2018 (P240,183 x 12%) 28,822
Add: Contingent franchise fee revenue (P50,000 x 8%) 4,000
Less: Indirect cost – Expense as incurred (22,009)
Net income under accrual basis 98,850

Number 46 Answer B

Costs incurred to date as of December 31, 2018 440,000


Divided by total cost as of 2018 (440,000+P660,000) /1,100,000
Percentage of completion for 2018 40%

Construction revenue for year 2018 (1,000,000 x 40%) 400,000

Number 47 Answer C

Costs incurred to date as of December 31, 2019 (440,000+680,000) 1,120,000


Divided by total cost as of 2019 (440,000+680,000+280,000) / 1,400,000
Percentage of completion for 2019 80%

Contract price 1,500,000


Total cost 2019 1,400,000
Gross profit 100,000
Multiply by ____80%
Cumulative realized gross profit 2019 80,000
Add : Realized loss 2018 100,000
Realized gross profit 2019 180,000

Contract price 2018 1,000,000


Total cost 2018 1,100,00
Realized loss for 2018 ( 100,000)

Number 48 Answer A

Contract price as of December 31, 2019 1,500,000


Multiply by percentage of completion as of December 31, 2019 x 80%
Construction in Progress on December 31, 2019 1,200,000

Number 49 Answer C

Contract price as of 2019 1,000,000


Less: Total costs as of 2019 (800,000+250,000) (1,050,000)
Cumulative gross loss for 2019 ( 50,000)
Realized gross loss for 2018 ( 200,000)
Realized gross profit for 2019 150,000

Contract price 1,000,000


Total cost 2018 (360,000 + 840,000) 1,200,000
Realized loss 2018 ( 200,000)

Number 50 Answer A

Progress Billings as of December 31, 2020 (P1,000,000) x (30%+20%+40%) 900,000


Less: Construction in Progress as of December 31, 2020 (Costs incurred to date) 870,000
Excess of Progress Billings over Construction in Progress on 12/31/2020 ( 30,000)
Page 10

Number 51 Answer B

Cumulative billings as of December 31, 2020 (P1,000,000) x (30%+20%+40%) 900,000


Mobilization fee deductible from first billing (P1,000,000 x 5%) ( 50,000)
Total of receivables as of December 31, 2020 (120,000+450,000+180,000) (750,000)
Balance of Accounts Receivable on December 31, 2020 100,000

Number 52 Answer A

Sales revenue of the branch 500,000


Cost of goods sold of the branch (30,000+100,000+250,000-50,000) (330,000)
Operating expenses of the branch ( 40,000)
Net income reported by the branch in its separate income statement 130,000

Number 53 Answer D

Ending inventory of the home office at cost 80,000


Ending inventory of the branch from outsider at cost (50,000 x 26%) 13,000
Ending inventory of the branch from home office last year at cost (50,000 x 24%) / 9,600
1.25)
Ending inventory of the branch from home office this year at cost (50,000 x 50%) / 20,000
1.25)
Ending inventory of the entity combined statement of financial position 122,600

Number 54 Answer D

Overstatement of beginning inventory from home office 4,800


Overstatement of shipment during the year (250,000 – 200,000) 50,000
Unadjusted overvaluation of inventory from home office 54,800
Overstatement of ending inventory from home office ( 7,400)
Overstatement of cost of goods sold 47,400

Beginning inventory from home office (80% x 30,000) 24,000


Cost (24,000 / 125%) 19,200
Overstatement of beginning inventory from home office 4,800

Ending inventory from home office (74% x 50,000) 37,000


Cost (37,000 / 125%) 29,600
Overstatement of ending inventory from home office 7,400

Number 55 Answer C

Home Office account before branch profit 180,000


Branch loss
Sales 225,500
Cost of goods sold:
Shipments from home office 250,000
Inventory, 12/31 65,000 185,000
Gross profit 40,500
Expenses 55,500 ( 15,000)
Home Office account balance, 12/31 165,000

Number 56 Answer C

Acquired from Home Office (80% x P30,000)/120% 20,000


Acquired from outsiders (20% x P30,000) 6,000
Branch inventory at cost, 12/31 26,000

Page 11

Number 57 Answer A

Sales (74,000+22,000) 96,000


Cost of goods sold:
Shipment from Home Office at cost (54,000/120%) 45,000
Purchases 26,000
Cost of goods available for sale 71,000
Inventory, at cost, 12/31 26,000 45,000
Gross profit 51,000
Expenses (38,000 + 12,000) 50,000
Branch net income insofar as Home Office is concerned 1,000
Number 58 Answer A

Entity B (acquiree)

Current assets 500,000


Noncurrent assets at fair value 1,300,000
Current liabilities at fair value ( 600,000)
Noncurrent liabilities ( 500,000)
Net assets of acquiree at fair value 700,000

Shares issued at fair value (10,000 x 20) 200,000


Bonds payable issued at fair (500,000 x 110%) 150,000
Total consideration 750,000
Net assets of acquiree at fair value 700,000
Goodwill 50,000

Number 59 Answer B

Direct cost of business combination 40,000


Indirect cost of business combination 30,000
Total amount to be expensed as incurred 70,000

Number 60 Answer C

Total assets of A at carrying amount:


Current assets 1,000,000
Noncurrent assets 2,000,000
Total assets of B at fair value:
Current assets 500,000
Noncurrent assets 1,300,000
Goodwill 50,000
Total consolidated assets 4,850,000
Payment for:
Share issuance cost ( 10,000)
Bond issue cost ( 20,000)
Acquisition related cost ( 40,000)
Indirect cost of combination ( 30,000)
Total assets after business combination 4,750,000
Page 12
Number 61 Answer D

Total liabilities of A at carrying amount:


Current liabilities 200,000
Noncurrent liabilities 300,000
Total liabilities of B at fair value:
Current liabilities 600,000
Noncurrent liabilities 500,000
Fair value of bonds payable issued (500,000 x 110%) 550,000
Bond issue cost ( 20,000)
Total liabilities after combination 2,130,000

Number 62 Answer B

Controlling interest (80,000/100,000) 80%


Noncontrolling interest 20%

Net assets of acquiree 1,600,000


Overvaluation of asset ( 60,000)
Undervaluation of liability ( 40,000)
Fair value of net assets of acquiree 1,500,000
Multiply by 20%
Noncontrolling interest 300,000

Number 63 Answer D

Net assets of acquiree at carrying amount 1,600,000


Overvaluation of asset ( 60,000)
Undervaluation of liability ( 40,000)
Net assets of acquiree at fair value 1,500,000

Acquisition price 1,000,000


Net assets acquired (80% x 1,500,000) 1,200,000
Gain on bargain purchase 200,000

Number 64 Answer A

Existing interest (30,000 / 100,000) 30%

Existing interest on 1/1/2018 90,000


Share in net income of associate for six months (30% x 40,000) 12,000
Carrying amount 102,000
Fair value of existing interest (30,000 x 4) 120,000
Gain on remeasurement 18,000

Number 65 Answer D

Fair value of existing interest 120,000


Cost of additional interest 240,000
Fair value of noncontrolling interest 50,000
Total consideration 410,000
Fair value of net assets acquired ( 380,000)
Goodwill 30,000

Net assets of acquiree at carrying amount 400,000


Overvaluation of asset ( 50,000)
Overvaluation of liability 30,000
Fair value of net assets of acquiree 380,000
Page 13

Number 66 Answer D

Initial measurement of noncontrolling interest in net assets (P330,000 x 30%) 99,000


Noncontrolling interest in net income 40,800
Dividends declared by Entity B for NCI owners (20,000 x 30%) ( 6,000)
Noncontrolling interests in net assets on December 31, 2018 133,800

Net income reported by Entity B in its separate income statement 150,000


Amortization of undervaluation of machinery (80,000/4 years) ( 20,000)
Amortization of overvaluation of inventory (10,000 x 60%) 6,000
Adjusted net income of Entity B 136,000
Multiply by noncontrolling interest percentage of ownership x 30%
Noncontrolling interest in net income for 2018 40,800

Net assets of Entity B – 1/1/2018 260,000


Undervaluation of machinery 80,000
Overvaluation of inventory ( 10,000)
Net assets of Entity B at fair value 330,000
Number 67 Answer A

Net income reported by Entity A in its separate income statement 1,000,000


Gain on bargain purchase 21,000
Dividend income from Entity B (20,000 x 70%) ( 14,000)
Share in adjusted net income of Entity B (136,000 x 70%) 95,200
Consolidated net income attributable to parent shareholders 1,102,200

Acquisition cost 210,000


Fair value of noncontrolling interest 99,000
Total 309,000
Net assets of Equity B at fair value 330,000
Gain on bargain purchase 21,000

Reported net income of Entity B 150,000


Depreciation of undervalued machinery (80,000/4 years) ( 20,000)
Overvalued inventory sold (60% x 10,000) 6,000
Adjusted net income of Entity B 136,000
Multiply by controlling interest __70%
Share in net income of Equity B 95,200

Number 68 Answer C

Retained earnings of Entity A on January 1, 2018 2,000,000


Consolidated net income attributable to parent shareholders 1,102,200
Dividends declared by Entity B during 2018 ( 150,000)
Consolidated retained earnings on December 31, 2018 2,952,200

Number 69 Answer A

Sales revenue reported by Entity A for year 2020 2,000,000


Sales revenue reported by Entity B for year 2020 1,000,000
Intercompany sales during 2020 ( 400,000)
Consolidated sales revenue for year 2020 2,600,000

Page 14
Number 70 Answer B
Gross profit of Entity A for year 2020 (2,000,000 – 1,200,000) 800,000
Gross profit of Entity B for year 2020 (1,000,000 – 700,000) 300,000
Realized Gross Profit on January 1, 2020 inventory of Entity B 20,000
Unrealized Gross Profit December 31, 2020 of Entity A ( 72,000)
Consolidated gross profit for 2020 1,048,000

January 1, 2020 inventory of Entity B 280,000


Cost (280,000 / 140%) 200,000
Unrealized profit – 1/1/2020 80,000

Realized profit in 2020 – sold portion in 2020 (25% x 80,000) 20,000

December 31, 2020 inventory of Entity A 400,000

Gross profit (30% x 400,000) 120,000


Realized gross profit – sold portion in 2020 (40% x 120,000) ( 48,000)
Unrealized gross profit – December 31, 2020 72,000

Number 71 Answer C

Net income reported by Entity B in its separate income statement 200,000


Unrealized gross profit on upstream sale to Entity A ( 72,000)
Adjusted net income of Entity B 128,000
Multiple by noncontrolling interest x 40%
Noncontrolling interest in net income 51,200

Number 72 Answer D

Net income reported by Entity A in its separate income statement 500,000


Dividend income from Entity B (50,000 x 60%) 30,000
Realized gross profit on downstream sale to Entity B 20,000
Share in adjusted net income of Entity B (128,000 x 60%) 76,800
Consolidated net income attributable to parent’s shareholders 626,800

Number 73 Answer A

Depreciation of White Machinery ( 40,000 / 4 years) 10,000


Depreciation of Black Machinery (180,000 / 6 years) 30,000
Consolidated depreciation expense for 2020 40,000

Number 74 Answer B
White machinery 200,000
Accumulated depreciation ( 40,000)
Carrying amount – January 1, 2019 160,000
Depreciation – 2019 ( 10,000)
Depreciation – 2020 ( 10,000)
Carrying amount – December 31, 2020 140,000

Black machinery 270,000


Accumulated depreciation 180,000
Carrying amount – July 1, 2020 90,000
Depreciation for six months (30,000 x 6/12) ( 15,000)
Carrying amount – December 31, 2020 75,000

Total carrying amount (140,000 + 75,000) 215,000


Page 15

Number 75 Answer C

Net income reported by Entity B in its separate income statement 500,000


Realized gain on upstream sale of land 100,000
Unrealized loss on upstream sale of black machinery 30,000
Realized loss on upstream sale ( 5,000)
Adjusted net income of Entity B for 2020 625,000
Multiply by noncontrolling interest x 20%
Noncontrolling interest in net income for 2020 125,000

Sale price of land to Entity A 1,100,000


Cost of land 1,000,000
Realized gain on sale of land – sold in 2020 100,000

Sale price of black machinery to Entity A 60,000


Carrying amount (270,000 – 180,000) 90,000
Unrealized loss – July 1, 2020 30,000

Realized position of loss (30,000 / 3 years remaining x 6/12) 5,000

Cost of black machinery 270,000


Divide by annual depreciation (180,000 / 6 years expired) 30,000
Total life 9 years
Years expired 6
Remaining useful life 3 years

Number 76 Answer D

Net income reported by Entity A in its separate income statement 800,000


Dividend income from Entity B (150,000 x 80%) 120,000
Realized gain on downstream sale of white machinery ( 1,250)
Share in adjusted net income of Entity B (625,000 x 80%) 500,000
Consolidated net income attributable to parent shareholders 1,418,750

Cost of white machinery 200,000


Divide by annual depreciation (40,000 / 4 years) 10,000
Total life 20 years
Years expired 4
Remaining useful life 16 years

Sale price of white machinery to Entity B 180,000


Carrying amount (200,000 – 40,000) 160,000
Unrealized gain on sale – January 1, 2019 20,000

Realized gain on sale (20,000 / 16 years) 1,250

Number 77 Answer B

Acquisition price 900,000


Transaction cost _20,000
Carrying amount – December 31, 2020 920,000

Number 78 Answer B

Dividend income (90% x 30,000) 27,000

Page 16

Number 79 Answer C

Fair value on December 31, 2020 1,000,000

The cost of disposal is ignored.

Number 80 Answer D

Transaction cost which is expensed as incurred 20,000


Dividend income from subsidiary 27,000
Unrealized holding gain on change in fair value 100,000
Net effect on profit or loss under fair value model 107,000
Fair value of investment 1,000,000
Acquisition cost _900,000
Unrealized gain on change in fair value 100,000

Number 81 Answer D

Permanently restricted net asset 1,000,000

Only the fund which is to be invested indefinitely is considered permanent or regular


endowment fund.

Number 82 Answer B

Unspent dividend received for research 100,000


Unused fund for acquisition of service car (300,000 – 100,000) 200,000
Temporarily restricted net assets 300,000

Dividend received 150,000


Spent for research project ( 50,000)
Unspent dividend received 100,000

Fund for acquisition of service car 300,000


Fund used for acquisition (100,000)
Unused fund for acquisition of service car 200,000

Number 83 Answer A

Reclassification from temporarily restricted dividend received 50,000


Reclassification from temporarily restricted service car fund 100,000
Fund subject to discretion of board of trustees 500,000
Gain on sale of souvenir items (150,000 – 100,000) 50,000
Research expense (50,000)
Depreciation expense of service car (100,000/5 x 6/12) (10,000)
Unrestricted net assets 640,000

Sale price of souvenir items 150,000


Cost of souvenir items 100,000
Gain on sale 50,000
Page 17

Number 84 Answer A

Increase in temporarily restricted net assets during 2020 100,000

The receipt of the dividend income is classified as increase in temporarily restricted net
assets because it is restricted for acquisition of computer but none has been spent during
2020.

Number 85 Answer C

Reclassification from temporarily restricted net assets to unrestricted net assets during 2021 20,000
Depreciation expense of computer during 2021 (20,000/5 years) ( 4,000)
Increase in unrestricted net asset during 2021 16,000

Number 86 Answer B

All cash receipts with donor stipulation shall be classified in the Statement of Cash Flows
as financing activities

Cash donation to be invested indefinitely 1,000,000


Dividend received from investment for acquisition of computers _100,000
Cash receipts from financing activities 1,100,000

Number 87 Answer C

Cash disbursements for investing activities 20,000

All cash disbursement for acquisition of noncurrent asset shall be classified in the Statement
of Cash Flows as investing activities.

Cash paid for computer 20,000

88. D

89. C

90. A
91. A

92. A

Number 93 Answer C

1. To record the sale on October 15, 2018 in US dollar:


Accounts receivable 100,000
Sales 100,000

2. To record the remittance on November 16, 2018:


Cash 105,000
Accounts receivable 100,000
Foreign exchange gain 5,000

Page 18

Number 94 Answer D

September 1, 2018 (250,000 x $.20) 50,000


December 31, 2018 (250,000 x $.19) 47,500
Foreign exchange loss 2018 ( 2,500)

December 31, 31, 2018 47,500


February 1, 2019 (250,000 x $.22) 55,000
Foreign exchange gain 2019 7,500

Number 95 Answer B

Foreign currency payable – November 30, 2018 is (300,000 x $1.65) 495,000


Foreign currency payable – December 31, 2018 (300,000 x $1.62) 486,000
Foreign currency gain __9,000

Number 96 Answer D

Foreign currency payable – January 20, 2018 90,000


Foreign currency payment – March 20, 2018 96,000
Foreign currency loss 6,000

Foreign currency note payable 500,000


Dollar equivalent on December 31, 2018 520,000
Foreign currency loss 20,000

Foreign currency interest payable December 31, 2018 (500,000 x 10% x 6/12) 25,000
Dollar equivalent on December 31, 2018 26,000
Foreign currency loss 1,000

Total foreign currency loss for 2018 (6,000 + 20,000 +1,000) 27,000

Number 97 Answer C

Royalties for 2019 in Canadian dollars (10% x 50,000) 5,000


Multiply by spot rate – December 31, 2019 __.89
Royalties payable – December 31, 2019 4,450

Number 98 Answer A

Sales revenue at transaction rate on December 1, 2020 ($1,500 x P39) 58,500

Nonmonetary item such as sales shall be translated at transaction rate.

Number 99 Answer C

Carrying amount of accounts receivable at closing rate on December 1, 2020


($1,500 x P45) 67,500

Monetary item such as accounts receivable shall be translated at closing rate.

Page 19

Number 100 Answer D

Carrying amount of accounts payable at closing rate on December 31, 2020


($1,000 x P47) 47,000

Monetary item such as accounts payable shall be translated at closing rate.

Number 101 Answer A


Accounts receivable – December 1, 2020 ($1,500 x 39) 58,500
Accounts receivable – December 31, 2020 ($1,500,000 x 45) 67,500
Foreign currency gain 9,000

Accounts payable – November 1, 2020 ($1,000 x 42) 42,000


Accounts payable – December 31, 2020 ($1,000 x 47) 47,000
Foreign currency loss ( 5,000)

Net foreign currency gain (9,000 – 5,000) 4,000

Number 102 Answer A

Ordinary share capital 5,000


Preference share capital 8,000
Retained earnings _7,000
Net assets in US dollar – December 31, 2020 20,000
Net income for 2020 ( 1,000)
Dividends declared on December 1, 2020 __200
Net assets – December 31, 2019 19,200

Number 103 Answer A

Net assets at December 31, 2019 rate ($19,200 x P43) 825,600


Net income during 2020 at average rate ($1,000 x P44) 44,000
Dividends declared during 2020 at transaction rate on December 1, 2020 ($200 x P41) ( 8,200)
Net assets at December 31, 2020 at rolled amount 861,400
Net assets at December 31, 2020 at December 31,2020 rate ($20,000 x 45) (900,000)
Translation gain during 2020 in OCI 38,600

Number 104 Answer B

Retained earnings on December 31, 2019 at translated amount 300,000


Net income during 2020 at average rate ($1,000 x P44) 44,000
Dividends declared on December 1, 2020 at transaction rate ($200 x P41) ( 8,200)
Retained earnings on December 31, 2020 at translated amount 335,800

Number 105 Answer B

Total assets at closing rate on December 31, 2020 ($50,000 x P45) 2,250,000

Total liabilities at closing rate on December 31, 2020 ($30,000 x P45) 1,350,000
Ordinary shares at transaction rate on January 1, 2019 ($5,000 x P40) 200,000
Preference shares at transaction rate on July 1, 2019 ($8,000 x P42) 336,000
Retained earnings on December 31, 2020 at translated amount 335,800
Cumulative translation credit on December 31, 2020 (SQUEEZE) 28,200
Total liabilities and shareholders’ equity on December 31, 2020 2,250,000

Page 20

Number 106 Answer D

Actual cost per unit 6.00


Standard cost per unit 5.00
Direct material variance 1.00
Multiply by units acquired 400
Unvaporable variance 400

Number 107 Answer C

Standard units (100 x 3) 300


Direct materials used 250
Direct material usage variance 50
Multiply by standard cost 5
Favorable variance 250

Number 108 Answer A

Actual direct labor cost per hour 80


Standard direct labor cost 100
Direct labor rate variance 20
Multiply by hours used _30
Favorable variance 600

Number 109 Answer B

Actual direct labor hours 30


Standard direct labor hours (10 x 2) __20
Direct labor efficiency variance 10
Multiply by standard direct labor cost 100
Unfavorable variance 1,000

Number 110 Answer B

Decrease in direct materials during the year 500,000


Labor cost during the year 400,000
Actual factory overhead during the year 300,000
Total manufacturing costs during the year 1,200,000
Increase in work in process during the year ( 200,000)
Cost of goods manufactured during the year 1,000,000

Number 111 Answer A

Raw materials inventory beginning 200,000


Net purchases of raw materials 500,000
Raw materials inventory ending ( 300,000)
Raw materials used 400,000
Indirect materials used (400,000 x ¼) ( 100,000)
Direct materials used 300,000
Direct labor costs (800,000 x 7/8) 700,000
Applied Factory Overhead (700,000 x 80%) 560,000
Total manufacturing costs 1,560,000

Page 21

Number 112 Answer B

Total manufacturing costs 1,560,000


Work in process inventory beginning 500,000
Work in process inventory ending ( 200,000)
Total costs of goods manufactured 1,860,000

Number 113 Answer C

Indirect materials used (400,000 x ¼) 100,000


Indirect labor costs (800,000 x 1/8) 100,000
Depreciation of factory assets 100,000
Utilities on the factory during the year 300,000
Actual factory overhead during the year 600,000
Applied factory overhead (700,000 x 80%) (560,000)
Under application of factory overhead 40,000

Number 114 Answer D

Sale price at split off - Del 5.00


Processing cost ( 0.80)
Cost of disposal ( 0.20)
Net realizable value per unit 4.00
Multiply by number of units 5.00
Value of product Del 20.000

Number 115 Answer D

Units Fraction Cost


Product Alt 20,000 2/3 320,000
Product Tab 10,000 1/3 160,000
30,000 480,000

Total cost 500,000


Allocated to by-product Del ( 20,000)
Joint cost 480,000

Number 116 Answer C

Sale price at split off Fraction Cost


Alt (20,000 x 150) 3,000,000 3/5 288,000
Tab (10,000 x 200) 2,000,000 2/5 192,000
5,000,000 480,000

Number 117 Answer C

Direct materials purchased/used during the year 100,000


Direct labor costs during the year 200,000
Standard factory overhead (200,000 x 75%) 150,000
Finished goods inventory ending (120,000)
Cost of goods sold under backflush costing 330,000

Page 22

Number 118 Answer B

Conversion cost in Beginning Raw and in Process account 1,000


Conversion cost in Beginning Finished goods account 6,000
Conversion cost during the year (300,000 labor and 500,000 overhead) 800,000
Conversion cost in Ending Raw and In Process Account ( 7,000)
Conversion cost in Ending Finished Goods account ( 4,000)
Conversion cost included in cost of goods sold 796,000

Number 119 Answer A

Raw materials in Beginning Raw and in Process Account (5,000 – 1,000) 4,000
Raw materials in received 400,000
Raw materials in Raw and in Process Ending Inventory (13,000)
Raw materials backflushed to finished goods 391,000

Ending RIP, including conversion cost 20,000


Conversion cost ( 7,000)
Raw materials in RIP ending inventory 13,000

Number 120 Answer C

Raw Materials in Beginning Finished goods account 4,000


Raw Materials backflushed to finished goods 391,000
Raw Materials in Ending Finished Goods account ( 2,000)
Raw materials backflushed to cost of goods sold 393,000

Beginning finished goods including conversion cost 10,000


Conversion cost (6,000)
Raw materials in beginning finished goods 4,000

Ending finished goods including conversion cost 6,000


Conversion cost (4,000)
Raw materials in ending finished goods 2,000

Number 121 Answer A

Direct labor rate (1,000,000 / 100,000 bonus) 10


Multiply by direct labor hours used 7,000
Applied overhead 70,000

Number 122 Answer A

Materials handling per kg. (200,000 / 100,000) 2.00


Painting per unit (300,000 / 50,000) 6.00
Assembly per hour (500,000 / 10,000) 50.00

Material handling (10,000 kilos x 2.00) 20,000


Painting (3,000 units x 6.00) 18,000
Assembly (300 hours x 50,000) 15,000
Applied overhead 53,000

Page 23

Number 123 Answer A

Number 124 Answer D

Number 125 Answer C

Number 126 Answer A

Beginning Work in Process Inventory in units 10,000 units


Add: Units started during the period 30,000 units
Less: Ending Work in Process Inventory in units (5,000 units)
Units completed during the period 35,000 units

Direct materials Conversion


cost
Units completed 35,000 35,000 35,000
Ending Inventory 5,000 (5,000 x 10%) 500 (5,000 x 25%) 1,250
EUP under average costing 35,500 36,250
Total costs for average costing P103,000+P252,000
P107,500+P146,250
Divide by EUP under average costing 35,500 units 36,250 units
Cost per unit under average costing P10/unit (123) (A) P7/unit (124)
(D)

Direct materials Conversion


cost
Beginning inventory 10,000 (10,000 x 60%) 6,000 (10,000 x
30%) 3,000
Started and Completed 25,000 25,000 25,000
Ending Inventory 5,000 (5,000 x 10%) 500 (5,000 x 25%)
1,250
EUP under FIFO costing 31,500 29,250
Total costs for FIFO costing P252,000 P146,250
Divide by EUP under FIFO costing 31,500 units 29,250 units
Cost per unit under FIFO costing P8/unit (125) (C) P5/unit (126)
(A)

Number 127 Answer C

Beginning Work in process Inventory in units 10,000 units


Units started during the period 40,000 units
Units completed during the year (38,000 units)
Ending Work in Process Inventory in units (5,000 units)
Total spoilage in units 7,000 units
Normal spoilage during the year (40,000 units x 10%) (4,000 units)
Abnormal spoilage in units 3,000 units

Number 128 Answer A

Number 129 Answer B

Direct materials Conversion


cost
Units completed 38,000 38,000 38,000
Ending Inventory 5,000 (5,000 x 65%) 3,250 (5,000 x 80%) 4,000
Normal loss 4,000 (4,000 x 20%) 800 (4,000 x 45%) 1,800
Abnormal loss 3,000 (3,000 x 20%) 600 (3,000 x 45%) 1,350
EUP under average costing 42,650 45,150

Page 24

Number 130 Answer C

Number 131 Answer A

Direct materials Conversion


cost
Beginning inventory 10,000 (10,000 x 40%) 4,000 (10,000 x 35%) 3,500
US and Completed 28,000 28,000 28,000
Ending Inventory 5,000 (5,000 x 65%) 3,250 (5,000 x 80%) 4,000
Normal loss 4,000 (4,000 x 20%) 800 (4,000 x 45%) 1,800
Abnormal loss 3,000 (3,000 x 20%) 600 (3,000 x 45%) 1,350
EUP under FIFO costing 36,650 39,150

Number 132 Answer B

Accounts payable – December 1, 2020 ($1,000 x 45) 45,000


Accounts payable – December 31, 2020 ($1,000 x 44) 44,000
Foreign currency gain 1,000

Number 133 Answer A

Forward contract receivable – December 1, 2020 ($1,000 x 45) 45,000


Forward contract contract receivable – March 1, 2021 ( $1,000 x 49) 49,000
Foreign carrying gain 4,000

Number 134 Answer C

Forward contract receivable – 12/1/2018 ($10,000 x P48 forward rate) 480,000


Forward contract receivable – 12/31/2018 ($10,000 x P51 forward rate) 510,000
Forward contract gain 30,000

Number 135 Answer B

Accounts payable – November 30, 2018 ($10,000 x P45 spot rate) 450,000
Accounts payable – December 31, 2018 ($10,000 x P50 spot rate) 500,000
Foreign currency loss ( 50,000)

Number 136 Answer A

Forward contract receivable – 12/1/2018 ($10,000 x P49 forward rate) 490,000


Forward contract receivable – 12/31/2018 ($10,000 x P51 forward rate 510,000
Forward contract gain 20,000
Page 25

Number 137 Answer B

November 1, 2020 90-day forward rate ($2,000 x 41) 82,000


December 31, 2020 30-day forward rate ($2,000 x 46) 92,000
Commitment liability 10,000

Number 138 Answer A

Forward contract payable – December 31, 2020 ($2,000 x 30-day forward rate 92,000
46)
Forward contract payable – January 31, 2021 (2,000 x 44 spot rate January 31, 80,000
2021
Foreign currency gain 2021 4,000

Number 139 Answer A

Forward contract receivable – November 1, 2020 ($1,200 x 42 90-day forward 50,400


rate)
Forward contract receivable – December 31, 2020 ($1,200 x 44 30-day forward 52,800
rate)
Unrealized gain in OCI 2020 2,400

Number 140 Answer B

Forward contract receivable – December 31, 2020 ($1,200 x 44 30-day forward 52,800
rate)
Forward contract receivable – January 31, 2021 ($1,200 x 43 spot rate January 51,600
31, 2021
Unrealized loss in OCI 2021 1,200

Number 141 Answer A

Unrealized gain – 2020 2,400


Unrealized loss – 2021 (1,200)
Cumulative unrealized gain – December 31, 2021 1,200
Number 142 Answer D

Purchase price $1,200


Multiply by spot rate January 31, 2021 43
Cost of equipment 51,600

Number 143 Answer A

Option price 40
Market price November 1, 2020 (40)
Change in price 0
Multiply by $1,000
Intrinsic value of call option November 1, 2020 0

Option price 40
Market price December 31, 2020 ___44
Change in price 4
Multiply by $1,000
Intrinsic value of call option December 31, 2020 4,000
Intrinsic value of call option November 1, 2020 ___0
Unrealized gain in OCI 2020 4,000

Page 26

Number 144 Answer B

Fair value of call option – November 1, 2020 300


Intrinsic value – November 1, 2020 __0
Time value of call option November 1, 2020 300

Fair value of call option – December 31, 2020 4,500


Intrinsic value of call option – December 31, 2020 4,000
Time value of call option – December 31, 2020 500
Time value of call option – November 1, 2020 _300
Unrealized gain in profit or loss 2020 200

Number 145 Answer C

Option price 40
Market price January 31, 2021 ___43
Change in price 3
Multiply by $1,000
Intrinsic value of call option January 31, 2021 3,000
Intrinsic value of call option – December 31, 2020 _4,000
Unrealized loss in OCI 2021 1,000

END
FINANCIAL ACCOUNTING PROBLEMS

Problem I (Current assets)


An entity provided the following trial balance on June 30, 2015:
Cash overdraft ( 200,000) Property, plant and equipment, net 1,900,000
Accounts receivable, net 700,000 Accounts payable and accrued expenses 640,000
Inventory 1,200,000 Share capital 3,000,000
Prepaid expenses 200,000 Share premium 500,000
Land held for resale 2,000,000 Retained earnings 1,660,000
Checks amounting to P600,000 were written to vendors and recorded on June 30 resulting in
cash overdraft of P200,000. The checks were mailed on July 9. Land held for resale was sold for
cash on July 15. The financial statements were issued on July 31. On June 30, 2015, what total
amount should be reported as current assets?
a. 4,500,000
b. 4,100,000
c. 4,300,000
d. 2,500,000

Problem 2 (Total assets)


An entity was incorporated on January 1, 2015 with proceeds from the issuance of P7,500,000 in
shares and borrowed funds of P1,100,000. During the first year of operations, revenue from sales
and consulting amounted to P820,000, and operating costs and expenses totaled P640,000. On
December 15, the entity declared a P30,000 cash dividend, payable to shareholders on January
15, 2016. No additional activities affected owners’ equity in 2015. The liabilities increased to
P1,200,000 by December 31, 2015. What amount should be reported as total assets on December
31, 2015?
a. 8,850,000
b. 8,820,000
c. 7,870,000
d. 8,750,000

Problem 3 (Current liabilities)


An entity had the following liabilities on December 31, 2015:
Accounts payable 55,000
Unsecured notes, 8% due 7/1/2016 400,000
Accrued expenses 35,000
Contingent liability 450,000
Deferred tax liability 25,000
Senior bonds, 7%, due 3/31/2016 1,000,000
The contingent liability is an accrual for possible loss on a P1,000,000 lawsuit filed against the
entity. The legal counsel expects the suit to be settled in 2016 and has estimated that the entity
will be liable for damages in the range of P450,000 to P750,000. The deferred tax liability is
expected to reverse in 2016. What amount should be reported on December 31, 2015 for current
liabilities?
a. 515,000
b. 940,000
c. 1,490,000
d. 1,515,000

Page 2

Problem 4 (Net income)


An entity reported net income of P7,410,000 for the current year. The auditor raised questions
about the following amounts that had been included in net income:
Unrealized loss on equity investments at fair value through other comprehensive
income ( 540,000 )
Gain on early retirement of bonds payable 2,200,000
Adjustment of profit of prior year for error in depreciation, net of tax effect ( 750,000 )
Loss from fire ( 1,400,000 )
Gain from change in fair value attributable to the credit risk of financial liability
designated at fair value through profit or loss 500,000
What amount should be reported as adjusted net income?
a. 6,500,000
b. 7,200,000
c. 8,200,000
d. 8,700,000

Problem 5 (Retained earnings)


An entity provided the following information on December 31, 2015:

Total reported income since incorporation 1,700,000

Total cash dividends paid ( 800,000)

Unrealized holding loss on trading investment ( 120,000)

Total share dividends distributed ( 200,000)


Prior period adjustment recorded January 1, 2015 - credit 75,000

What amount should be reported as retained earnings on December 31, 2015?

a. 655,000
b. 700,000
c. 580,000
d. 775,000

Problem 6 (Cash computation)

An entity reported the checkbook balance on December 31, 2015 at P8,000,000. In addition, the
entity held the following items in the safe on that date:
Check payable to the entity, dated January 2, 2016 in payment of a sale,
not  included in December 31 check book balance 1,000,000
Check payable to the entity, deposited December 15 and included in
December 31 checkbook balance, but returned by bank on
      December 30 stamped “NSF”. The check was redeposited on
January 2, 2016 and cleared on January 5, 2016 3,000,000
Check drawn on the entity’s account, dated and recorded on
      December 31, 2015 but not mailed until January 15, 2016 2,500,000
Coins and currencies on hand 800,000
Three-month money market instruments 1,500,000
What is the correct amount of “cash” on December 31, 2015?
a. 7,500,000
b. 9,300,000
c. 8,300,000
d. 9,800,000

Page 3

Problem 7 (Impairment of accounts receivable)

An entity reported the following accounts receivable on December 31, 2015:


Customer A 1,000,000
Customer B 1,500,000
Customer C 2,000,000
Customer D 2,500,000
All other accounts receivable not individually significant 3,500,000
The entity determined that Customer A receivable is totally impaired and Customer B receivable
is impaired by P700,000. The other receivables from Customers C and D are not considered
impaired. The entity determined that a composite rate of 10% is appropriate to measure
impairment on the remaining accounts receivable. What is the total impairment loss of accounts
receivable for 2015?
a. 2,500,000
b. 2,050,000
c. 1,050,000
d. 2,750,000

Problem 8 (Current net receivables)

An entity reported current receivables on December 31, 2015 which consisted of the following:
Trade accounts receivable 930,000

Allowance for uncollectible accounts 20,000

Claim against shipper for goods lost in transit in November 2015 30,000

Selling price of unsold goods sent by the entity on consignment at 130% of     cost and
not included in the ending inventory
260,000

Security deposit on lease of warehouse used for storing inventories 300,000

What is the correct total of current net receivables on December 31, 2015?
a. 1,500,000
b. 1,200,000
c. 1,240,000
d. 940,000

Problem 9 (Measurement of notes receivables)

On December 31, 2015, an entity received two P2,000,000 notes receivable from customers. On
both notes, interest is calculated on the outstanding principal balance at the annual rate of 3%
and payable at maturity. The first note, made under customary trade terms, is due in nine months
and the second note is due in five years. The market interest rate for similar notes on December
31, 2015 was 8%. The PV of 1 at 8% due in nine months is .944, and the PV of 1 at 8% due in 5
years is .68. On December 31, 2015, what total carrying amount should be reported for the two
notes receivable?
a. 3,248,000

b. 3,494,400
c. 3,360,000

d. 3,564,000

Page 4

Problem 10 (Measurement of loan receivable)

A bank granted a 10-year loan to a borrower in the amount of P1,500,000 with stated interest rate
of 6%. Payments are due monthly and are computed to be P16,650. The bank incurred P40,000
of direct loan origination cost and P20,000 of indirect loan origination cost. In addition, the bank
charged the borrower a 4-point nonrefundable loan origination fee. What is the carrying amount
of the loan receivable to be reported initially by the bank?
a. 1,440,000
b. 1,480,000
c. 1,500,000
d. 1,520,000

Problem 11 (Cost of inventory)

An entity reported inventory on December 31, 2015 at P6,000,000 based on a physical count at
cost and before any necessary year-end adjustments relating to the following:
 Included in the physical count were goods billed to a customer FOB shipping point on
December 30, 2015. These goods had a cost of P125,000 and were picked up by the carrier
on January 7, 2016.
 Goods shipped FOB shipping point on December 28, 2015 from a vendor to the entity were
received on January 4, 2016. The invoice cost was P300,000.
What amount should be reported as inventory on December 31, 2015?
a. 5,875,000
b. 6,000,000
c. 6,175,000
d. 6,300,000

Problem 12 (Computation of accounts payable)

An entity reported accounts payable on December 31, 2015 at P4,500,000 before any necessary
year-end adjustments relating to the following transactions:
 On December 27, 2015, the entity wrote and recorded checks to creditors totaling P2,000,000
causing an overdraft of P500,000 in the entity’s bank account on December 31, 2015. The
checks were mailed on January 10, 2016.
 On December 28, 2015, the entity purchased and received goods for P750,000, terms 2/10,
n/30. The entity recorded purchases and accounts payable at net amount. The invoice was
recorded and paid January 3, 2016.
 Goods shipped FOB destination on December 20, 2015 from a vendor to the entity were
received January 2, 2016, The invoice cost was P325,000.
On December 31, 2015, what amount should be reported as accounts payable?
a. 7,575,000
b. 7,250,000
c. 7,235,000
d. 7,553,500

Page 5
Problem 13 (Retail inventory method)
On December 31, 2015, an entity provided the following information:
Cost Retail
Inventory, January 1 735,000 1,015,000
Purchases 4,165,000 5,775,000
Additional markup 210,000
Sales for the year totaled P5,500,000. Markdown amounted to P100,000. Under the
approximate lower of average cost or NRV retail method, what is the inventory on December
31, 2015?
a. 1,050,000
b. 1,400,000
c. 994,000
d. 980,000

Problem 14 (Gross profit method)


An entity budgeted the following sales.

June July August

Sales on account 1,800,000 1,840,000 1,900,000

Cash sales 180,000 200,000 260,000

All merchandise is marked up to sell at invoice cost plus 20%. Merchandise inventory at the beginning of
each month is 30% of that month's projected cost of goods sold. What is the amount of anticipated
purchases for July?

a. 1,632,000
b. 2,076,000
c. 1,700,000
d. 1,730,000

Problem 15 (Biological assets)


An entity provided the following information about assets in forest plantation:
Freestanding trees 5,000,000
Land under trees 900,000
Roads in forest 500,000
Animals related to recreational activities 2,000,000
Rubber trees and grape vines 1,500,000
What total amount should be reported as biological assets?
a. 5,000,000
b. 8,500,000
c. 6,500,000
d. 9,900,000

Problem 16 (Machinery)
On September 1, 2015, an entity purchased a new machine on a deferred payment basis. A down
payment of P200,000 was made and 4 annual installments of P600,000 each are to be made beginning on
September 1, 2016. The cash equivalent price of the machine was P2,300,000. Due to an employee strike,
the entity could not install the machine immediately and thus incurred P30,000 of storage cost. Cost of
installation excluding the storage cost amounted to P80,000. What is the total cost of the machine?

a. 2,300,000
b. 2,380,000
c. 2,410,000
d. 2,600,000

Page 6

Problem 17 (Borrowing cost)

During 2015, an entity constructed an asset costing P10,000,000. The weighted average accumulated
expenditures on the asset during the year totaled P6,000,000. To help pay for construction, P4,400,000
was borrowed at 10% on January 1, 2015, and funds not needed for construction were temporarily
invested in short-term securities, yielding P90,000 in interest revenue. Other than the construction
funds borrowed, the only other debt outstanding during the year was a P5,000,000, 10-year, 9% note
payable dated January 1, 2012. What is the amount of interest that should be capitalized during 2015?

a. 600,000
b. 300,000
c. 494,000
d. 944,000

Problem 18 (Depletion)

In 2015, an entity purchased property with natural resources for P28,000,000. The property had a
residual value of P5,000,000. However, the entity is required to restore the property to the
original condition at a discounted amount of P2,000,000. In 2015, the entity spent P1,000,000 in
development cost and P3,000,000 in building. In 2016, an amount of P4,000,000 was spent for
additional development on the mine. Production began in 2016 and the tons extracted totaled
3,000,000 in 2016 and 2,500,000 in 2017. The remaining tons totaled 7,000,000 and 3,500,000,
respectively on December 31, 2016 and December 31, 2017. What amount of depletion should
recognized in 2017?
a. 10,500,000
b. 12,250,000
c. 9,000,000
d. 8,750,000

Problem 19 (Revaluation)

On June 30, 2015, an entity reported the following information:


Equipment at cost 30,000,000
Accumulated depreciation 10,500,000
The equipment was measured using the cost model and depreciated on a straight line basis over
10-year period. On December 31, 2015, the management decided to change the basis of
measuring the equipment from the cost model to the revaluation model. The equipment was
revalued to the fair value of P27,000,000 with remaining useful life of 5 years. The income tax
rate is 30%. What amount should be reported as revaluation surplus on December 31, 2015?
a. 7,500,000
b. 5,250,000
c. 6,300,000
d. 9,000,000

Page 7

Problem 20 (Computer software)

During the current year, an entity incurred the following costs to develop and produce a routine, low-
risk computer software product:
Completion of detailed program design or working model 1,300,000

Cost incurred for coding and testing to establish technological feasibility 1,000,000

Other coding costs after establishment of technological feasibility 2,400,000

Other testing costs after establishment of technological feasibility 2,000,000

Costs of producing product masters for training materials 1,500,000

Duplication of computer software and training materials from product master 2,500,000

Packaging product 900,000

What amount should be capitalized initially as software cost?

a. 5,400,000
b. 3,700,000
c. 5,900,000
d. 6,900,000

Problem 21 (Start up costs)

An entity, a major winery, begins construction of a new facility in Mindanao. The following costs are
incurred in conjunction with the start-up activities of the new facility:

Production equipment 8,150,000

Travel costs of salaried employees 400,000

License fees 140,000

Training of local employees for production and maintenance operations 1,200,000

Advertising costs 850,000

What amount of start up costs should be expensed?

a. 9,750,000
b. 1,600,000
c. 1,390,000
d. 0

Problem 22 (Intangible assets)

On January 1, 2013, an entity purchased patent at a cost of P1,920,000 at which date the
remaining legal life was 16 years. On January 1, 2015, the useful life of the patent was
determined to be only 8 years from the date of acquisition. On January 1, 2015, the entity paid
P800,000, of which three-fourths was for a trademark, and one-fourth was for the other entity’s
agreement not to compete for a 5-year period in the line of business covered by the trademark.
The entity considered the life of the trademark indefinite. Moreover, the entity agreed to pay
P50,000 to the other entity as consulting fee each year for 5 years payable every January 1. What
is the amortization of intangible assets for 2015?
a. 320,000
b. 280,000
c. 250,000
d. 370,000

Page 8

Problem 23 (Goodwill)

On December 31, 2015, an entity purchased for P40,000,000 cash all of the outstanding ordinary shares
of another entity when the subsidiary’s statement of financial position showed net assets of
P32,000,000. The subsidiary’s assets and liabilities had fair value different from the carrying amount as
follows:

Carrying amount Fair value

Property, plant and equipment, net 50,000,000 57,500,000

Other assets 5,000,000 0

Long-term debt 30,000,000 28,000,000


What amount should be reported as goodwill in the December 31, 2015 consolidated statement of
financial position of the acquirer and its wholly-owned subsidiary?

a. 3,500,000
b. 2,500,000
c. 7,500,000
d. 8,000,000

Problem 24 (Financial asset at fair value through other comprehensive income)

On January 1, 2015, an entity purchased nontrading equity securities which are irrevocably designated
at fair value through other comprehensive income:

Purchase price Transaction cost Market – 12/31/2015

Security A 1,000,000 100,000 1,500,000

Security B 2,000,000 200,000 2,400,000

Security C 4,000,000 400,000 4,700,000

On July 1, 2016, the entity sold Security C for P5,200,000. What amount should be credited to retained
earnings as a result of the sale of the investment in 2016?

a. 800,000
b. 500,000
c. 300,000
d. 0

Problem 25 (Interest on loans)

An entity frequently borrowed from the bank in order to maintain sufficient operating cash. The
following loans were at a 12% interest rate with interest payable at maturity. The entity repaid each loan
on scheduled maturity date.

Date of loan Amount Maturity date Term of loan

November 1, 2014 500,000 October 31, 2015 1 year


February 1, 2015 1,500,000 July 31, 2015 6 months

May 1, 2015 800,000 January 31, 2016 9 months

The entity recorded interest expense when the loans are repaid. As a result, interest expense of
P150,000 was recorded in 2015. If no correction is made, by what amount would interest expense be
understated for 2015?

a. 54,000
b. 62,000
c. 64,000
d. 72,000

Page 9

Problem 26 (Compensated absences)

An entity has 35 employees who work 8-hour days and are paid hourly. On January 1, 2013, the entity
began a program of granting the employees 10 days of paid vacation each year. Vacation days earned in
2013 may first be taken on January 1, 2014.

Hourly Vacation Days Earned Vacation Days Used

Year Wages by Each Employee by Each Employee

2013 25.80 10 0

2014 27.00 10 8

2015 28.50 10 10

The entity has chosen to accrue the liability for compensated absences at the current rate of pay in
effect when the compensated time is earned. What is the accrued liability for compensated absences on
December 31, 2015?
a. 94,920
b. 90,720
c. 79,800
d. 95,760

Problem 27 (Finance lease - lessee)

An entity leased equipment for the entire nine-year useful life, agreeing to pay P1,000,000 at the start of
the lease term on January 1, 2015, and P1,000,000 annually on each January 1 for the next eight years.
The present value on January 1, 2015 of the nine lease payments over the lease term using the rate
implicit in the lease which the lessor knows to be 10% was P6,330,000. The January 1, 2015 present
value of the lease payments using the incremental borrowing rate of 12% was P5,970,000. The entity
made a timely second lease payment. What amount should be reported as finance lease liability on
December 31, 2016?

a. 5,330,000
b. 4,863,000
c. 4,970,000
d. 4,467,000

Problem 28 (Finance lease - lessor)

An entity leased equipment to an unrelated party on July 1, 2015 for an eight-year period
expiring June 30, 2023. Equal payments under the lease are P600,000 and are due on July 1 of
each year. The first payment was made on July 1, 2015. The implicit rate of interest
contemplated is 10%. The cash selling price of the equipment is P3,500,000 and the carrying
amount is P2,800,000. The lease is appropriately recorded as a sales type lease. What total
amount of income should be recorded for the year ended December 31, 2015?
a. 700,000
b. 525,000
c. 990,000
d. 845,000
Page 10

Problem 29 (Sale and leaseback)

On January 1, 2015, an entity sold a machine for P5,000,000. The fair value of the machine was
P6,500,000 on the date of sale. The machine had a carrying amount of P7,000,000 and remaining life of
15 years. The entity immediately leased back the machine for 5 years at an annual rental that was
determined to be sufficiently lower than the market rent. What total amount of loss should be
recognized immediately in 2015?

a. 400,000
b. 800,000
c. 500,000
d. 0

Problem 30 (Employee benefit – IFRS 19R)

An entity provided the following information for the current year:


Current service cost 500,000
Interest on projected benefit obligation 600,000
Interest income on plan assets 350,000
Loss on plan settlement 250,000
Past service cost during the year 300,000
Actual return on plan assets 850,000
Actuarial loss during the year 200,000
Contribution to the plan 1,500,000
What is the employee benefit expense for the current year?
a. 1,300,000
b. 1,050,000
c. 1,500,000
d. 1,100,000

Problem 31 (Actual return on plan assets)

An entity provided the following data related to the pension plan.

December 31, 2015 December 31, 2016


Defined benefit obligation 8,400,000    11,100,000

Plan assets at fair value 9,000,000                9,900,000

Net actuarial loss 1,440,000                   1,500,000

Discount rate 10%                          9%

Expected rate of return 8%                          7%

The contribution was P1,260,000 in 2016 and benefits paid totaled P1,125,000. What was the actual
return on plan assets in 2016?

a. 900,000
b. 765,000
c. 600,000
d. 465,000

Page 11

Problem 32 (Vacation pay expense)

An entity grants all employees two weeks of paid vacation for each full year of employment.
Unused vacation time can be accumulated and carried forward to succeeding years and will be
paid at the salaries in effect when vacations are taken or when employment is terminated. There
was no employee turnover in 2015. Additional information relating to the year ended December
31, 2015 is as follows:

Liability for accumulated vacations on January 1, 2015 350,000


Pre-2015 accrued vacations taken from January 1, 2015 to September 30,  2015
     (the authorized  period for vacations) 200,000
Vacations earned for work in 2015 adjusted to current rate 300,000
The entity granted a 10% salary increase to all employees on October 1, 2015, the annual salary
increase date. What amount should be reported as vacation pay expense for 2015?
a. 450,000
b. 335,000
c. 315,000
d. 300,000

Problem 33 (Termination benefits IFRS 19R)

An entity is committed to close a factory in 10 months and shall terminate the employment of all
the remaining employees of the factory. Under the termination plan, an employee leaving before
closure of factory shall receive on termination date a cash payment of P20,000. However, an
employee that renders service until closure of the factory shall receive P60,000. There are 120
employees at the factory. The entity expects 20 employees to leave before closure and 100
employees to render service until closure. What amount should be recognized as termination
benefit?
a. 2,400,000
b. 6,400,000
c. 2,000,000
d. 4,000,000

Problem 34 (Income tax)

An entity reported P9,000,000 income before provision for income tax. The following data are
provided for the current year:

Rent received in advance 1,600,000


Income from exempt municipal bonds 2,000,000
Depreciation deduction for income tax purposes in excess of depreciation
reported for financial reporting purposes 1,000,000
Tax payment during the current year 500,000
Income tax rate 30%
What amount of current income tax liability should be reported at year-end?
a. 1,780,000
b. 2,280,000
c. 2,580,000
d. 2,880,000
Page 12

Problem 35 (Bonds payable)

An entity has outstanding a 7%, ten-year P100,000 facevalue bond. The bonds was originally
sold to yield 6% annual interest. The entity uses the effective interest method to amortize bond
premium and does not elect the fair value option for reporting financial liabilities. On June 30,
2015, the carrying amount of the outstanding bond was P105,000. What amount of unamortized
premium on bond should be reported on June 30, 2016?

a. 1,050
b. 3,950
c. 4,300
d. 4,500

Problem 36 (Share-based compensation)

On January 1, 2015, an entity granted to employees 10,000 share options. On January 1, 2016,
the entity granted to employees an additional 20,000 share options.
Date Fair value of share
January 1, 2015 20
December 31, 2015 22
January 1, 2016 25
December 31, 2016 30

The shares vest at the end of a four-year period. There are no forfeitures. What amount should be
recorded as compensation expense for 2016?
a. 175,000
b. 205,000
c. 225,000
d. 500,000

Problem 37 (Retained earnings)

An entity provided the following data for the year ended December 31, 2015:

Retained earnings unappropriated, January 1 200,000

Overdepreciation of 2014 due to prior period error 100,000

Net income for 2015 1,300,000

R Retained earnings appropriated for treasury shares (original balance is P500,000

but reduced by P200,000 by reason of reissuance of the treasury shares) 300,000

Retained earnings appropriated for contingencies (beginning balance P700,000.

but increased by current appropriation of P100,000) 800,000

Cash dividends paid to shareholders 500,000

Change in accounting policy from FIFO to average – credit adjustment 150,000

What is the balance of unappropriated retained earnings on December 31, 2015?

a. 1,150,000
b. 1,350,000
c. 1,950,000
d. 1,750,000
Page 13

Problem 38 (Diluted earnings per share)

An entity reported the following capital structure:

2015 2016
Outstanding shares:
Ordinary shares 110,000 110,000
Convertible preference shares 10,000 10,000
During 2015, the entity paid preference dividends of P3 per share. The preference shares are
convertible into 20,000 ordinary shares. Net income for 2015 was P850,000. The income tax rate
is 30%. What amount should be reported as diluted earnings per share for 2015?
a. 6.31
b. 6.54
c. 7.08
d. 7.45

Problem 39 (Cash basis)

An entity had the following beginning and ending balances in prepaid expenses and accrued
liabilities for the current year:

Prepaid expenses Accrued liabilities


Beginning balance 5,000 8,000
Ending balance 10,000 20,000

Debits to operating expenses totaled P100,000. What amount was paid for operating expenses
during the current year?
a. 83,000
b. 93,000
c. 107,000
d. 117,000

Problem 40 (Current cost)

An entity disclosed supplemental information on the effects of changing prices. The entity computed
the increase in current cost of inventory as follows:

Increase in current cost (nominal peso) 1,500,000


Increase in current cost (constant peso) 1,200,000
What amount should be disclosed as the inflation component of the increase in current cost?

a. 2,700,000
b. 1,500,000
c. 1,200,000
d. 300,000

Page 14

Problem 41 (Accrual basis)

An entity acquired rights to a patent under a licensing agreement that required an advance royalty
payment when the agreement was signed. The entity remitted royalties earned and due under the
agreement on October 31 each year. Additionally, on the same date, the entity paid, in advance,
estimated royalties for the next year. The entity adjusted prepaid royalties at year-end. The entity
provided the following information for the year ended December 31, 2015:

January 1 Prepaid royalties 650,000

October 31 Royalty payment charged to royalty expense 1,100,000

December 31 Year-end credit adjustment to expense 250,000

On December 31, 2015, what amount should be reported as prepaid royalties?

a. 250,000
b. 400,000
c. 850,000
d. 900,000

Problem 42 (Accrual basis)

An entity had a balance of P820,000 in the professional fees expense account on December 31, 2015,
before considering year-end adjustments relating to the following:

 Consultants were hired for a special project at a total fee not to exceed P650,000. The entity had
recorded P550,000 of this fee based on billings for work performed in 2015.

 The attorney’s letter requested by the auditors dated January 31, 2016, indicated that legal fees of
P60,000 were billed on January 15, 2016 for work performed in November 2015, and unbilled fees
for December 2015 were P70,000.

What amount should be reported for professional fees expense for 2015?
a. 1,050,000
b. 950,000
c. 880,000
d. 820,000

Problem 43 (Disclosures)

An entity reported the following information in the year-end financial statements:


Capital expenditures 1,000,000
Finance lease payments 125,000
Income taxes paid 325,000
Dividends paid 200,000
Interest payments 220,000
What total amount should be reported as supplemental disclosures in the statement of cash flows
prepared using the indirect method?
a. 545,000
b. 745,000
c. 1,125,000
d. 1,870,000
Page 15

Problem 44 (Operating activities)

An entity reported net income of P3,000,000 for the current year. Changes occurred in certain
accounts as follows:
Equipment 250,000 increase
Accumulated depreciation 400,000 increase
Note payable 300,000 increase
During the year, the entity sold equipment costing P250,000 with accumulated depreciation of
P150,000 for a gain of P50,000. In December of the current year, the entity purchased equipment
costing P500,000 with  P200,000 cash and a 12% note payable of P300,000. What amount
should be reported as net cash provided by operating activities?
a. 3,400,000
b. 3,500,000
c. 3,550,000
d. 3,600,000

Problem 45 (Operating activities)

An entity reported net income of P5,000,000 for the current year. Depreciation expense was
P1,900,000. The following working capital accounts changed:

Accounts receivable 1,100,000 increase

Nontrading equity investment 1,600,000 increase

Inventory 730,000 increase

Nontrade note payable 1,500,000 increase

Accounts payable 1,220,000 increase

Under the indirect method, what net amount of adjustments is required to reconcile net income to net
cash provided by operating activities?
a. 4,950,000
b. 1,050,000
c. 1,290,000
d. 310,000

Page 16

SOLUTIONS
Problem 1 Answer A
Cash (600,000 -200,000 overdraft) 400,000
Accounts receivable 700,000
Inventory 1,200,000
Prepaid expenses 200,000
Land held for resale 2,000,000
Total current assets 4,500,000

Problem 2 Answer A
Liabilities 1,200,000
Share capital 7,500,000
Retained earnings 150,000
Total liabilities and equity 8,850,000
Revenue from sales and consulting 820,000
Operating costs and expenses ( 640,000)
Net income 180,000
Dividend declared ( 30,000)
Retained earnings 150,000

Problem 3 Answer C
Accounts payable 55,000
Unsecured notes 400,000
Accrued expenses 35,000
Serial bonds 1,000,000
Total current liabilities 1,490,000
The contingent liability is only disclosed.
Under IFRS, the deferred tax liability is noncurrent regardless of the reversal period.

Problem 4 Answer C
Net income per book 7,410,000
Unrealized loss- other comprehensive income erroneously deducted 540,000
Prior period error erroneously deducted 750,000
Gain on credit risk – other comprehensive income erroneously added ( 500,000)
Adjusted net income 8,200,000

The gain on early retirement of bonds payable and the loss from fire are properly included in net
income.

Problem 5 Answer D
Total reported income 1,700,000
Total cash dividends paid ( 800,000)
Total share dividends distributed ( 200,000)
Prior period adjustment – credit 75,000
Retained earnings – December 31, 2015 775,000
The unrealized holding loss on trading investment is ignored because it is already included in the
reported income since incorporation.

Page 17

Problem 6 Answer C
Checkbook balance 8,000,000
NSF check (3,000,000)
Undelivered check drawn 2,500,000
Coins and currencies 800,000
Total cash 8,300,000

The check payable to the entity is properly not included because it is postdated January 2, 2016.
Technically, the three-month money market instruments are cash equivalents but not cash.

Problem 7 Answer A
Customer A 1,000,000
Customer B 700,000
Total other receivables 800,000
Total impairment loss 2,500,000
Customer C 2,000,000
Customer D 2,500,000
Other accounts receivable 3,500,000
Total other receivables for collective assessment of impairment 8,000,000
Under IFRS significant accounts receivable not impaired should be combined with other
accounts receivable not individually significant for collective assessment of impairment.

Problem 8 Answer D
Trade accounts receivable 930,000
Allowance for uncollectible accounts ( 20,000)
Claim against shipper 30,000
Total current net receivables 940,000

The selling price of unsold goods on consignment should be excluded from accounts receivable
but the cost should be included in inventory.
The security deposit is classified as noncurrent.

Problem 9 Answer D
Long-term note receivable – second note 2,000,000
Interest on note (2,000,000 x 3% x 5 years) 300,000
Total maturity 2,300,000
Multiply by PV factor .68
Present value of note receivable 1,564,000
Short-term note receivable – first note 2,000,000
Total carrying amount of notes receivable 3,564,000
The long-term note receivable should be discounted even if is interest-bearing because the
interest rate is unreasonably low compared to the market rate.
The short-term note receivable is reported at face amount because the discount is usually not
material.

Page 18

Problem 10 Answer B
Face amount 1,500,000
Direct origination cost 40,000
Origination fee charged against borrower (4% x 1,500,000) ( 60,000)
Initial carrying amount 1,480,000
The direct origination cost is a deferred charge and the origination fee received from the
borrower is unearned income and the two should be included in the measurement of loan
receivable.
The indirect origination cost is an outright expense.

Problem 11 Answer D
Physical count 6,000,000
Good in transit purchased FOB shipping point 300,000
Total inventory 6,300,000

The goods billed to a customer are properly included in inventory because the term is FOB
shipping point and the goods are delivered January 7, 2016.

Problem 12 Answer C
Accounts payable per book 4,500,000
Reversal of undelivered checks 2,000,000
Goods purchased, received and recognized at net amount (750,000 x 98%) 735,000
Accounts payable to be reported 7,235,000
The undelivered checks should be restored to the cash balance and accounts payable.
The goods purchased and received on January 2, 2016 should be excluded from accounts payable
because the term is FOB destination.

Problem 13 Answer D
Cost Retail
Inventory – January 1 735,000 1,015,000
Purchases 4,165,00 5,775,000
0
Additional markup _______ 210,000
_
Goods available for sale 4,900,00 7,000,000
0
Conservative cost ratio (4,900,000 / 7,000,000) 70%
Sales (5,500,000)
Markdown ( 100,000)
Ending inventory at retail 1,400,000
At cost (70% x 1,400,000) 980,000
The lower of average cost or NRV retail method is the same as the conservative or conventional
method. Thus, the markdown is ignored in computing the cost ratio.
Page 19

Problem 14 Answer D
Cost of goods sold:
June (1,980,000 / 120%) 1,650,000
July (2, 040,000 / 120%) 1,700,000
August (2,160,000 / 120%) 1,800,000
Inventory – July 1 (30% x 1,700,000) 510,000
Purchases (SQUEEZE) 1,730,000
Goods available for sale 2,240,000
Inventory – July 31 (30% x 1,800,000) ( 540,000)
Cost of goods sold - July 1,700,000
The amount of purchases for July is computed by working back from the cost of goods sold.

Problem 15 Answer A
Freestanding trees 5,000,000
The land under trees and roads in forest should be included in property, plant and equipment.
Under IFRS, animals related to recreational activities as in game parks, and bearer plants, such
as rubber trees and grape vines should be accounted for as property, plant and equipment.

Problem 16 Answer B
Cash equivalent price 2,300,000
Installation cost 80,000
Total cost of machine 2,380,000

The storage cost is an outright expense.


Problem 17 Answer C
Average expenditures 6,000,000
Specific borrowing (4,400,000)
General borrowing 1,600,000

Interest on specific borrowing (4,400,000 x 10%) 440,000


Interest income on temporary investment of specific borrowing ( 90,000)
Interest on general borrowing (1,600,000 x 9%) 144,000
Total capitalized interest 494,000

Page 20

Problem 18 Answer D
Purchase price 28,000,000
Development cost – 2015 1,000,000
Development cost – 2016 4,000,000
Estimated restoration cost 2,000,000
Total cost 35,000,000
Residual value ( 5,000,000)
Depletable amount 30,000,000

Production in 2016 3,000,000


Remaining estimate – December 31, 2016 7,000,000
Total estimate – January 1, 2016 10,000,000

Rate per unit (30,000,000 / 10,000,000) 3.00


Depletion for 2016 (3,000,000 x 3) 9,000,000

Production in 2017 2,500,000


Remaining estimate – December 31, 2017 3,500,000
Total estimate – January 1, 2017 6,000,000

Depletable amount 30,000,000


Depletion 2016 ( 9,000,000)
Remaining depletable amount 21,000,000

New rate (21,000,000 / 6,000,000) 3.50

Depletion 2017 (2,500,000 x 3.50) 8,750,000

Problem 19 Answer C
Accumulated depreciation – 6/30/2015 10,500,000
Depreciation from July 1 to December 31, 2015 (30,000,000 / 10 x 6/12) 1,500,000
Accumulated depreciation – 12/31/2015 12,000,000

Cost 30,000,000
Accumulated depreciation ( 12,000,000)
Carrying amount 18,000,000
Fair value 27,000,000
Revaluation surplus 9,000,000
Deferred tax liability (30% x 9,000,000) ( 2,700,000)
Net revaluation surplus 6,300,000

Problem 20 Answer C
Other coding cost after establishment of technological feasibility 2,400,000
Other testing costs after establishment of technological feasibility 2,000,000
Costs of producing product masters 1,500,000
Total capitalized cost of computer software 5,900,000

The completion of detailed program design and the cost incurred to establish technological
feasibility should be expensed immediately.

The duplication of computer software and packaging product should be charged to inventory.
Page 21
Problem 21 Answer B
Travel costs of employees 400,000
Training of local employees 1,200,000
Total start up costs to be expensed 1,600,000
The production equipment should be capitalized.
The license fees and advertising costs should be expensed but not within the purview of start up
costs.

Problem 22 Answer A
Patent - January 1, 2013 1,920,000
Amortization for 2013 and 2014 (1,920,000 / 16 x 2)   ( 240,000)
Carrying amount – January 1, 2015 1,680,000
Purchase price 800,000
Trademark (3/4 x 800,000) ( 600,000)
Noncompetition agreement 200,000
Patent (1,680,000 / 6 years remaining) 280,000
Noncompetition agreement (200,000 / 5 years) 40,000
Total amortization for 2015 320,000
The patent has a remaining life of 6 years because the revised life is 8 years from the date of
acquisition and two years already expired.
The trademark is not amortized because the life is indefinite.
The annual consulting fee is an outright expense.

Problem 23 Answer A
Net assets per book 32,000,000
Fair value of property, plant and equipment greater 7,500,000
Fair value of other assets zero ( 5,000,000)
Fair value of long-term debt lower 2,000,000
Net assets at fair value 36,500,000
Acquisition cost 40,000,000
Goodwill 3,500,000
The net assets should be recognized at fair value in a business combination.

Problem 24 Answer A
Purchase price of security C 4,000,000
Transaction cost 400,000
Total cost 4,400,000
If the equity investment is measured at fair value through other comprehensive income (FVOCI),
the transaction cost is capitalized
Market value of security C 12/31/2015 4,700,000
Historical cost 4,400,000
Unrealized gain – OCI 12/31/20015 300,000

Journal entry on July 1, 2016


Cash 5,200,000
Unrealized gain – OCI 300,000
Financial asset – FVOCI 4,700,000
Retained earnings 800,000

Under the final version of IFRS 9, any change in fair value of an equity investment measured at
FVOCI is permanently excluded from profit or loss under all circumstances but may transferred
to equity or retained earnings.

Page 22

Problem 25 Answer A

January 1, 2015 to October 31, 2015 (500,000 x 12% x 10/12) 50,000


February 1, 2015 to July 31, 2015 (1,500,000 x 12% x 6/12) 90,000
May 1, 2015 to December 31, 2015 (800,000 x 12% x 8/12) 64,000
Correct interest expense 204,000
Recorded interest expense 150,000
Interest expense understated 54,000

Problem 26 Answer A

Total vacation days – 2013, 2014 and 2015 30


Total vacation days used (8 + 10) 18
Unused vacation days 12

From 2014 2
From 2015 10
Total unused vacation days - FIFO 12

2014 (35 employees x 8 hours x 2 x P27) 15,120


2015 (35 x 8 x 10 x P28.50) 79,800
Accrued liability – 12/31/2015 94,920
Problem 27 Answer B

Date Payment 10% interest Principal Present value


1/1/2015 6,330,000
1/1/2015 1,000,000 - 1,000,000 5,330,000
1/1/2016 1,000,000 533,000 467,000 4,863,000
The relevant present value is the amount computed using the 10% implicit rate.
The first payment on January 1, 2015 is applied all to principal

Problem 28 Answer D

Present value – 7/1/2015 (cash price) 3,500,000


Payment on 7/1/2015 – all applicable to principal ( 600,000)
Present value – 7/1/2015 2,900,000

Interest income from July 1, 2015 to June 30, 2016 (10% x 2,900,000) 290,000

Cash price 3,500,000


Carrying amount 2,800,000
Gain on sale 700,000
Interest income 7/1/2015 to 12/31/2015 (290,000 x 6/12) 145,000
Total income 845,000

Page 23

Problem 29 Answer B

Fair value of machine 6,500,000


Carrying amount 7,000,000
Impairment loss ( 500,000)

Sale price 5,000,000


Fair value 6,500,000
Deferred loss ( 1,500,000)

Impairment loss 500,000


Amortization of deferred loss (1,500,000 / 5 years) 300,000
Total loss to be recognized in 2015 800,000

If the leaseback is an operating lease and the sale price is below fair value of the asset
compensated by below market rent:

a. The difference between the sale price and the fair value is a deferred loss to be amortized
over the lease term.
b. If the fair value is below the carrying amount, the carrying amount is written down to fair
value and the writedown is recognized immediately as an impairment loss.

Problem 30 Answer A

Current service cost 500,000


Interest on projected benefit obligation 600,000
Interest income on plan assets ( 350,000)
Loss on plan settlement 250,000
Past service cost during the year 300,000
Total employee benefit expense 1,300,000

Problem 31 Answer B

Plan assets at fair value – 12/21/2015 9,000,000


Contribution to plan 2016 1,260,000
Actual return on plan assets (SQUEEZE) 765,000
Total 11,025,000
Benefits paid in 2016 ( 1,125,000)
Plan assets at fair value – 12/31/2016 9,900,000

The actual return or plan assets is “squeezed” by working back from ending plan assets at fair
value.

Problem 32 Answer C

Accumulated vacations – 1/1/2015 350,000


Vacation taken in 2015 200,000
Liability balance – 1/1/2015 150,000

Vacations earned in 2015 300,000


Adjustment of accumulated vacations – 1/1/2015 (10% x 150,000) 15,000
Total vacation pay expense 315,000

Page 24

Problem 33 Answer A

Termination benefit (120 employee x P20,000) 2,400,000

Total payment until closure 60,000


Termination benefit ( 20,000)
Additional benefit considered as short-term benefit 40,000

Short-term benefit (100 employees x 40,000) 4,000,000

Under IFRS, the additional amount paid to employees who render service until closure is no
longer a termination benefit but short-term benefit.

Problem 34 Answer A

Financial income 9,000,000


Rent received in advance 1,600,000
Tax exempt income ( 2,000,000)
Tax depreciation in excess of financial depreciation (1,000,000)
Taxable income 7,600,000

Current tax expense (30% x 7,600,000) 2,280,000


Tax payment during the year ( 500,000)
Current tax liability 1,780,000

Problem 35 Answer C

Interest paid (7% x 100,000) 7,000


Interest expense (6% x 105,000) 6,300
Premium amortization 700

Carrying amount – 6/30/2015 105,000


Face amount 100,000
Premium on bonds payable – 6/30/2015 5,000
Amortization 7/1/2015 to 6/30/2016 ( 700)
Unamortized premium – 6/30/2016 4,300

Problem 36 Answer A

Share options on January 1, 2015 (10,000 x 20) 200,000


Share options on January 1, 2016 (20,000 x 25) 500,000

The share options are measured at fair value on the date of grant and allocated over the vesting
period.

Share options on January 1, 2015 (200,000 /4 years) 50,000


Share options on January 1, 2016 (500,000 / 4 years) 125,000
Total compensation expense for 2016 175,000

Page 25

Problem 37 Answer B

Retained earnings – January 1 200,000


Prior period error – overdepreciation 100,000
Net income 1,300,000
Retained earnings appropriated for treasury shares reverted to unappropriated balance 200,000
Increase in retained earnings appropriated for contengencies ( 100,000)
Cash dividends paid ( 500,000)
Change in accounting policy - credit 150,000
Retained earnings unappropriated - December 31 1,350,000

Problem 38 Answer B

Ordinary shares outstanding 110,000


Potential ordinary shares from convertible preference shares 20,000
Total ordinary shares 130,000
Diluted EPS (850,000 net income / 130,000) 6.54

Problem 39 Answer B

Operating expenses 100,000


Beginning prepaid expenses ( 5,000)
Ending prepaid expenses 10,000
Beginning accrued liabilities 8,000
Ending accrued liabilities ( 20,000)
Operating expenses paid 93,000

Problem 40 Answer D

Increase in current cost – nominal 1,500,000


Increase in current cost – constant 1,200,000
Increase in current cost due to inflation 300,000

Problem 41 Answer D

Prepaid royalties – January 1 650,000


Increase in prepaid royalties credited to expense 250,000
Prepaid royalties – December 31 900,000

Page 26
Problem 42 Answer B

Professional fees expense per book 820,000


Accrued legal fees – November 60,000
Accrued legal fees – December 70,000
Adjusted professional fees expense 950,000
The entity already recorded P550,000 out of total consultants’ fee of P650,000. The balance of
P100,000 is not recognized because no work has been performed as yet.

Problem 43 Answer A

IFRS requires the following disclosures when preparing the statement of cash flows:

Income taxes paid 325,000


Interest payments 220,000
Total 545,000

Problem 44 Answer B

Increase in accumulated depreciation 400,000


Add : Accumulated depreciation of equipment sold 150,000
Depreciation for the year 550,000

Net income 3,000,000


Depreciation for the year 550,000
Gain on sale of equipment ( 50,000)
Net cash flows - operating 3,500,000

Problem 45 Answer C

Depreciation 1,900,000
Increase in accounts receivable (1,100,000)
Increase in inventory ( 730,000)
Increase in accounts payable 1,220,000
Net adjustment to net income as an addition 1,290,000
The increase in nontrading equity investment is an investing activity.
The increase in nontrade note payable is a financing activity.
Page 27

SITUATION PROBLEM I – BANK RECONCILIATION

An entity had the following bank reconciliation on June 30, 2015:


Balance per bank statement, June 30 3,000,000
Deposit transit 400,000
Total 3,400,000
Outstanding checks ( 900,000)
Balance per book, June 30 2,500,000
The bank statement for the month of July showed the following:
Deposits (including P200,000 note collected for the depositor entity) 9,000,000
Disbursements (including P140,000 NSF check and P10,000 service charge) 7,000,000
All reconciling items on June 30 cleared through the bank in July. The deposit in transit
amounted to P1,000,000 and the outstanding checks totaled P600,000 on July 31.
1. What is the amount of cash in bank that should be reported on July 31, 2015?
a. 5,000,000
b. 5,400,000
c. 4,550,000
d. 4,900,000

2. What is the cash balance per ledger on July 31, 2015?


a. 5,350,000
b. 5,550,000
c. 4,500,000
d. 5,400,000

3. What is the amount of cash receipts for book for the month of July?
a. 9,800,000
b. 8,600,000
c. 9,400,000
d. 9,600,000

4. What is the amount of cash disbursements per book for the month of July?
a. 7,300,000
b. 6,700,000
c. 6,850,000
d. 6,550,000

Page 28

SOLUTION – SITUATION PROBLEM I

Question 1 Answer B

Balance per bank – June 30 3,000,000


July bank deposits 9,000,000
July bank disbursements ( 7,000,000)
Balance per bank – July 31 5,000,000
July deposits in transit 1,000,000
July outstanding checks ( 600,000)
Adjusted bank balance 5,400,000
Question 2 Answer A

Balance per ledger – July 31 (SQUEEZE) 5,350,000


Note collected by bank in July 200,000
NSF check in July ( 140,000)
Service charge in July ( 10,000)
Adjusted book balance 5,400,000

The balance per book on July 31 is “squeezed” by working back from the adjusted balance.

Question 3 Answer C

Deposits per bank statement for July 9,000,000


Note collected by bank in July ( 200,000)
Deposit in transit – June 30 ( 400,000)
Deposit in transit – July 31 1,000,000
Cash receipts per book for July 9,400,000

Question 4 Answer D

Disbursements per bank statement for July 7,000,000


NSF check in July ( 140,000)
Service charge in July ( 10,000)
Outstanding checks – June 30 ( 900,000)
Outstanding checks – July 31 600,000
Cash disbursements per book for July 6,550,000
Page 29

SITUATION PROBLEM 2 – ACCOUNTS RECEIVABLE

From inception of operations, an entity provided for uncollectible accounts expense under the
allowance method and provisions were made monthly at 2% of credit sales. No year-end adjustments to
the allowance account were made. The balance in the allowance for doubtful accounts was P1,000,000
on January 1, 2015. During 2015, credit sales totaled P20,000,000, interim provisions for doubtful
accounts were made at 2% of credit sales, P200,000 of bad debts were written off, and recoveries of
accounts previously written off amounted to P50,000. An aging of accounts receivable was made for the
first time on December 31, 2015 as follows:

Classification Balance Uncolletible

November – December 6,000,000 10%

July – October 2,000,000 20%

January – June 1,500,000 30%

Prior to January 1, 2015 500,000 50%

Based on the review of collectibility of the account balances in the “prior to January 1 2015” aging
category, additional accounts totaling P100,000 are to be written off on December 31, 2015. Effective
December 31, 2015, the entity adopted the aging method for estimating the allowance for doubtful
accounts.

1. What is the required allowance for doubtful accounts on December 31, 2015?
a. 1,650,000
b. 1,950,000
c. 1,700,000
d. 1,450,000

2. What amount should be reported as doubtful accounts expense in the income statement for
2015?
a. 1,200,000
b. 1,650,000
c. 900,000
d. 950,000

3. What is the year-end adjustment to the allowance for doubtful accounts on December 31,
2015?
a. 900,000 debit
b. 900,000 credit
c. 500,000 debit
d. 500,000 credit

4. What is the net realizable value of accounts receivable on December 31, 2015?
a. 9,900,000
b. 8,250,000
c. 8,350,000
d. 8,200,000

Page 30

SOLUTION – SITUATION PROBLEM 2

Question 1 Answer A

6,000,000 x 10% 600,000


2,000,000 x 20% 400,000
1,500,000 x 30% 450,000
500,000 – 100,000 x 50% 200,000
Required allowance – December 31, 2015 1,650,000

Question 2 Answer C
Allowance for doubtful accounts – January 1 1,000,000
Recoveries of accounts written off 50,000
Doubtful accounts expense (SQUEEZE) 900,000
Total 1,950,000
Accounts written off (200,000 + 100,000) ( 300,000)
Allowance for doubtful accounts – December 31 1,650,000

The doubtful accounts expense is squeezed by working back from the ending allowance for
doubtful accounts.

Question 3 Answer D

Correct doubtful accounts expense 900,000


Recorded doubtful accounts expense (2%) x 20,000,000 sales) 400,000
Increase in allowance - credit 500,000

Question 4 Answer B

November – December 6,000,000


July – October 2,000,000
January – June 1,500,000
Prior January 1, 2015 (500,000 – 100,000) 400,000
Accounts receivable – December 31, 2015 9,900,000
Allowance for doubtful accounts ( 1,650,000)
Net realizable value 8,250,000
Page 31

SITUATION PROBLEM 3 – GROSS PROFIT METHOD

On December 31, 2015, a fire damaged the warehouse and factory of an entity completely destroying
the goods in process inventory. There was no damage to the raw materials, finished goods and factory
supplies The physical inventory revealed the following.

January 1 December 31

Raw materials 1,700,000 2,000,000

Goods in process 4,300,000 0

Finished goods 6,000.000 4,500,000

Factory supplies 500,000 400,000

The gross profit margin historically approximated 30% of sales. The sales for the year amounted to
P20,000,000. Raw material purchases totaled P4,000,000. Direct labor costs for the year amounted to
P5,000,000, and manufacturing overhead has been applied at 60% of direct labor.

1. What is the cost of raw materials used?


a. 5,700,000
b. 3,700,000
c. 3,800,000
d. 3,600,000

2. What is the total manufacturing cost?


a. 13,000,000
b. 11,800,000
c. 11,700,000
d. 11,600,000

3. What is the cost of goods sold?


a. 12,000,000
b. 16,000,000
c. 13,000,000
d. 14,000,000
4. What is the cost of goods in process inventory destroyed by fire?
a. 3,500,000
b. 3,800,000
c. 2,500,000
d. 1,500,000

Page 32

SOLUTION – SITUATION PROBLEM 3

Question 1 Answer B

Raw materials – January 1 1,700,000


Purchases 4,000,000
Raw materials available for use 5,700,000
Raw materials – December 31 ( 2,000,000)
Raw materials used 3,700,000

Question 2 Answer C
Raw materials used 3,700,000
Direct labor 5,000,000
Manufacturing overhead (60% x 5,000,000) 3,000,000
Total manufacturing cost 11,700,000

The change in the factory supplies is no longer considered because it is already part of the
manufacturing overhead applied.

Question 3 Answer D

Cost of goods sold (70% x 20,000,000) 14,000,000

The cost ratio is 70% because the gross profit rate is 30% on sales.

Question 4 Answer A

Total manufacturing cost 11,700,000


Goods in process – January 1 4,300,000
Total goods in process 16,000,000
Goods in process – December 31 (SQUEEZE) ( 3,500,000)
Cost of goods manufactured 12,500,000
Finished goods – January 1 6,000,000
Goods available for sale 18,500,000
Finished goods – December 31 ( 4,500,000)
Cost of goods sold 14,000,000

The cost of ending goods in process is computed by working back from the cost of goods sold.
Page 33

SITUATION PROBLEM 4 – INVESTMENT IN ASSOCIATE

On January 1, 2015, an entity acquired a 10% interest in an investee for P3,000,000. The
investment was accounted for under the cost method. During 2015, the investee reported net
income of P4,000,000 and paid dividend of P1,000,000. On January 1, 2016, the entity acquired
a further 15% interest in the investee for P8,500,000. On such date, the carrying amount of the
net assets of the investee was P36,000,000 and the fair value of the 10% existing interest was
P3,500,000. The fair value of the net assets of the investee is equal to carrying amount except for
an equipment whose fair value was P4,000,000 greater than carrying amount. The equipment had
a remaining life of 5 years. The investee reported net income of P8,000,000 for 2016 and paid
dividend of P5,000,000 on December 31, 2016.

1. What amount of investment income should be recognized in 2015?


a. 400,000
b. 100,000
c. 500,000
d. 300,000

2. What is the goodwill arising from the acquisition on January 1, 2016?


a. 3,000,000
b. 2,000,000
c. 2,500,000
d. 0

3. What total amount of income should be recognized by the investor in 2016?


a. 2,000,000
b. 2,500,000
c. 2,300,000
d. 1,800,000

4. What is the carrying amount of the investment in associate on December 31, 2015?
a. 12,550,000
b. 12,350,000
c. 11,950,000
d. 12,750,000
Page 34

SOLUTION – SITUATION PROBLEM 4

Question 1 Answer B

Dividend income (10% x 1,000,000) 100,000

Under cost method, the investment income is based on dividend declared or paid.

Question 2 Answer B

Existing 10% interest remeasured at fair value 3,500,000


New 15% interest 8,500,000
Total cost – January 1, 2016 12,000,000
Net assets acquired (25% x 36,000,000) ( 9,000,000)
Excess of cost over carrying amount 3,000,000
Excess attributable to equipment whose fair value is greater than carrying amount
(25% x 4,000,000) ( 1,000,000)
Goodwill 2,000,000
Question 3 Answer C

Share in net income (25% x 8,000,000) 2,000,000


Amortization of excess attributable to equipment (1,000,000 / 5 years) ( 200,000)
Net investment income 1,800,000

Fair value of 10% interest 3,500,000


Historical cost 3,000,000
Remeasurement gain 500,000
Net investment income 1,800,000
Total income in 2016 2,300,000

If the investment in associate is achieved in stages the old interest is remeasured at fair value
through profit or loss.

Question 4 Answer A

Total cost 1/1/2016 12,000,000


Net investment income 1,800,000
Share in cash dividend (25% x 5,000,000) ( 1,250,000)
Carrying amount – 12/31/2016 12,550,000

Page 35
SITUATION PROBLEM 5 – PROPERTY, PLANT AND EQUIPMENT

January 1, 2015, an entity disclosed the following balances:

Land 4,000,000
Land improvements 1,300,000

Buildings 20,000,000

Machinery and equipment 8,000,000

During the current year, the following transactions occurred:

* A tract of land was acquired for P2,000,000 cash as a building site.

* A plant facility consisting of land and building was acquired in exchange for 200,000 shares
of the entity. On the acquisition date, each share had a quoted price of P45 on a stock
exchange. The plant facility was carried on the seller’s books at P1,600,000 for land and
P5,400,000 for the building at the exchange date. Current appraised values for the land and
the building, respectively, are P2,000,000 and P8,000,000. The building has an expected life
of forty years with a P200,000 residual value.

* Items of machinery and equipment were purchased at a total cost of P4,000,000. Additional
costs incurred were freight and unloading P100,000 and installation P300,000. The
equipment has a useful life of ten years with no residual value.

* Expenditures totaling P1,200,000 were made for new parking lot, street and sidewalks at the
entity’s various plant locations. These expenditures had an estimated useful life of fifteen
years.

* Research and development costs were P1,100,000 for the year.

* A machine costing P200,000 on January 1, 2008 was scrapped on June 30, 2015. Straight
line depreciation had been recorded on the basis of a 10-year life with no residual value. A
machine was sold for P500,000 on July 1, 2015. Original cost of the machine sold was
P700,000 on January 1, 2012, and it was depreciated on the straight line basis over an
estimated useful life of eight years and a residual value of P50,000.

1. What is the total cost of land on December 31, 2015?


a. 7,800,000
b. 7,600,000
c. 8,000,000
d. 6,800,000

2. What is the total cost of land improvements on December 31, 2015?


a. 1,200,000
b. 3,600,000
c. 1,300,000
d. 2,500,000

3. What is the total cost of buildings on December 31, 2015?

a. 28,000,000
b. 25,400,000
c. 27,200,000
d. 27,000,000

4. What is total cost of machinery and equipment on December 31, 2015?

a. 12,400,000
b. 11,500,000
c. 11,000,000
d. 11,700,000

Page 36

SOLUTION – SITUATION PROBLEM 5

Question 1 Answer A

Land – January 1 4,000,000


Land acquired for cash 2,000,000
Land acquired by issuing shares (2/10 x 9,000,000) 1,800,000
Land – December 31 7,800,000

Quoted price of shares issued for land and building (200,000 x P45) 9,000,000

Current appraized value :


Land 2,000,000
Building 8,000,000
Total 10,000,000
The total cost of the land and building is equal to the quoted price of the shares which is
allocated prorata to the land and building based on the current appraised value.

Question 2 Answer D

Land improvements – January 1 1,300,000


Expenditures for parking lot, street and sidewalks 1,200,000
Balance – December 31 2,500,000

Question 3 Answer C

Buildings – January 1 20,000,000


Building acquired by issuing shares (8/10 x 9,000,000) 7,200,000
Balance – December 31 27,200,000

Question 4 Answer B

Machinery and equipment - January 1 8,000,000


Machinery and equipment purchased 4,000,000
Freight and unloading 100,000
Installation 300,000
Machinery scrapped ( 200,000)
Machinery sold ( 700,000)
Machinery equipment – December 31 11,500,000

END
Chapter 4
Installment Sales
Installment sales problems have appeared very often in the CPA exam. Therefore, candidates
should be familiar with the accounting techniques applicable to this topic.

When a sale is made on the installment basis, the buyer usually makes a down payment and
promises to pay the balance in regular installments over a specified period of time. Profit on
installment sales is recognized only when earned. Although there are several theoretical points
at which the profit can be assumed to be earned, for CPA examinations purposes, the choice is
generally limited to the installment method.

Installment Method

Under this method, income is recognized only when collections are made. Problems requiring
the use of the installment method of recognizing income have appeared quite regularly in the
CPA exam. The following are the typical problems often encountered in the CPA exam:

1. Computation of Gross Profit Rate for each year of sales.


2. Computation of Realized Gross Profit for each year of sales.
3. Computation of Deferred Gross Account balance at the end of year.
4. Computation of Gain or Loss on repossessions.

Computation of Gross Profit Rate

To compute the realized gross profit in proportion to the collections made, it is necessary to
determine the gross profit rate for each year’s operations. The following are the formulas in
computing gross profit rate:

Gross Profit
Current year sales: Gross Profit Rate = Installment Sales

Prior year sales:

Deferred Gross Profit (Beg.) – Prior Year Sales


Gross Profit Rate =
Installment Accounts Receivable (Beg.) – Prior Year Sales

Computation of Realized Gross Profit

Once the gross profit rates are known, it is possible to compute the realized gross profit based
on cash collections. The formula to be used is:
Realized Gross Profit = Collections (excluding interest) x Gross Profit Rate (based on sale)

Missing Factors. In as much as the realized gross profit under the installment method depends
upon cash collections of receivables, it is important that the amounts collected must be known.
However, in some problems, the collections are not specifically stated. Such collections must be
reconstructed from related information available from the data given. The candidate should
remember the following format in computing the collections:

Current Prior
Year Year
Sales Sales
Installment accounts receivable – beginning xx xx
Installment accounts receivable – end (xx) (xx)
Total credits xx xx
Credit for repossessions (unpaid balance) (xx) (xx)
Credit for installment A/C written off (xx) (xx)
Credit representing collections xx xx
Computation of Deferred Gross Profit, End

To compute the balance of Deferred Gross profit at the end of the year, the following formula
may be used:

Installment Account Receivable – End x GPR = Deferred Gross Profit – End


Or
x
Deferred Gross Profit – before adjustment x
x
Less: Realized gross profit x
x
Deferred Gross Profit - End x

Computation of Gain or Loss on Repossession

If a customer does not make an installment payment at the specified time, it is necessary
to repossess the merchandise in order for the seller to minimize his loss.

The gain or loss on repossession is computed as follows:

Fair value of repossessed merchandise xx


Less: Unrecovered cost -
Unpaid balance xx
Less: deferred gross profit (unpaid balance x GP rate) xx xx
Gain (loss) on repossession xx
The fair value of repossessed merchandise at the time of repossession should be
before reconditioning cost and before adding a normal gross profit from sale of
repossessed merchandise.
Trade In

This type of installment sales used by car dealers, whereby an old car is received as down
payment from the buyer for sale of the new car. Usually the old car traded-in is overvalued to
induce the trade-in. for problem solving purposes the overvaluation is computed using a
formula below:

Trade-in value allowed on the old car Pxx


Less: Actual value
Estimated selling price Pxx
Less: Normal gross profit from the sale of used car Pxx
Reconditioning costs xx xx xx
Overallowance on the old car Pxx
The overallowance is treated as a deduction from the selling price of the new car. When there
is overallowance on the old car traded-in, the gross profit rate is computed as follows:

Gross profit ÷ Net Sales (net of overallowance)

The realized gross profit is also computed as follows:

Collections (cash + actual value of old car) x GPR


PROBLEMS

1. Oro Company began operations on January 1, 2012 and appropriately uses the
installment sales method of accounting. The following data are available for 2012 and 2013:

2012 2013
Installment sales P1,500,000 P1,800,000
Gross profit on sales 30% 40%
Cash collections from:
2012 sales 500,000 600,000
2013 sales - 700,000

The realized gross profit for 2013 is:

a. P720,000
b. 520,000
c. 460,000
d. 280,000

2. Roco Corp., which began business on January 1, 2013, appropriately uses the
installment sales method of accounting for income tax reporting purposes. The
following data are available for 2013:

Installment accounts receivable, 12/31/2013 P200,000


Installment sales for 2013 350,000
Gross profit on sales 40%

Under the installment method, what would be Roco’s deferred gross profit at December 31,
2013?

a. P20,000
b. 90,000
c. 80,000
d. 60,000

3. Gray Co., which began operations on January 1, 2013, appropriately uses the
installment method of accounting. The following information pertains to Gray operations for
the 2013:

Installment sales P500,000


Regular sales 300,000
Cost of installment sales 250,000
Cost of regular sales 150,000
General and administrative expenses 50,000
Collections on installment sales 100,000

In its December 31, 2013 statement of financial position, what amount should Gray report
as deferred gross profit?

a. P250,000
b. 200,000
c. 160,000
d. 75,000

4. Filstate Co. is a real estate developer that began operations on January 2, 2013.
Filstate appropriately uses the installment method of revenue recognition. Filstate sales are
made on the basis of a 10% downpayment, with the balance payable over 30 years. Filstate
gross profit percentage is 40%. Relevant information for Filstate first year of operations is as
follows:

Sales P16,000,000
Cash collections 2,020,000
The realized gross profit and deferred gross profit at December 31, 2013 are:

a. P808,000 and P5,592,000


b. 5,040,000 and 808,000
c. 5,600,000 and 808,000
d. 808,000 and 6,400,000

5. Long Co., which began operations on January 1, 2013, appropriately uses the
installment method of accounting. The following information pertains to Long’s operations for
the year 2013:

Installment sales P1,000,000


Regular sales 600,000
Cost of installment sales 500,000
Cost of regular sales 300,000
General and administrative expenses 100,000
Collections on installment sales 200,0000

What is the total comprehensive income on December 31, 2013?

a. P400,000
b. 200,000
c. 300,000
d. 100,000
6. Kiko Co. began operations on January 1, 2013 and appropriately uses the installment
method of accounting. The following information pertains to Kiko’s operations for 2013:

Installment sales P800,000


Cost of installment sales 480,000
General and administrative expenses 80,000
Collections on installment sales 300,000

The balance in the deferred gross profit account at December 31, 2013 should be:

a. P120,000
b. 150,000
c. 200,000
d. 320,000

7. Tayag Corp., which began operations in 2013, accounts for revenues using the
installment method. Tayag’s sales and collections for the year were P60,000 and P35,000,
respectively. Uncollectible accounts receivable of P5,000 were written off during 2013.
Tayag’s gross profit rate is 30%. On December 31, 2013, what amount should Tayag
report as deferred revenue?

a. P10,500
b. 9,000
c. 7,500
d. 6,000

8. Laya Corp., which began operations on January 2, 2013, appropriately uses the installment
sales method of accounting. The following information is available for 2013:

Installment accounts receivable, December 31, 2013 P800,000


Deferred gross profit, December 31, 2013
(before recognition of realized gross profit for 2013) 560,000
Gross profit on sales 40%

For the year ended December 31, 2013, realized gross profit on sales should be:

a. P320,000
b. 340,000
c. 320,000
d. 240,000

9. Dulce Co., which began operations on January 1, 2012, appropriately uses the
installment method of accounting to record revenues. The following information is
available for the years ended December 31, 2012 and 2013:
2012 2013
Installment sales P1,000,000 P1,800,000
Gross profit realized on sales made in:
2012 150,000 90,000
2013 - 200,000
Gross profit percentages 30% 40%

What amount of installment accounts receivable should Dulce report in its December 31,
2013, statement of financial position?

a. P1,225,000
b. 1,300,000
c. 1,700,000
d. 1,775,000

10. On January 2, 2012, Black Co. sold a used machine to White, Inc. for P900,000, resulting
in a gain of P270,000. On that date, White paid P150,000 cash and signed a P750,000 note
bearing interest at 10%. The note was payable in three annual installments of P250,000
beginning January 2, 2013. Black appropriately accounted for the sale under the installment
method. White made a timely payment of the first installment on January 2, 2013, of P325,000,
which included accrued interest of P75,000. What amount of deferred gross profit should Black
report at December 31, 2013?

a. P150,000
b. 172,500
c. 180,000
d. 225,000

11. White Plains, Inc. sells residential lots on installment basis. The following data was
taken from the accounting records of the company as at December 31, 2013:

Installment accounts receivable, January 1 P755,000


Installment accounts receivable, December 31 840,000
Deferred gross profit, January 1 339,750
Installment sales 950,000

Complete (1) the realized gross profit on December 31, 2013 and (2) the balance of the
Deferred Gross Profit account on December 31, 2013.

a. (1) P389,250; and (2) P378,000


b. (1) 427,500; and (2) 389,250
c. (1) 330,750; and (2) 427,000
d. (1) 378,000; and (2) 339,750
12. In August, 2012, Mega World Inc. sold condominium units costing P1,440,000 for
P2,400,000 receiving P350,000 cash and a mortgage note for the balance payable in monthly
installments. Installment received in 2010 reduced the principal of the note to
a balance of P2,000,000. The buyer defaulted on the note at the beginning of 2013,
and the property was repossessed. The property had a fair market value of P1,150,000 at the
time of repossession.

Compute the gain (loss) on repossession if (1) profit is recognized at the point of sale and
(2) gross profit is recognized in proportion to collections.

a. (1) P(850,000); and (2) P(50,000)


b. (1) (850,000); and (2) (450,000)
c. (1) 850,000; and (2) (450,000)
d. (1) (50,000); and (2) 50,000

13. Sarao Motors sells locally manufactured jeeps on installment basis. Data presented
below related to the company’s operations for the last three calendar years:

2013 2012 2011


cost of installment sales P8,765,625 P7,700,000 P4,950,000
Gross profit rates on sales 32% 30% 38%

Installment accounts receivable, 12/31:


From 2013 sales 9,728,125
From 2012 sales 3,025,000 8,387,500
From 2011 sales 1,512,500 4,812,500

On December 31, 2013 how much is the (1) total realized gross profit and (2) deferred gross
profit?

a. (1) P3,044,250; and (2) P4,020,500


b. (1) 3,044,250; and (2) 4,125,000
c. (1) 3,733,750; and (2) 4,020,500
d. (1) 6,993,250; and (2) 4,020,500

14. Polo Company appropriately uses the installment sales method of recognizing revenue.
On December 31, 2013, the accounting records show unadjusted balances of the following:

Installment accounts receivable – 2011 P12,000


Installment accounts receivable – 2012 40,000
Installment accounts receivable – 2013 130,000
Deferred gross profit – 2011 10,500
Deferred gross profit – 2012 28,900
Deferred gross profit – 2013 96,000
Gross profit rates:
2011 35%
2012 34%
2013 32%

For the year ended December 31, 2013, compute (1) total realized gross profit and (2)
the total cash collections in 2013:

a. (1) P182,000; and (2) P135,400


b. (1) 76,000; and (2) 233,000
c. (1) 158,000; and (2) 368,400
d. (1) 106,000; and (2) 97,600

15. Bally Company, which began operations on January 2, 2013 appropriately, uses
the installment method of revenue recognition. The following data pertains to the
company’s operations for the 2013:

Installment sales P1,000,000


Cost of installment sales 500,000
Collections on installment sales 150,000
Installment accounts receivable written off 50,000

What is the balance of Deferred Gross Profit account – 2013 on December 31, 2013?

a. P500,000
b. 150,000
c. 400,000
d. 320,000

16. Nike Company, which began operations on January 5, 2012, appropriately uses the
installment method of revenue recognition. The following information pertains to the
company’s operations for 2012 and 2013:

2012 2013
Sales P300,000 P450,000
Collections from:
2012 sales 100,000 50,000
2013 sales -0- 150,000
Accounts written off from
2012 sales 25,000 75,000
2013 sales -0- 150,000
Gross profit rates 30% 40%
What amount should Nike Company report as deferred gross profit in its December 31, 2013
statement of financial position?

a. P75,000
b. 80,000
c. 112,000
d. 125,000

17. The following accounts appeared in the accounting records of Adidas Sales Company
as of December 31, 2013:

Installment accounts receivable – 2012 P15,000Repossessions P3,000


Installment accounts receivable – 2013 200,000Installment sales 425,000
Inventory, December 31, 2012 70,000Regular sales 385,000
Purchases 555,000Deferred gross profit - 2012 54,000

Additional information:

Installment accounts receivable – 2012, January 1, 2013 P120,00


Inventory of new and repossessed merchandise, December 31, 2013 95,000
Gross profit rate on regular sales 30%

Repossession was made during the year, 2013. It was a 2012 sale and the corresponding
uncollected balance at the time of repossession was P7,200.

Compute (1) the total realized gross profit for 2013 and the (2) loss on repossession:

a. (1) P129,510; and (2) P960


b. (1) 129,510; and (2) 1,464
c. (1) 245,000; and (2) 960
d. (1) 85,500; and (2) 1,464

18. Mango Company, which sells appliances started operations on January 10,2013 operates
on a calendar year basis, and uses the installment method of revenue recognition. The following
data were taken from the 2010 and 2011 accounting records:
2012 2013
Installment sales P480,000 P620,000
Gross profit rates based on cost 25% 20%
Cash collection on 2012 sales 130,000 240,000
Cash collection on 2013 sales 160,000
What is the amount of realized gross profit to be recognized on December 31,2013?
a. P124,500
b. P100,000
c. P92,000
d. P74,667

19. Lacoste Corporation has been using the cash method of revenue recognition. All sales
are made on account with notes receivable given by the customers. The income statement
for 2013 presented the following data:
Revenues – collection on principal P32,000
Revenues – interes 3,600
Cost of goods purchases (includes 45,200
inventory of goods on hand P2,000)
The balances due on the notes on December 31 were as follows:
Notes receivable P62,000
Unearned interest income 7,167
Assuming the use of the installment method of revenue recognition, what is the realized gross
profit on December 31,2013?
a. P16,080
b. P25,586
c. P18,060
d. P43,633

20. Sta. Lucia Realty Corporation sells residential subdivision lots on installment basis. The
following data were taken from the company’s accounting records as of December
31,2013. The company uses a uniform gross profit rate:
Installment accounts receivable:
January 1,2013 P1,510,000
December 31,2013 1,680,000
Unrealized gross profit – January 1,2013 679,500
Installment sales – 2012 1,180,000
Installment sales - 2013 1,900,000

a. P778,500
b. P679,500
c. P756,500
d. P630,500

21. The following information pertains to a sale of real estate by RR Co. to SS Co.
on December 31,2012:
Carrying amount P2,000,000
Sales price:
Cash P300,000
Purchase money mortgage 2,700,000 3,000,000
The mortgage is payable in nine annual installments of P300,000 beginning December 31,2013
plus interest of 10%. The December 31,2013 installment was paid as scheduled,
together with interest of P270,000. RR uses the cost recovery method to account for the sale.
What amount of income should RR recognize in 2013 from the real estate sale and its
financing?
a. P570,000
b. P370,000
c. P270,000
d. P0

22. Action Inc. sold a fitness equipment on installment basis on October 1,2013. The unit
cost to the company was P60,000 but the installment selling price was set at P85,000. Terms of
payment included the acceptance of a used equipment with a trade-in value of P30,000. Cash
of P5,000 was paid in addition to the traded-in equipment with the balance to be paid in ten
monthly installments due at the end of each month commencing the month of sale.

It would require P1,250 to recondition the used equipment so that it could be resold for
P25,000. A 15% gross profit was usual from sale of used equipment. The realized gross profit
from the 2013 collections amounted to
a. P4,000
b. P34,000
c. P10,000
d. P8,000

23. M & J Corp. which sells goods on installment basis, recognizes at year end gross profit on
collections which is consisted of cost and gross profit. It reported the following:
January 1 December 31
Installment receivables
2011 P120,100 0
2012 1,722,300 P337,200
2013 0 2,050,450
Sales and cost of sales for the three years are as follows:
2011 2012 2013
Sales P1,900,000 P2,610,000 P3,010,0000
Cost of sales 1,235,000 1,425,000 1,896,300
In 2013 the company repossessed merchandise with resale value of P8,500 from customers
who defaulted in payments. The sales were made in 2012 for P27,000 on which P16,000 was
collected prior to default. As collections are made, the company debits cash and credits
installment receivable. For default and repossessions, the company debits installment
receivable. The amount of adjustment on the inventory of repossessed merchandise to the
extent of the unrealized gross profit was
a. Zero
b. A decrease of P6,240
c. A decrease of P2,500
d. A decrease of P3,740

24. On October 2013, Haybol Realty Co. sold to Mae Balay a property for P500,000 which is
carried in its books for P250,000. The company received P100,000 on the date of the sale and a
mortgage note for P400,000 payable in twenty (20) semiannual installments of P20,000 plus
interest on the unpaid principal at 16% per annum.

The realized profit to be recognized by Haybol Realty Corp. in 2013 if gross profit is
recognized periodically in proportion to collections would be
a. P50,000
b. P100,000
c. P60,000
d. P250,000

25. Quincy Enterprises uses the installment method of accounting and has the following
data at year-end:
Gross margin on cost 66 2/3%
Unrealized gross profit P192,000
Cash collection including down payments 360,000
What was the total amount of sale on installment basis?
a. P480,000
b. P648,000
c. P552,000
d. P840,000

26. The Brownout, Inc. began operating at the start of the calendar year 2013 uses the
installment method of accounting:
Installment sales P400,000
Gross margin based on cost 66 2/3%
Inventory, Dec. 31,2013 80,000
General and administrative expenses 40,000
Accounts receivable, Dec. 31,2013 320,000
The balance of the deferred gross profit account at December 31,2013 should be:
a. P192,000
b. P96,000
c. P128,000
d. P80,000

27. Tear Drops Corp. started operations on 1 January 2012 selling home appliances and
furniture on installment basis. For 2012 and 2013 the following represented operational details.
In thousand Pesos
2012 2013
Installment sales P1,200 P1,500
Cost of installment sales 720 1,050
Collections on installment sales
2012 630 450
2013 0 900
On 7 January 2013, an installment sale account in 2010 defaulted and the merchandise with a
market value of P15,000 was repossessed. The related installment receivable balance as of date
of default and repossession was P24,000.

The balance of the unrealized gross profit as of the end of 2013 wa


a. P218,400
b. P192,000
c. P360,000
d. P275,000

28. Four J Co. sold goods on installment. For the year just ended the following were
reported:
Installment sales P3,000,000
Cost of installment sales 2,025,000
Collections on installment sales 1,800,000
Repossessed accounts 200,000
Fair market value of repossessions 120,000

a. (P15,000)
b. P15,000
c. (P80,000)
d. P5,000

29. A refrigerator was sold to Fernandina Castro for P16,000, which included a 40% markup
on selling price. She made a down payment of 20%, payment of four of the remaining 16 equal
payment and defaulted on further payments. The refrigerator was repossessed, at which time
the fair value was determined to be P6,800.

The repossession resulted to the following (loss) gain:


a. P(1,040)
b. P1,040
c. P4,056
d. P2,960

30. The Company uses the installment method of accounting to recognize income, Pertinent
data are as follows:
2011 2012 2013
Installment sales P300,000 P375,000 P360,000
Cost of sales 225,000 285,000 252,000
Balances of Deferred Gross Profit at Year end
2011 P52,500 P15,000 P-
2012 - 54,000 9,000
2012 - - 72,000
The total balance of the Installment Accounts Receivable on December 31,2013 is:
a. P270,000
b. P277,500
c. P279,500
d. P300,000

31. In its first year of operations, Guijo Company’s sales were as follows:
Sales basis Mark-up on cost Sales
Cash 25% P250,000
Charge 33-1/3% 400,000
installment 50% 600,000
The cost of goods sold for the year was P900,000.
No. 31 – Continued
If collections on installment sales during the year amounted to P240,000, how much was the
total gross profit realized at the end of the year?
a. P50,000
b. P60,000
c. P80,000
d. P230,000

32. A sale on installment basis was made in 2013 for P8,000 at a gross profit of P2,800. At
the end of 2013, when the installment account receivable had a balance of P3,500, it was
ascertained that the customer would be unable to make further payments. The merchandise
was then repossessed and was appraised at a value of P1,500. The loss on repossession was:
a. P3,500
b. P2,000
c. P775
d. P1,775

33. On January 1,2012 Blim Company commenced its sales of gas stoves. Separate accounts
were set up for installment and cash sales, but perpetual inventory record was not kept. On the
installment sales of a down payment of 1/3 was required, with the balance payable in 18 equal
monthly installments.

The transactions of the Blim Company are as follows:


2012 2013
Sales:
New gas stoves for cash P27,000 P37,000
New gas stoves on installment
(including the 1/3 cash 235,000 330,000
down payment)
Purchases 193,000 215,000
Physical inventories at
December 31:
New gas stoves at cost 45,500 60,000
Cash collections on installment contracts, exclusive of down payments:
2012 sales 54,000 77,000
2013 sales - 70,000
No. 33 – Continued

The realized gross profit for the year 2013 that would be reported on the income statement
amounted to:
a. P131,530
b. P140,000
c. P123,350
d. P131,500

34. The data below are taken from the records of Jess Appliance Co., which sells appliances
exclusively on the installment basis.
2011 2012 2013
Installment sales P365,500 P417,800 P610,750
Gross profit 36% 39% 40%
The balance in the Installment Accounts Receivable controlling accounts at the beginning and
end of 2013 were:
2013
From sales made in: January 1 December 31
2011 P17,400 P-
2012 205,400 25,800
2013 - 305,520
There was one repossession recorded during 2013, it related to a 2012 sale. The repossessed
appliance was sold at its fair value of P200, which equaled the uncollected balance in the
customer’s installment accounts receivable.

The total realized gross profit on prior year sales on December 31, 2013 and the gain (loss)
from the sale of the repossesses appliance are:
a. P76,230 and P(78)
b. P76,230 and P78
c. P69,966 and P78
d. P75,230 and P78
35. Mr. Matias Manuel is a dealer in appliance who sells on an installment basis. A
refrigerator which originally cost P924 was sold by him for P1,650 to Jose Santos who made a
down payment of P220, but defaulted in subsequent payments.
No. 35 – Continued
Mr. Manuel repossessed the refrigerator at an appraised value of P460. To improve its salability,
he expended P60 for reconditioning. He was able to sell the refrigerator to Pedro Reyes for
P1,000 at a down payment of the first installment of P250.

The realized gross profit from the first installment sale (to Jose Santos) and from the second
installment sale (to Pedro Reyes) are:
a. P96.80 and P100
b. P26.40 and P120
c. P96.80 and P120
d. P26.40 and P100

36. The Bengal Furniture Company appropriately used the installment sales method in
accounting for the following installment sale. During 2013 Bengal sold furniture to an individual
of P3,000 at a gross profit of P1,200. On June 1 2013, this installment account receivable had a
balance of P2,200 and it was determined that no further collections would be made. Bengal
therefore repossessed the merchandise. When reacquired, the merchandise was appraised as
being worth only P1,000. In order to improve its salability, Bengal incurred costs P100 for
reconditioning. What should be the loss on repossessions attributable to this merchandise?
a. P220
b. P320
c. P880
d. P1,100

37. Standard Sales Corporation accounts for sales on the installment basis. The balances of
control accounts for Installment Contracts Receivable at the beginning and end of 2013 were:
Jan. 1,2013 Dec. 31,2013
Installment contract receivable - 2011 P24,020 -
Installment contract receivable – 2012 344,460 P67,440
Installment contract receivable – 2013 - 410,090
No. 37 – continued

During 2013, the company repossessed a refrigerator which had been sold in 2012 for P5,400 and P3,200
had been collected prior to default. The company sales and cost of sales figures are summarized below:

2011 2012 2013


Net sales P380,000 P432,000 P602,000
Cost of sales 247,000 285,120 379,260
The resale price of the repossessed merchandise is P2,000 after reconditioning cost of P200 and a normal
gross profit of 35%.

The total realized gross profit on December 31,2013 and the gain (loss) on repossession are:

a. P172,892.5 and P(381)


b. P172,852.5 and P(452)
c. P142,500 and P(452)
d. P142,500 and P452

38. The 680 Appliance Company reports gross profit on the installment basis. The following data are
available:

2011 2012 2013


Installment sales P240,000 P250,000 P300,000
Cost of goods – installment sales 180,000 181,250 216,000
Gross profit 60,000 68,750 84,000
Collections:

2011 installment contracts P45,000 P75,000 P72,500


2012 installment contracts 47,500 80,000
2013 installment contracts 62,500

Defaults:
Unpaid balance of 2011
Installment contracts P12,500 P15,000
Value assigned to repossessed
Merchandise 6,500 6,000
Unpaid balance of 2012
Installment contracts 16,000
Value assigned to repossessed
Merchandise 9,000

No. 38 - Continued
The total realized gross profit after loss on repossession for 2013 is:

a. P49,775
b. P57,625
c. P48,975
d. P56,625

39. Partial trial balance of Lakan Appliance Corporation as of the end of the fiscal year September
30,2013 follows:
Debit Credit
Deferred gross profit – 2012 P50,000
Installment account receivable - 2012 P12,500
Installment account receivable – 2013 150,000
Installment sales 375,000
Inventory, September 30,2012 62,500
Loss on repossession 3,750
Purchases 435,000
Repossessions 2,500
sales 312,500

The post closing trial balance on September 30,2012 shows the following balances of certain
accounts:

Installment contract receivable - 2012 P100,000


Deferred gross profit – 2012 50,000
The gross profit rate on regular sales during the year was 30%

The inventory of new and repossessed merchandise on September 30,2013 amounted to


P75,000. Unpaid balance on repossessed merchandise sale of 2012 is P6,250.

The total realized gross profit on December 31,2013 is:


a. P141,875
b. P101,250
c. P40,625
d. P140,875

40. Carlos Labung Appliance Co., sold a stove, costing P1,000 for P1,600 on September 2012. The
down payment was P160, and the same amount was to be paid at the end of each succeeding month.
Interest was charged on the unpaid balance of the contract at ½ of 1% a month, payments being
considered as applying first to accrued interest and the balance to principal.

After paying a total of P640, the customer defaulted. The stove was repossessed in February 2013. It was
estimated that the stove had a value of P560 on a depreciated cost basis.

The realized gross profit and the gain (loss) on repossession on December 31,2013 are:
a. P232.76 and P(52.07)
b. P240.00 and P(52.07)
c. P232.76 and P(40.00)
d. P240.00 and P(40.00)

41. The Julia Appliance company makes all sales on installment contracts and accordingly reports
income on the installment basis. Installment contracts receivables are accounted for by years.
Defaulted contracts are recorded by debiting Loss on Repossession account and crediting the
appropriate Installment Contract Receivable account for the unpaid balance at the time of
default. All repossessions and trade-ins are recorded at realizable values. The following data
relate to the transactions during 2012 and 2013
2012 2013
Installment sales P150,000 P198,500
Installment contract receivable, Dec. 31:
2012 sales 80,000 25,000
2013 sales 95,000
Purchases 100,000 120,000
New merchandise inventory, Dec. 31 at cost 10,000 26,000
Loss on repossessions 6,000
The company auditor disclosed that the inventory taken on December 31,2013 did not include certain
merchandise received as a trade-in on December 2,2013 for which an allowance was given. The
realizable value of the merchandise is P1,500 which was also the allowance on the trade-in. No entry
was made to record this merchandise on the books at the time it was received. In 2013, a 2012 contract
was defaulted and the merchandise was repossessed. At the time of default, the repossessed
merchandise had a fair value of P2,500. The repossessed merchandise was neither recorded nor
included in the physical inventory on December 31,2013.

The total realized gross profit at December 31,2013 and the adjusted gain (loss) on repossession are:
Realized Gross profit Gain(Loss) on repossesion
a. P70,000 P1,100
b. P70,000 (P1,100)
c. P50,400 P1,100
d. P50,400 (P1,100)

42. Kanlaon Corporation started operations on January 1,2012, selling home appliances and
furniture sets both under cash and under installment basis. Data on the installment sales operations for
the two years ended December 31, 2012 and 2013 are as follows:
2012 2013
Installment sales P400,000 P500,000
Cost of installment sales 240,000 350,000
Cash collections on:
2012 installment contracts 210,000 150,000
2013 installment contracts - 300,000
The balance of the Deferred Gross profit account on December 31,2013 is:
a. P130,000
b. P160,000
c. P190,000
d. P76,000

43. United Trading accounts for sales under the installment method. On January 1,2013 its ledger
accounts included the following balances:
Installment Receivable, 2011 P38,500
Installment Receivable, 2012 155,000
Deferred Gross Profit, 2011 11,550
Deferred Gross Profit, 2012 62,000

Installment sales in 2013 were made at a 42% gross profit rate. December 31,2013 account balances
before adjustments were as follows:

Installment Receivable, 2011 P-0-


Installment Receivable, 2012 42,000
Installment Receivable, 2013 100,500
Deferred Gross Profit, 2011 11,550
Deferred Gross Profit, 2012 62,000
Deferred Gross Profit, 2013 75,810
The total realized gross profit on December 31,2013 is:
a. P90,350
b. P97,510
c. P98,910
d. P97,350

44. Presented below is the unadjusted trial balance, as of December 31,2013 of Moslim Products
Corporation:
Cash P5,000
Installment Accounts Receivable - 2012 40,000
Installment Accounts Receivable - 2013 140,000
Inventory, December 31,2013 200,000
Other Assets 497,000
Trade Accounts Payable P50,000
Unrealized Gross Profit - 2011 10,000
Unrealized Gross Profit – 2012 86,000
Unrealized Gross Profit – 2013 100,000
Capital stock 600,000
Retained Earnings 80,000
Repossession Gain 6,000
Operating expenses 50,000 ________
P932,000 P932,000
The cost of goods sold had been uniform over the years at 60% of sales, and the company adopts
perpetual inventory procedures. On the installment sales, the company charges installment accounts
receivable and credits inventory and unrealized gross profit accounts.

Repossessions of merchandise have been made during 2013 due to some customers’ failure to pay
maturing installments. The analysis of these transactions have been summarized as follows:
Inventory P7,500
Unrealized gross profit - 2011 800
Unrealized gross profit – 2012 2,400
Installment accounts receivable - 2011 2,000
Installment accounts receivable – 2012 6,000
Repossession gain 2,700

The repossessed merchandise were unsold at December 31,2013 and it was ascertained that these were
booked, upon repossession, at their original cost. A fair valuation would be a sales price of P10,000 after
recorditioning cost of P1,000 and a normal gross profit.

The realized gross profit from 2013 sales and the gain (loss) on repossession on December 31,2013 are:
a. P44,000 and (P200)
b. P44,000 and P200
c. P56,000 and P300
d. P56,000 and P200

45. The following selected accounts appeared in the trial balance of Union Sales as of December
31,2013
Debit Credit
Installment Accounts Receivable, 2012 sales P15,000
Installment Accounts Receivable, 2013 sales 200,000
Inventory, December 31,2012 70,000
Purchases 555,000
Repossessions 3,000
Regular Sales P385,000
Installment sales 425,000
Unrealized Gross Profit, 2012 54,000

Additional information:

Installment Accounts Receivable, 2012 sales,


As of December 31,2012 P120,000
Inventory of new and repossessed
Merchandise, December 31,2013 95,000
Gross profit rate on regular sales during the year 30%

Repossession was made during the year on a 2012 sale and the corresponding uncollected amount at
the time of repossession was P7,750.

The total realized gross profit on December 31,2013 and the (loss) on repossession are:
a. P85,5000 and P(1,262.5)
b. P129,262.5 and P(1,262.5)
c. P43,762.5 and P1,262.5
d. P119,622.5 and P1,262.5
46. The books of Paiyakan Company show the following account balances on December 31,2013:
Accounts receivable P313,750
Deferred gross profit (before adjustment) 38,000
Analysis of the accounts receivable reveals the fo llowing:
Regular accounts P207,500
2012 installment accounts receivable 16,250
2013 installment accounts receivable 90,000
Sales on installment basis in 2012 were made at 30% above cost, and in 2013 at 33-1/3% above cost.
Expenses paid relating to installment sales were P1,500.

How much is the total comprehensive income on installment sales?


a. P10,000
b. P10,250
c. P11,000
d. P11,500

47. The Famcor Sales Company employs the perpetual inventory basis in the accounting for new
cars. On August 15,2012, a new car costing P165,000 and with a list price of P220,000 was sold to Rose
Castro. The company granted Ms. Castro an allowance of P85,000 on the trade-in of her old car, the
current value if which was estimated to be P81,700; the balance of P135,000 was payable as follows:
P35,000 cash at the time of purchase and twenty monthly payments of P5,000 starting September 1,
2012.

On April 1,2013, Ms. Castro defaulted in the payment of the March 1,2013, installment. The new car
sold was repossessed, and its value to the seller was P40,000.

The total realized gross profit and the gain (loss) on repossession on December 31,2013 are:
a. P32,616.62 and P(13,298)
b. P32,616.62 and P13,298
c. P37,388.62 and P15,810.62
d. P27,844.62 and P(15,810.62)

48. The Jade Appliances Company started business on January 1,2013. Separate accounts were
established for installment and cash sales. On installment sales, the price was 106% of the cash sales
price. A standard installment contract was used whereby a down-payment of ¼ of the installment price
was required, with the balance payable in 15 equal monthly installment. (the interest charge per month
is 1% of the unpaid cash sale price equivalent at each installment.)

Installment receivable and installment sales were recorded at the contact price. When contracts were
defaulted, the unpaid balances were charged to Bad Debts Expense. The following data are available:
Sales:
Cash sales P126,000
Installment sales 265,000
Repossessed sales 230

Inventory, January 1,2013:


Merchandise inventory 58,060

Purchases, 2013
New merchandise 209,300

Inventories, physical, December 31,2013


New merchandise 33,300
Repossessed inventory 180

Cash collections on installment contract 2013:


Down payments 66,250
Subsequent installments (including interest of P9,252.84 on
all contracts except on defaulted contracts) 79,341
Five contracts totaling P1,060 were defaulted, in each case after 3 monthly installments were paid.

Interest should be recognized in the period earned.

The total realized gross profit on December 31,2013 is:

a. P99,024.85
b. P99,084.87
c. P99,184.85
d. P95,024.85

49. The following data were taken from the records of Camille Appliance Company before its
accounts were closed for the year 2013. The company sells exclusively on the installment basis and its
uses the installment method of recognizing profit:
2009 2010 2011
Installment sales P400,000 P440,000 P420,000
Cost of installment sales 240,000 272,800 256,200
Operating expenses 100,000 94,000 96,000
Balances as of December 31:
Inst. Contracts Receivable -2011 220,000 110,000 28,000
Inst. Contracts Receivable -2012 250,000 92,000
Inst. Contracts Receivable -2013 238,000
During 2013, because some customers can no longer be located, the company wrote off P9,000 of the
2011 installment accounts and P2,800 of the 2012 installment accounts as uncollectible.
Also during 2013, a customer defaulted and the company repossessed merchandise appraised at P2,400
after costs reconditioning estimated at P400. The merchandise had been purchased in 2011 by a
customer who still owed P5,000 at the date of the repossession.

The total comprehensive income on December 31,2013 is:


a. P157,156
b. P61,000
c. P60,156
d. P59,156

50. Jing Trading Company, which started operations on January 2,2012, sells video equipment on
installment terms. Whenever a contract is in default, Jing repossesses the merchandise and writes this
off to a Loss on Defaulted Contracts account. Information regarding the repossessed goods are not
recorded in the books but are kept on a memo basis. Proceeds from the sale of these goods are credited
to the Loss on Defaulted Contracts account. The following information are taken from the books of Jing:
December 31
2013 2012
Installment contracts receivable, 2012 P2,000 P31,500
Installment contracts receivable, 2013 40,000 -
Sales 125,000 75,000
Loss on defaulted contracts 4,275 250
Allowance for defaulted contracts 2,250 2,250
Additional information:
a. No repossessed video equipment was sold in 2012 or 2013 for more than the unpaid
balance of the original contract. A further analysis of the Loss on Defaulted Contracts accounts showed
the following breakdown:
2012 2013
Contracts Contracts
Contracts written off P3,750 P1,500
Less: sales of repossessed goods 800 175
Loss a defaulted contracts P2,950 P1,325

The repossessed goods on hand on December 31,2013, all of which were repossessed from 2012
contracts, are valued at P200.

b. The P2,000 balance of the Installment Contracts Receivable 2012 account is currently due
and collectible.
c. The gross profit rates on installment sales were 40% in 2012 and 42% in 2013.
d. The rate of bad debts loss for 2013 is estimated to be the same as the 2012 experiences rate
based on sales:
The required balance of the allowance for Defaulted Contracts account and the realized gross profit on
December 31,2013 from 2012 sales are:
a. P3,675 and P10,300
b. P3,675 and P9,300
c. P3,675 and P10,300
d. P4,675 and P9,300

ANSWERS

1. C 11.A 21.D 31.D 41.B


2. C 12.A 22.D 32.C 42.D
3. B 13.A 23.D 33.A 43.A
4. A 14.B 24.A 34.B 44.B
5. C 15.C 25.D 35.C 45.B
6. C 16.A 26.C 36.B 46.B
7. D 17.A 27.A 37.B 47.A
8. D 18.D 28.A 38.A 48.A
9. C 19.A 29.B 39.A 49.C
10. A 20.A 30.B 40.A 50.A

SOLUTIONS AND EXPLANATIONS


1. The answer can be computed by using the basic formula, collections x gross profit
rate.
2012 sales 2013 sales
Collections during 2013 P600,000 P700,000
Gross profit rate 30% 40%
Realized gross profit P180,000 P280,000

Total realized gross profit (P180,000 + P280,000) 460,000

2. Installment account receivable, 12/31/13 P200,000


Gross profit rate 40%
Deferred gross profit, December 31,2013 P80,000

3. Installment sales P500,000


Collections 100,000
Installment accounts receivable, 12/31/13 400,000
Gross profit rate (P250,000/P500,000) 50%
Deferred gross profit, 12/31/13 P200,000
Or
Deferred gross profit(P500,000 – P250,000) 250,000
Realized gross profit, 12/31/13 (P100,000x50%) 50,000
Deferred gross profit, 12/31/13 P200,000

4. Realized gross profit (P2,020,000 x 40%) P808,000


Deferred gross profit, 12/31/13:
Installment accounts receivable, 12/31/13
(P16,000,000 - P2,020,000) P13,980,000
Gross profit rate 40%
Deferred gross profit, 12/31/13 P5,592,000

5. Regular sales P600,00


cost of regular sales 300,000
Gross profit on regular sales P300,000
Realized gross profit on installment sales:
Collections P200,000
Gross profit rate (P500,000/P1,000,000) 50% 100,000
Total realized gross profit 400,000
General and administrative expense 100,000
Total comprehensive income P300,000

6. Installment sales P800,000


Cost of installment sales 480,000
Deferred gross profit 320,000
Realized gross profit (P300,000 x 40%*) 120,000
Deferred gross profit, 12/31/13 P200,000
*Gross Profit Rate (P320,000/P800,000) = 40%

7. Installment sales P60,000


Less: Collections P35,000
Accounts written off 5,000 40,000
Installment accounts receivable, 12/31/13 20,000
Gross profit rate 30%
Deferred gross profit, 12/31/13 P6,000

8. Installment sales (P560,000/40%) P1,400,000


Less: installment accounts receivable, 12/31/13 800,000
Collections P600,000
Gross profit rate 40%
Realized gross profit P240,000

9.

2012 Sales 2013 Sales Total


Installment sales P1,000,000 P2,000,000
Collections (RGP/GPR)
During 2010 (P150,000/P30%) (500,000)
During 2011:
2010 sales (P90,000/30%) (300,000)
2011 sales (P200,000/40%) ________ (500,000)
Installment accounts receivable 12/31/13 P500,000 P1,200,000 P1,700,000

10. Deferred gross profit (gain) P270,000


Realized gross profit:
Down payment P150,000
Installment collections excluding interest:
(P325,000 – P75,000) 250,000
Total collections 400,000
Gross profit rate (P270,000/P900,000) 30% 120,000
Deferred gross profit, 12/31/13 P150,000

11. Installment accounts receivable, January 1 P755,000


Installment sales 950,000
Total P1,705,000
Less: Installment accounts receivable, Dec. 31 840,000
Collections 865,000
Gross profit rate (P339,750/P755,000) 45%
Realized gross profit 389,250
Installment accounts receivable, December 31 P840,000
Gross profit rate 45%
Deferred gross profit, December 31 P378,000

12. (1) Profit is recognized at the point of sale

Fair value of repossessed property P1,150,000


Less: Unrecovered cost (unpaid balance) 2,000,000
Loss on repossession P(850,000)

(2) Profit is recognized in proportion to collections

Fair value of repossessed property P1,150,000


Less: Unrecovered cost
Unpaid balance P2,000,000
Deferred gross profit (P2,000,000 x 40%) 800,000 1,200,000
Loss on repossession P(50,000)
13. (1) total realized gross profit
2013 2012 2011
Installment accounts receivable, 1/1/13 P12,890,625 P8,387,500 P1,512,500
Installment accounts receivable, 12/31/13 9,728,125 3,025,000 -0-
Collections during 2013 P3,162,500 P5,362,500 P1,512,500
Gross profit rates 32% 30% 28%
Realized gross profit, 12/31/13 P1,012,000 P1,608,750 P423,500
(Total, P3,044,250)

(2) deferred gross profit, December 31,2013:


2013 2012 2011
Installment accounts receivable, 12/31/13 P9,728,125 P3,025,000 P-0-
Gross profit rates 32% 30% 28%
Deferred gross profit, 12/31/13 P3,113,000 P907,500 P-0-

14. (1) Total realized gross profit


2011 2012 2013
Deferred gross profit before adjustment P10,500 P28,900 P96,000
Deferred gross profit, end:
2011 sales (P12,000 x 35%) 4,200
2012 sales (P40,000 x 34%) 13,600
2011 sales (P130,000 x 32%) ______ _______ 41,600
Realized gross profit, 12/31/13 P6,300 P15,300 P54,400
Total (P76,000)

(2) Total collections in 2013


2011 2012 2013
Installment accounts receivable, beg
2011 sales (P10,500/35%) P30,000
2012 sales (P28,900/34%) 85,000
2011 sales (P96,000/32%) P300,000
Installment accounts receivable, end 12,000 40,000 130,000
Collections during 2013 P18,000 P45,000 P170,000
Total (P233,000)

15.

Installment sales P1,000,000


Collections (150,000)
Accounts written off (50,000)
Installment accounts receivable, 12/31/13 800,000
Gross profit rate (P500,000/P1,000,000) 50%
Deferred gross profit, 12/31/13 P400,000

16. The balance of Deferred Gross Profit Account on December 31,2013 is computed follows:
2012 2013
Sales P300,000 P450,000
Collections (150,000) (150,000)
Accounts written off (100,000) (150,000)
Installment accounts receivable, 12/31/13 P50,000 P150,000
Gross profit rates 30% 40%
Deferred gross profit, 12/31/13 P15,000 P60,000
Total (P75,000)

17. (1) Realized gross profit, December 31,2013


Regular Sales P385,000
Cost of regular sales (70%) 269,500
Gross profit on regular sales (30%) 115,500
Realized gross profit on installment sales (Sched 1) 128,510
Total realized gross profit P245,010

Schedule 1:
2012 2013
Installment accounts receivable, 1/1/13 P120,000 P425,000
Installment accounts receivable, 12/31/13 15,000 200,000
Total credit 105,000 225,000
Less: credit for repossession (unpaid balance) 7,200 -0-
Collections P97,800 P225,000
Gross profit rates:
2012 sales (P54,000/P120,000) 45%
2013 sales (Schedule 2) _______ 38%
Realized gross profit, 12/31/13 P44,010 P85,500
Total (P129,510)

Schedule 2:
Installment sales P425,000
Cost of installment sales:
Inventory, January 1,2013 P70,000
Purchases 555,000
Inventory, December 31,2013 (New) (92,000)
Cost of sales 533,000
Cost of regular sales 269,500 263,500
Gross profit on installment sales P161,500
Gross profit rate (P161,500/P425,000) 38%

(2) loss on repossession

Repossession merchandise P3,000


Unrecovered cost:
Unpaid balance P7,200
Deferred gross profit (P7,200 x 45%) 3,240 3,960
P(960
Loss on repossession )

18. Total realized gross profit on December 31,2013 is computed below:


2012 2013
Collections during 2013 P240,000 P160,000
Gross profit rates on sales 25%/125% 20%/120%
Realized gross profit P48,000 P26,667
Total (P74,667)

19.
Collections during 2013 P32,000
Gross profit rate:
Installment sales:
Notes receivable (P32,000 + P62,000 + P3,600) P97,600
Unearned interest income (P7,167 + P3,600) (10,767)
Installment sales P86,833
Cost of installment sales (P45,200 – P2,000) 43,200
Gross profit P43,633
Gross profit rate (P46,633/ P86,833) 50.25%
Realized gross profit P16,080

20. Collections during 2013 (P1,510,000 + P1,900,000 – P1,680,000) P1,730,000


Gross profit rate (P679,500/ P1,510,000) 45%
Realized gross profit, 2013 P778,500

21 Zero, because the total cost of P2,000,000 is not yet fully recovered. The total collections
applying to principal as of December 31, 2013 is only P330,000 (P300,000 + P30,000), so no income is
yet to be recognized.

22. First the over- allowance on the equipment traded- in should be computed as follows:
Trade- in value P30,000
Actual value:
Estimated sales price 25,000
Less: Reconditioning Cost 1,250
Gross profit(25,000 x 15%) 3,750 5,000 20,000
Over allowance P10,000

The over allowance is treated as a deduction from the selling price of new equipment.
The realized gross profit can now be computed as show below:
Collections
Downpayment:
Cash 5,000
Actual value of Trade- in 20,000 25,000
Installment collection (3 mos. X 5,000) 15,000
Total 40,000
Gross Profit Rate – (15,000/ 75,000) 20%
Realized gross profit, 12/31/2013 8,000

23. the unrealized gross profit relating to the unpaid balance of P11,000 (P27,000-P16,000) is
3,740 (11,000x34%). The inventory of repossessed merchandise is to be decreased by this amount.

24. Collection during 2013 100,000


Gross profit rate (250,000/500,000) 50%
REALIZED GORSS PROFIT 50,000

25. Installment accounts receivable-end:


Unrealized gross profit-end 192,000
Divide by GPR on sales (66-2/3% / 116-2/3%) 40% 480,000
ADD: Collections 360,000
Installment Sales P840,000

26. Installment accounts receivable P320,000


Gross Profit Rate on Sales (66- 2/3% / 166-2/3%) 40%
Deferred gross profit, 12/31/2013 P128,000

27.
In Thousand Pesos
2012 Sales 2013 Sales
Installment sales P1,200 P1,500
Collection:
During 2012 (630)
During 2013 (450) (900)
Repossession (unpaid balance) (24) -

Installment accounts receivable, 12/31/2013 96 600


Gross Profit rate (GP/IS) 40% 30%
Deferred Gross Profit, 12/31/2013 P38.4 P180
Total balance is P218,400 (P38,400 + 180,000)
28. Fair market value of repossessed merchandise P120,000
Less: Unrecovered cost 200,000
Unpaid balance 65,000 135,000

Loss on repossession P(15,000)


29. Fair value of repossessed merchandise P6,800
Unrecovered Cost:
Unpaid balance:
Sales 16,000
Collections:
Downpayment 3,200
Installment 3,200 6,400 9,600
Deferred gross profit (9,600 x 40%) 3,840 5,760
Gain on repossession P1,040

30.
2012 Sales 2013 Sales
Deferred gross profit – Dec.31,2013 P9,000 P72,000
Divide by GPR (GP/IS) 24% 30%
Installment accounts receivable, Dec.31,2013 P37,500 P240,000
Total balance of receivable on Dec. 31,2013 is
(P37,500 + 240,000) P277,500

31. Gross profit rate based on sales:


Cash (25%/125%) 20%
Charge (33-1/3% / 133-1/3%) 25%
Installment (50% - 150%) 33.33%
Total realized gross profit:
Cash sales (250,000 x 20%) P50,000
Charge sales (400,000 x 25%) 100,000
Installment Sales (240,000 x 33.33%) 80,000
Total P230,000

32. Appraised value of repossessed merchandise P1,500


Less: Unrecovered cost:
Unpaid balance 3,500
Less: Deferred gross profit (3,500 x 35% *) 1,225 2,275

Loss on repossession P775


*Gross profit rate (P2,800 / 8,000) 35%

33.
2012 2013
Sales Sales
Collections:
Downpayment (1/3 of sales) - P110,000
Collection of installment receivables 77,000 70,000

Total 77,000 180,000


Gross Profit rate (schedule 1) 44% 45%

Realized gross profit on Installment Sales 33,880 81,000


Realized gross profit on Cash Sales 2013 (P37,000 x 45%) - 16,650
Realized Gross Profit (P131,530) P33,880 97,650

Schedule 1 2013 2012


Sales Sales
Sales: Cash 37,000 27,000
Installment 330,000 235,000
Total 367,000 262,000
Cost of Sales:
Inventories, 1/1 45,500 -
Purchases 215,000 193,000

Total 260,500 193,000


Inventories 12/31 60,000 45,500
Cost of sales 200,500 147,500
Gross Profit P166,500 P114,500
Gross profit rate (GP/IS) 45% 44%

34. P76,230 represents the total realized gross profit based on 2013 collections of
Installment Accounts Receivable of 2011 and 2012 sales.
2011 2012
Sales Sales
Collections:
Installment accounts receivable, 1/1/13 P17,400 P205,400
Installment accounts receivable, - 25,800
12/31/13
Total credits 17,400 179,600
Less: credit for repossession ______ 200

Collections during 2013 17,400 179,400


Gross profit rate 36% 39%
Realized gross proft, 12/31/13 P6,264 P69,966

Total realized gross profit:


(P6,264 + P69,966) P76,230

A P78 gain is realized from the sale of the repossessed merchandise as computed below:
Sales price P200 Unrecovered cost:

Unpaid balance P200


Less: deferred gross profit (P200 x 39%) 78 122
Gain on repossession P78

35. on the first installment, a profit of P96.80 is realized which is computed as follows:
Installment sales P1,650
Cost of sales 924
Gross profit P726

Gross profit rate 44%

Realized gross profit


Collections P220
Gross profit rate 44%
Realized gross profit P96.8
On the second installment, a profit of P120 is realized as shown below:
Sales P1,000
Cost of repossessed merchandise:
Appraised value P460
Add: reconditioning cost 60 520
gross profit P480
Gross profit rate (P480/P1,000) 48%
Realized gross profit:
Collections: P250
Gross profit rate 48%
Realized gross profit P120
36. Appraised value of repossessed merchandise P1,000
Unrecovered cost:
Unpaid balance P2,200
Less: deferred gross profit (P2,200 x 40%) 880 1,320
Loss on repossession (P320)
Gross profit rate (P1,200 + P3,000 ) =40%

37. The realized gross profit is computed as follows:


Year of sales
2011 2012 2013
Installment contract receivable, 1/1/13 P24,020 P344,460 P602,000
Installment contract receivable, 12/31/13 - 67,440 410,090
Total credit 24,020 277,020 191,910
Credit for repossession ______ 2,200 _______
Collections 24,020 274,820 191,910
Gross profit rate:
2011: 133,000/380,000 35%
2012:146,880/432,000 34%
2013:22,740/602,000 ______ _______ 37%
Realized gross profit P8,407 P93,438.8 P71,006.7

Total realized gross profit, 12/31/13:


2011 P8.407
2012 93,438.80
2013 71,006.70
Total P172,852.5

The loss on repossession is computed as follows:

Actual value of repossession merchandise:


Resale price P2,000
Less: Reconditioning cost P300
Gross profit (P2,000 x 35%) 700 1,000 P1,000

Unrecovered cost
Unpaid balance (P5,400-P3,200) P2,200
Less deferred gross profit (P2,200 x 34%) 748 1,452
Loss on repossession P(452)
38. This is computed by deducting the loss on repossession from the total realized gross
profit: Year of Sales
2011 2012 2013 Total
Collections P72,500 P80,000 P62,500
Gross profit rate
2011:P60,000/P240,000 25%
2012:P68,750/P250,000 27.5%
2013:P84,000/P300,000 ______ ______ 28%
Realized gross profit P18,125 P22,000 P17,500 57,625

Loss on repossession
Value of repossessed merchandise P6,000 P9,000
Unrecovered cost:
Unpaid balance 15,000 16,000
Less: deferred gross profit
2011:P15,000x25% 3,750
2012:P16,000x27% _____ 4,400
Unrecovered cost 11,250 11,600
Loss on repossession (P5,250) (P2,600) (7,850)
Total realized gross profit after loss on repossession P49,775

39. The computation is as follows:


Year of sales
2012 2013
Installment contract receivable, 1/1/13 P100,000 P375,000
Installment contract receivable, 12/31/13 (12,500) (150,000)
Total credit 87,500 225,000
Credit for repossession (6,250) -
Collections 81,250 225,000
Gross profit rate (schedule ) 50% 45%
Realized gross profit (P141,875) P40,625 P101,250

Schedule 1 : gross profit rate


2012 sales:
Gross = Deferred gross profit – 2012, 9/30/2012 P50,000 = 50%
profit rate Inst. Contract rec’ble – 2012, 9/30/2012 100,000

2013 sales:

Installment sales P375,000


Less: cost of installment sales-
Cost of goods sold:
Inventories, 9/30/12 P62,500
Purchases 435,000
Cost of goods available 497,500
Less: inventories, 9/30/12
(P75,000-P2,500) 72,500
Cost of goods sold 425,000
Less: cost of regular sales (70% x P312,500) 218,750 206,250
Gross profit on installment sales P168,750
Gross profit rate: (P168,750/P375,000) 45%

40. The realized gross profit is computed as follows:


Collections applying to principal (Sch. 1) P620.69
Gross profit rate (P600/P1,600) 37.5%
Realized gross profit rate P232.76

The loss on repossession is computed below:

Fair value of repossessed merchandise P560


Less: unrecovered cost
Unpaid balance (sch. 1) P979.31
Less: deferred gross profit (P979.31 x 37.5%) 367.24 612.07
Loss on repossession (P52.07)

Schedule 1:

Date (1) Total (2) Applying to (3) Applying to (4) Balance of


payment Interest 005 principal (1) principal (4)
x (4) – (2) – (3)
Sept. 30 P1,600
Sept. 30 P160 - P160 1,440
Oct. 31 160 7.20 152.8 1287.20
Nov. 30 160 6.44 153.56 1,133.64
Dec. 31 160 5.67 154.33 979.31
640 P19.31 P620.69

41. P70,000 is the sum of the realized gross profit in 2012 and 2013 which are computed as follows:
2012 2013
Installment contract receivable, P80,000 P200,000
beg. (1/1/13)
Installment contract receivable, 25,000 95,000
beg. (1/1/13)
Total credits 55,000 105,000
Less: credit for repossession 6,000 -
Collections 49,000 105,000
Gross profit rate (schedule 1) 40% 48%
Realized gross profit 12/31/13 (P70,000) P19,600 P50,400

The P1,100 adjusted loss is determined as follows:


Value of repossessed merchandise P2,500
Unrecovered cost:
Unpaid balance 6,000
Less: deferred gross profit (P6,000 x 40%) 2,400 3,600
Adjusted loss on repossession (P1,100)
Schedule 1 – gross profit rates:

2012 Sales:
Installment sales P150,000
Cost sales:
Purchases P100,000
Merchandise inventory, 12/31 10,000 90,000
Gross profit P60,000
Gross profit rate (P60,000/P150,000) 40%

2013 Sales:

Adjusted installment sales


(P198,500 + P1,500, Trade-in) P200,000
Cost of sales:
Merchandise inventory, 1/1 P10,000
Purchases 120,000
Goods available for sale 130,000
Merchandise inventory, 12/31 26,000 104,000
Gross profit P96,000
Gross profit rate (P96,000/P200,000) 48%

42. The balance of deferred gross profit on Dec. 31,2013 is computed as follows:

2012 2013
Sales Sales
Installment sales P400,000 P500,000
Collections in 2012 (210,000)
Collections in 2013 (150,000) (300,000)
Installment contract receivable, 12/31/13 40,000 200,000
Gross profit rate (GP/IS) 40% 30%
Deferred gross profit, 12/31/13 (P76,000) P16,000 P60,000

43. Deferred gross profit before adjustment:

2011 sales P11,550


2012 sales 62,000
2013 sales 75,810
Total 149,360
Less: deferred gross profit, end (IAR end X GPR)
2011 sales -
2012 sales (P42,000 x 40%) P16,810
2013 sales(P100,500 x 42%) 42,210 59,010
Total realized gross profit, 12/31/13 P90,350
2012 GPR: P62,000/P155,000 = 40%

44. The total realized gross profit is computed below:


2013 Installment sales:
Unrealized gross profit, 2013 P100,000
Divided by GPR on sales ÷ 40% P250,000
Less: Installment receivable – 2013,12/31/13 140,000
Collection from 2013 sales 110,000
Gross profit rate 40%
Realized gross profit on 2013 sales P44,000

The gain (loss) on repossession is computed as follows:

Actual value of repossessed


merchandise:
Sales price P10,000
Less: reconditioning cost P1,000
Gross profit (P10,000 x 40%) 4,000 5,000 P5,000
Less: unrecovered cost
Unpaid balance:
2011 accounts P2,000
2012 accounts 6,000 8,000
Deferred gross profit:
2011 account(P2,000 x 40%) 800
2012 account(6,000 x 40%) 2,400 3,200 4,800
Gain on repossession P200

45. Total realized gross profit is computed below:


Year of sales
2012 2013
Sales Sales
Installment receivable, 1/1/13 P120,000 P425,000
Installment receivable, 12/31/13 (15,000) (200,000)
Defaulted balance (7,750) -
Collections 97,250 225,000
Gross profit rates 45% 38%
Realized gross profit, 12/31/13 P43,762.50 P85,500
Total (P129,562.50)

Gross profit rate:


2012 sales (P54,000/P120,000) 45%
2013 sales P425,000
Installment sales
Cost of installment sale:
Inventory, 1/1 70,000
Purchases 555,000
Inventory, 12/31 (95,000)
Repossession 3,000
Total 533,000
Cost of regular sale (70% x P385,000) 269,500 263,500
Gross profit 161,500
GPR(P161,500/P425,000) 38%

The loss on repossession is computed as follows:


Value of repossessed merchandise P3,000
Less: unrecovered cost:
Unpaid balance P7,750
Deferred gross profit (7,750 x 45%) 3,487.50 4,262.50
Loss on repossession P1,262.50

46.
Deferred gross profit, before adjustment P38,000
Less: deferred gross profit applicable to
Uncollected installment accounts:
2012: P16,250 x 30%/130% P3,750
2013:P90,000 x 25% 22,500 26,250
Realized gross profit P11,750
Less: Expenses 1,500
Net income on installment sales P10,250

47. The computation of the realized gross profit is shown below:


List price P220,000
Less: trade-in overallowance P85,000-P81,700 3,300
Adjusted selling price P216,700 100%
Less: cost of sales 165,000 76.14%
Gross profit 51,700 23.86%
Value of old car trade-in P 81,700
Cash received at time of sale 35,000
Installment collected: P5,000 x4 20,000
Total collections in 2013 136,700
Multiply by gross profit rate .2386
Realized gross profit as of December 31,2013 P32,616.62

Gain (loss) on repossession is computed as follows:


Adjusted selling price P216,700
Less: collections
In 2012 (No.47) P136,700
In 2013: P5,000 x 2 10,000 146,700
Defaulted balance P70,000
Multiply by cost rate .7614
Unrecovered cost P53,298
Value of repossessed car P40,000
Less: unrecovered cost 53,298
Repossession gain (loss) P(13,298)

48.
Cash sales P126,000
Installment sales collected
Downpayment (P265,000 x ¼) P66,250
Subsequent installments P79,341
Less: interest (9,252.84)
Interest on defaulted contracts (sch.1) (20.67) 70,067.49 136,317.49
Total collection P262,317.49
Gross profit rate (sch.2) 37.75%
Realized gross profit, 12/31/13 P99,024.85

Schedule 1 – interest on defaulted contracts:

The total interest is determined through the use of the following table:
Installment (1) Equivalent (2) Contact (3) Interest (4) Cash
number cash sales 1- sales income1% collection
(4-3) price2-4 x1
First month P1,000 P1,060 265
Second month 735 795 7.35 53
Third month 689.35 742 6.89 53
Fourth month 689.35 689 6.43 53
Total interest earned 20.67

Schedule 2 – gross profit rate:

The 37.75% gross profit rate is determined as follows:


Sales: P126,000
Cash sales 250,000
Installment sales at cash 376,000
sales price (P265,000/106%)
Total sales at cash sales
price
Cost of sales:
Merchandise inventory, January 1 P58,060
Purchases 209,300
Goods available for sale 267,360
Less: merchandise inventory, Dec. 31 33,300 234,060
Gross profit 141,940
Gross profit rate (P141,940/P376,000) 37.75%

49.
Total realized gross profit (Sch.1) P157,156
Loss on repossession (Sch.2) (1,000)
Total realized gross profit loss on repossession 156,156
Operating expenses 96,000
Net income, Dec. 31,2013 P60,156

Schedule 1 – realized gross profit

2011 2012 2013


Sales Sales Sales
Inst. Contract receivable, 1/1/13 P110,000 P250,000 P420,000
Inst. Contract receivable, 21/31/13 (28,000) (92,000) (238,000)
Accounts written off (9,000) (2,800) -
Defaulted accounts (5,000) - -
Collections 68,000 155,200 182,000
Gross profit rate (GP/IS) 40% 38% 39%
Realized gross profit (P157,156) P27,200 P58,976 P70,980

Schedule 2 - loss o repossession:

Appraised value of repossessed merchandise P2,400


Less: reconditioning cots 400
Actual value at time of repossession 2,000
Less: unrecovered cost
Unpaid balance 5,000
Deferred gross profit (P5,000 x 40%) 2,000 3,000
Loss on repossession P(1,000)

51. The computation of the required balance of the allowance for defaulted contracts account
is shown below:
2013 Bad debts rate
Loss on defaulted contracts P250
Contracts written off 3,750
Sales of repossessed goods (800)
Value of repossessed goods (200)
Total 3,000
Divided by 2012 sales ÷75,000
Rate of bad debt loss 4%

Estimated loss from 2013 sales (125,000 x 4%) P5,000


Less: loss on defaulted contract – 2013 sales 1,325
Required balance of allowance, Dec. 31,2013 P3,675

The realized gross profit on Dec. 31,2013 from 2012 Sales is computed below:
Installment contract receivable – 2012, 1/1/13 P31,500
Installment contract receivable – 2012, 12/31/13 (2,000)
Installment contract, receivable written off – 2012 sales (3,750)
Collections during 2013 25,750
Gross profit rate – 2012 40%
Realized gross profit from 2012 sales, 12/31/13 P10,300

Вам также может понравиться